Luria’s Autobiography........................................................................................................ 5

Chapter 1......................................................................................................................... 5

Follow on Comment.................................................................................................... 6

Chapter 2......................................................................................................................... 8

Chapter 3 “Vygotsky”.................................................................................................. 11

Luria on research methodology, chapter 3................................................................. 12

Chapter 4: Cultural Differences in Thinking............................................................. 15

Chapter 5: Mental Development in Twins............................................................... 18

Chapter 6: Verbal Regulation of Behavior..................................................................... 21

Luria, Chapter 7............................................................................................................. 24

Luria Chapter 8.............................................................................................................. 27

Summary of Chpt 9.: Mechanisms of the Brain in The Autobiography of Alexander Luria............................................................................................................................... 29

Wk 2: Vygotsky................................................................................................................ 32

Vygotsky, Mind in Society, Ch 4: Internalization of Higher psychological Functions 32

Joanne Price............................................................................................................... 32

From: "david leitch" (dleitch@ucsd.edu)................................................................... 33

From: "Matt Brown" (mjb001@ucsd.edu)............................................................... 36

From: "M Cole" (lchcmike@gmail.com)................................................................... 37

From: "M Cole" (lchcmike@gmail.com)................................................................... 37

L.S. Vygotsky, Mind and Society (1978), Ch. 5 The Problem of Method................... 37

Vygotsky Mind in Society, Chapter 5 Problems of Method........................................ 40

Anna.......................................................................................................................... 40

David Mather:........................................................................................................... 45

From: momoko kashima............................................................................................ 46

Ana Marjanovic-Shane.............................................................................................. 47

Thinking, Sign and Inner Speech.................................................................................... 49

From: gordon wells.................................................................................................... 49

From: "M Cole" (lchcmike@gmail.com)................................................................... 52

From: gordon wells.................................................................................................... 52

From: "Matt Brown" (mjb001@ucsd.edu)............................................................... 53

From: gordon wells.................................................................................................... 54

From: "M Cole" (lchcmike@gmail.com)................................................................... 56

From: gordon wells.................................................................................................... 56

From: "M Cole" (lchcmike@gmail.com)................................................................... 57

Voloshinov Summary.................................................................................................... 58

From: gordon wells.................................................................................................... 58

From: natalia gajdamaschko....................................................................................... 60

Voloshinov From: "Steve Gabosch" (sgabosch@comcast.net)................................. 62

From: "Mary Bryson" (mary.bryson@ubc.ca)......................................................... 65

: "M Cole" (lchcmike@gmail.com)............................................................................ 66

From: "Ana Marjanovic-Shane" (anamshane@speakeasy.net)................................. 66

From: "M Cole" (lchcmike@gmail.com)................................................................... 67

From: "M Cole" (lchcmike@gmail.com)................................................................... 69

From: "Ana Marjanovic-Shane" (anamshane@speakeasy.net)................................. 69

From: "M Cole" (lchcmike@gmail.com)................................................................... 70

From: gordon wells.................................................................................................... 71

From: "M Cole" (lchcmike@gmail.com)................................................................... 73

From: "M Cole" (lchcmike@gmail.com)................................................................... 74

Vygotsky, Thought and Word (Ch 7, Sec 2)................................................................. 74

Vygotsky, Thought and Word, Ch 7 Sec 3.................................................................... 76

Vygotsky, Thought and Word, Ch. 7, section 4............................................................ 78

Nathaniel smith.......................................................................................................... 78

Vygotsky, Thought and Word (Ch.7, Section 5):.......................................................... 78

From: "noga shemer" (nnsevilla@yahoo.com).......................................................... 78

From: momoko kashima............................................................................................ 81

Ana Marjanovic-Shane.............................................................................................. 82

From: "M Cole" (lchcmike@gmail.com)................................................................... 85

From: "M Cole" (lchcmike@gmail.com)................................................................... 85

Mike, Everyone,........................................................................................................ 86

From: momoko kashima............................................................................................ 87

From: "tamara ball" (tball@ucsc.edu)........................................................................ 91

From: "M Cole" (lchcmike@gmail.com)................................................................... 93

From: "Steve Gabosch" (sgabosch@comcast.net)..................................................... 94

From: "M Cole" (lchcmike@gmail.com)................................................................... 94

Wertsch Voices of the Mind  Chapter 2 “A Sociocultural Approach to Mind”........... 94

From: "emma johnson" (ekjohnso@ucsd.edu).......................................................... 94

Comments on Unit of Analysis from I don’t know where........................................... 97

From: "Mike Cole" (mcole@ucsd.edu)..................................................................... 97

Week 3............................................................................................................................. 100

From: "david leitch" (dleitch@ucsd.edu)................................................................. 100

Section 3.2 of Leontiev’s activity consciousness and personality.............................. 102

From: "emma johnson" (ekjohnso@ucsd.edu)........................................................ 102

Leont’ev, Ch 3 - The Problem of Activity and Psychology Section 3.3..................... 104

Joanne Price............................................................................................................. 104

Leontiev – The Problem of activity and Psychology  3.4 The relationship of Internal and External Activity................................................................................................... 106

From: "Robert Lecusay" (rlecusay@ucsd.edu)....................................................... 107

From: "david leitch" (dleitch@ucsd.edu)................................................................. 109

Engestrom, The Emergence of Learning Activity  as a Historical Form of Human Learning (First Lineage)............................................................................................... 110

Antonieta Mercado February 3, 2006..................................................................... 110

Engestrom: Learning Activity article (part 2).............................................................. 115

From: "judy brown" (mmjbrown@connect.carleton.ca)......................................... 119

From: "Ana Marjanovic-Shane" (anamshane@speakeasy.net)............................... 120

Hi Ana and Judy and others!................................................................................... 121

Cole, “Putting Culture in the Middle,” Part 1 (pp. 116-130)..................................... 122

From: "David Mather" (dmather@ucsd.edu).......................................................... 124

Davydov...................................................................................................................... 124

From: "xavier cagigas" (xcagigas@ucsd.edu)............................................................ 124

Steve Gabosh........................................................................................................... 126

From: "Peter Moxhay" (moxhap@portlandschools.org)........................................ 131

From: "judy brown" (mmjbrown@connect.carleton.ca)......................................... 131

From: "Robert Lecusay" (rlecusay@ucsd.edu)....................................................... 132

Week 4 Readings.......................................................................................................... 135

Interobjectivity, Ideality and Dialectics Yrjö Engestrom............................................ 135

by Joanne Price........................................................................................................ 135

Latour “On Interobjectivity”....................................................................................... 138

From: nathaniel smith.............................................................................................. 138

Latour – middle............................................................................................................ 139

From: "xavier cagigas" (xcagigas@ucsd.edu)............................................................ 139

Latour – Final Section: From the study of the soul of society to that of its body..... 140

From: "noga shemer" (nnsevilla@yahoo.com)........................................................ 141

From: Johannes Knigge............................................................................................ 142

From ?...................................................................................................................... 143

From: lars rossen..................................................................................................... 144

From: "Steve Gabosch" (sgabosch@comcast.net)................................................... 144

From: "Donna Russell" (russelldl@umkc.edu)........................................................ 145

From: "Mike Cole" (mcole@ucsd.edu)................................................................... 146

Hutchins: How a Cockpit Remembers its Speed........................................................ 146

From: "emma johnson" (ekjohnso@ucsd.edu)........................................................ 146

Week 5: Links to Pragmatism and Symbolic interactionism.................................... 149

From: "David Mather" (dmather@ucsd.edu).......................................................... 150

From: "Steve Gabosch" (sgabosch@comcast.net)................................................... 151

From: "Robert Lecusay" (rlecusay@ucsd.edu)....................................................... 154

From: "xavier cagigas" (xcagigas@ucsd.edu)............................................................ 156

From: "Steve Gabosch" (sgabosch@comcast.net)................................................... 158

From: "xavier cagigas" (xcagigas@ucsd.edu)............................................................ 161

From: "Steve Gabosch" (sgabosch@comcast.net)................................................... 164

From: "Steve Gabosch" (sgabosch@comcast.net)................................................... 165

From: "Mike Cole" (mcole@ucsd.edu)................................................................... 165

From: "Steve Gabosch" (sgabosch@comcast.net)................................................... 167

From: "Matt Brown" (mjb001@ucsd.edu)............................................................. 171

From: "Mike Cole" (mcole@ucsd.edu)................................................................... 172

From: "Matt Brown" (mjb001@ucsd.edu)............................................................. 173

From: "Steve Gabosch" (sgabosch@comcast.net)....................................................... 174

Hickman article: part I................................................................................................. 178

From: "emma johnson" (ekjohnso@ucsd.edu)........................................................ 178

-Emma...................................................................................................................... 178

From: "Matt Brown" (mjb001@ucsd.edu)............................................................. 181

John Dewey's Pragmatic Technology by Larry Hickman (sec 3)........................... 184

From: "antonieta mercado" (amercado@weber.ucsd.edu)....................................... 184

From: "emma johnson" (ekjohnso@ucsd.edu)........................................................ 188

From: "Matt Brown" (mjb001@ucsd.edu)............................................................. 189

From: "emma johnson" (ekjohnso@ucsd.edu)........................................................ 190

From: "antonieta mercado" (amercado@weber.ucsd.edu)....................................... 191

Xavier's post:......................................................................................................... 191

From: "Steve Gabosch" (sgabosch@comcast.net)................................................... 193

Mike Cole wrote this post that did not make it to the webboard this morning 2/18:................................................................................................................................. 193

From: "xavier cagigas" (xcagigas@ucsd.edu)............................................................ 195

From: "xavier cagigas" (xcagigas@ucsd.edu)................................................................ 197

From Judy............................................................................................................... 199

From: "Mike Cole" (mcole@ucsd.edu)................................................................... 201

From Anna................................................................................................................... 202

From: "Matt Brown" (mjb001@ucsd.edu)................................................................. 203

Overington, Kenneth Burke and the method of dramatism......................................... 205

Overington, “Kenneth Burke and the Method of Dramatism,” Part I (pp. 131-143)................................................................................................................................. 205

From: "david leitch" (dleitch@ucsd.edu)................................................................. 207

Kenneth Burke – Selections from A Grammar of Motives............................................. 210

From: "Robert Lecusay" (rlecusay@ucsd.edu)........................................................... 210

Summary...................................................................................................................... 211

Chpt. 1: Container and Thing Contained..................................................................... 213

Week 6............................................................................................................................. 215

Vygotsky, Mead, and the New Sociocultural Studies of Identity Dorothy Holland and William Lachicotte, Jr.................................................................................................. 215

From: "joanne price" (jprice@activmanuals.com)................................................... 215

Lotman, “The Semiosphere”....................................................................................... 218

From: "joanne price" (jprice@activmanuals.com)................................................... 218

From: "noga shemer" (nnsevilla@yahoo.com)........................................................ 219

From: "Mike Cole" (mcole@ucsd.edu)................................................................... 221

From: "Matt Brown" (mjb001@ucsd.edu)............................................................. 221

From: "Mary Bryson" (mary.bryson@ubc.ca)....................................................... 221

First half of Wertsch Chapter Three........................................................................... 222

David Leitch............................................................................................................ 222

Wertsch Ch. 4, Part 2.................................................................................................. 224

From: "David Mather" (dmather@ucsd.edu).......................................................... 224

Week 7: Dialogic Approaches to Mediation................................................................... 228

R. Engestrom, “Voice as communicative action”......................................................... 228

From: "David Mather" (dmather@ucsd.edu).......................................................... 228

From: gordon wells.................................................................................................. 229

Ritva Engestrom...................................................................................................... 232

From: gordon wells.................................................................................................. 234

From: "Ana Marjanovic-Shane" (anamshane@speakeasy.net)............................... 235

From: "Ana Marjanovic-Shane" (anamshane@speakeasy.net)............................... 235

From: "Ana Marjanovic-Shane" (anamshane@speakeasy.net)............................... 236

Chpt. 4 Middleton and Brown: Virtualising Experience: Henri Bergson on Memory 236

From: "Robert Lecusay" (rlecusay@ucsd.edu)....................................................... 237

Week 8 Emails................................................................................................................. 241

Memory in Madagascar, Jennifer Cole........................................................................ 241

From: "joanne price" (jprice@activmanuals.com)................................................... 241

Brockmeier, “Remembering and Forgetting: Narrative as Cultural Memory”............. 244

From: "noga shemer" (nnsevilla@yahoo.com)........................................................ 244

Middleton and Smith Ch 2 – first half – on Bartlett................................................... 246

From: nathaniel smith.............................................................................................. 246

Middleton & Brown Ch 3 part 1................................................................................. 249

From: "Matt Brown" (mjb001@ucsd.edu)............................................................. 249

From: "Matt Brown" (mjb001@ucsd.edu)............................................................. 250

Middleton & Brown Ch 3 part II................................................................................ 250

From: "xavier cagigas" (xcagigas@ucsd.edu)............................................................ 250

Chpt. 4: Middleton and Brown: Virtualising Experience: Henri Bergson on Memory..................................................................................................................................... 253

From: "Robert Lecusay" (rlecusay@ucsd.edu)....................................................... 253

Boesch, “The Sound of the Violin”............................................................................. 256

From: "noga shemer" (nnsevilla@yahoo.com)........................................................ 256

Week 9 Emails................................................................................................................. 258

Michael Bamberg “Positioning”.................................................................................. 258

From: "antonieta mercado" (amercado@weber.ucsd.edu)....................................... 258

Bruner, Two Modes of Thought................................................................................. 261

From: "xavier cagigas" (xcagigas@ucsd.edu)............................................................ 261

From: "beth ferholt" (bferholt@weber.ucsd.edu).................................................... 263

From: "beth ferholt" (bferholt@weber.ucsd.edu).................................................... 263

Bruner, Two Modes of Thought – 2nd half................................................................. 264

From: "xavier cagigas" (xcagigas@ucsd.edu)............................................................ 264

Stanley and Billig: Dilemmas of storytelling and identity........................................... 266

From: "emma johnson" (ekjohnso@ucsd.edu)........................................................ 266

 

Luria’s Autobiography

Chapter 1

 

A couple of things stick out in the first chapter of Luria's autobiography. One is the impact of the Revolution on Luria's life. Unlike many of his western colleagues, Luria, in his view, lived through times that were very socially and politically tumultuous, and the effect of those times had a significant effect on his development. Luria was caught up in a chaotic whirlwind of ideas that was very exciting but, as he puts it, "not at all conducive... to systematic, scientific inquiry."

The second thing that sticks out, as Mike pointed out, is Luria's ability to make something out of nothing. In complete social and intellectual chaos, Luria was able to find an interesting and diverse set of intellectual roots. In the midst of extreme shortages, Luria was able to found a journal and print it on borrowed yellow soap-paper.

The two opposed elements that helped shaped Luria's life-long psychological project appear in his early education. On the one hand, there was the early laboratory psychology, associated with both Wundt and Høffding, with its focus on highly controlled conditions, basic mental elements, mental associations. On the other hand, there were the critics of laboratory psychology, including Rickert, Windelband, and especially Dilthey. Dilthey focused on a descriptive approach that attempted to understand individual people as they lived and behaved in the world. Luria's career was shaped by trying to find a middle path, escaping the dichotomy between nomothetic and idiographic thinking in psychology.

Nomothetic:  of our relating to the study or discovery of general scientific laws.

Idiographic:  relating to or concerned with discrete or unique facts or events

A few other figures appear as early influences, especially psychoanalysis, William James, and Bekhterev and Pavlov. Luria was especially interested in taking the insights of psychoanalysis and attempting to subject them to more objective analysis.

Follow on Comment

A couple of things stick out in the first chapter of Luria's autobiography. One is the impact of the Revolution on Luria's life. Unlike many of his western colleagues, Luria, in his view, lived through times that were very socially and politically tumultuous, and the effect of those times had a significant effect on his development. Luria was caught up in a chaotic whirlwind of ideas that was very exciting but, as he puts it, "not at all conducive... to systematic, scientific inquiry."

The second thing that sticks out, as Mike pointed out, is Luria's ability to make something out of nothing. In complete social and intellectual chaos, Luria was able to find an interesting and diverse set of intellectual roots. In the midst of extreme shortages, Luria was able to found a journal and print it on borrowed yellow soap-paper.

The two opposed elements that helped shaped Luria's life-long psychological project appear in his early education. On the one hand, there was the early laboratory psychology, associated with both Wundt and Høffding, with its focus on highly controlled conditions, basic mental elements, mental associations. On the other hand, there were the critics of laboratory psychology, including Rickert, Windelband, and especially Dilthey. Dilthey focused on a descriptive approach that attempted to understand individual people as they lived and behaved in the world. Luria's career was shaped by trying to find a middle path, escaping the dichotomy between nomothetic and idiographic thinking in psychology.

A few other figures appear as early influences, especially psychoanalysis, William James, and Bekhterev and Pavlov. Luria was especially interested in taking the insights of psychoanalysis and attempting to subject them to more objective analysis.

 


 

Chapter 2

 

Punchline: Luria made the most of his first professional appointment, designing an experiment that provided objective evidence of subjective mental states.

This chapter outlines Luria's early professionalization and life as an experimental psychologist shortly after the Revolution. In 1923, Luria moved to Moscow at the invitation of Prof. Kornilov of the Moscow Institute of Psychology. Luria claims to have been invited because the institute was in need of experimentally-oriented psychologists open to the materialist lines the Institute was re-orientating itself along in order to conform better to Marxist philosophy. This re-orientation was reflected in the renaming of the individual laboratories to include 'reaction': "All this was meant to eliminate any traces of subjective psychology and to replace it with a kind of behaviorism." (31)

Luria claims ambivalence to this materialist orientation. On the one hand, he believed strongly in objectivity in psychology. On the other hand, the focus on 'reaction' and the distribution of 'mental energy' seemed like an oversimplification of a vastly more complex mental system. In order to address this conflict, Luria devised an experiment that would attempt to incorporate both objective measures of subjects' internal mental states and a more nuanced vision of the human mind. He did so by turning back to his earlier readings in Jungian analysis.

Earlier experimental work had suggested that confusion on the part of a subject was reflected in a disruption of motor tasks; "the smooth curve I usually obtained was distorted in a way that seemed to reflect the subject's uncertainty." (32) The objective reflection of confusion could be combined with psychoanalysis if verbal associative tasks were combined simultaneously with a motor task. If the two occurred simultaneously, then they could be treated as a single functional system, and so distortions in the motor task, which could be measured objectively, could be taken as evidence of internal mental distortion related to the associative task.

Luria worked with Alexei Leontiev (who he had met at the Institute) in the following experiment: research assistants told a story to several subjects, who were told to remember the story, but not to tell anyone that they knew it. Those subjects were then placed into a pool with subjects who had not received the narrative treatment, and were asked to participate in an experiment in which they would squeeze a bulb in their hand (motor function) while responding to a list of seventy words, of which ten were important to the story in the narrative treatment (internal mental distortion), while free-associating (internal mental state). Disruptions of the motor function could then be tracked by exposure to the narrative treatment to determine if an internal mental distortion was taking place; if those subjects told the story reacted differently than the non-treated subjects when the important words were read, then Luria and Leontiev would have evidence regarding their mental states. The experiment was a success, and led to immediate applications in criminal justice.

This application was useful to Luria and Leontiev, because it allowed them access to a population who were experiencing strong, stable emotions: criminals and accused criminals. The presence of strong, stable emotional states was useful to Luria, since psychological experiments frequently had difficulty in producing emotional states that were both strong and stable; strong emotions tended to dissipate. Luria and Leontiev adapted their earlier laboratory experiment to include details of the crime, rather than a story told be assistants.

"One of the first things we discovered is that strong emotions prevent a subject from forming stable, automatic motor and speech responses." (35) This disruption was systemic, rather than localized; this made it difficult to distinguish the criminals from the accused, both of whom had strong, stable emotional states. They finally settled on a process in which the subject was exposed to a variety of stimuli within a single subject. Luria claimed success with this method, although he does not elaborate here on his process for determining what constitutes a subject category.

We see Luria's characteristic modesty from his description of this experiment. Rather than focusing on his ground-breaking ability to find objective measures of subjective mental states, he soft-pedals the experiment, describing it primarily in terms of its use as an early lie-detector.

This modesty may also be the reason that Luria earlier (30) downplays his ability to understand Marxist philosophy, although Mike suggests another possible explanation for this in his excellent introduction. (12) At any rate, the relationship between Luria and Marxism is an important element of this chapter, both in his short discussion on page 30, and also in the nuances of the trend Luria identifies towards objectivity in psychology.
The Autobiography of Alexander Luria


Chapter 3 “Vygotsky”



This chapter opens with the assertion that “… Vygotsky was a genius.” (p38) Luria then describes Vygotsky’s extemporaneous speaking style, for which he required no notes or prepared materials. In class Mike suggested that while this is probably no exaggeration, we need to remember that Luria often underplayed his own contributions, and stood behind others as a way to protect himself from the political violence that was so much a part of his experience. Mike also explained that Luria had a way of being overly self-effacing, in order to signal that, contrary to his modest assertions, he did in fact contribute a lot to the groundbreaking work that he was involved with.

Luria goes on to explain the basic principles for analysis and lab practice that he and Leontiev worked on with Vygotsky. Their focus was to synthesize the two major intellectual tendencies in understanding human psychological processes—coming from the natural and social sciences. What they sought was an experimental practice that could help to uncover the workings of “higher” psychological functioning, an ability to focus on the entire person—not just reflexes, and on the person in a social context. Despite living in difficult circumstances, they worked hard to incorporate and be in touch with a wide range of intellectual influences. Luria asserts that Marxism was the central organizing theory, although also says that he was not an expert Marxist. On Page 42 he talks about Piaget—with whom they had fundamental disagreements, but whose clinical practices were influential for their work.

On Page 44 the three names for this new, synthetic practice are briefly explained—Instrumental, Cultural, and Historical, all three of which were then used in different combinations to describe what they were doing, which was to work from a theory in which “the origins of higher forms of conscious behavior were to be found in the individual’s social relations with the external world.” (p43) “Instrumental” refers to the ability of humans to actively modify their own behavior and cognitive functioning (i.e. tying a string around your finger to remember something). “Cultural” involves the structured way in which society organizes tasks and tools—epitomized in language, the “tool of tools”. “Historical” is not separated from cultural (Mike says “culture in history in the present”). Tools such as language, writing, and arithmetic carry with them the history of the many generations who developed and perfected them.

The rest of the chapter is devoted to describing some of the early experimental work that they did together, which laid the groundwork for the rest of Luria’s career. “The kindergarten and the clinic were equally attractive avenues of approach to the difficult analytic problems.” (p.57, concluding paragraph of the chapter). They studied development and disability/rehabilitation. People in these times function in a less synthetic, holistic way.

This summary will get to be as long as the chapter if I describe all the experiments, but from our class discussion it seems that one really important thing to note is that Vygotsky’s work and theories predicted (helped to found?) the field of neuropsychology—heralding a much more complex understanding of the ways that neurology affects behavior, and the ways that learning and behavior in turn affect the brain. This is detailed on pages 54 and 55.

-Emma

 

Luria on research methodology, chapter 3

I was especially interested in taking a close look at chapter 3, where Luria capsulizes many of the theories and research ideas of the Vygotsky school that he helped found.  I wound up annotating groups of paragraphs in terms of the major themes Luria develops.

Below are my annotations on Luria's pages on research, which Emma could only touch on in her summary (BTW, I liked her summary, and all the summaries, I found them very useful).  What is especially interesting about this section of chapter 3, which Luria devotes to the research work they did in the 1920's, is the emphasis he places on the method of studying the influence of auxiliary stimuli in various kinds of behavior, and how this concept underlaid all their experiments.

Auxiliary:  supplementary

Making a generalization about this, I would say that this method of finding ways of exposing the use of auxiliary stimuli in cognitive behavior (e.g. studying things like the use of auxiliary stimuli to remember primary inform! ation, use of egocentric speech in planning, use of external rules to control motor behavior such as jumping) exemplified the essential theory of the Vygotsky school, that the development of higher psychological functions was a process of internalization - internalization of external activity, particularly social activity and engagement with tools and language.  What each of these experiments had in common was the exposure and analysis of this internalization process by rendering visible the mechanisms involved - the mechanisms of internalizing external, auxiliary stimuli - transforming this stimuli into internal methods (including internal auxiliary stimuli) for the self-guidance of conscious behavior.


pg 45
Experimental arrangements were constantly being explored by the troika.  The arrangements incorporated the theory that as higher psychological processes take shape, the structure of behavior changes.
pg 46
A. R. Leontiev studied how auxiliary stimuli can be used to help remember, distinguishing "natural remembering" from remembering stimulated by the use of “auxiliary stimuli."  The idea of using two sets of stimuli, one that was supposed to be mastered and another to serve as an instrument to do so, became a central methodological tool in all these studies of memory.
pg 47
Natalia Morozova studied the development of complex choices in small children.  The use of auxiliary stimuli was introduced into these choice reaction experiments, and demonstrated that the remembering used in choice reactions required rules governing their acquisition that were similar to those used in mediated remembering that were found in the Leontiev studies.
pg 48
R. E. Levina studied the role of speech in children's problem-solving activity.  The role of egocentric speech in carrying out, and as a child gets older, planning an action, was of particular interest.  The underlying theory of this work was the same as the other studies - words and speech became methods of self-guidance.
pg 49
Alexander Zaporozhets applied this concept of the internalization of external activity (use of egocentric speech in planning, use of auxiliary stimuli to remember primary information) to motor behavior.  In this case, the influence of the use of external rules were applied to children learning how, for example, to control their own jumping.
pg 50
L. S. Sakharov applied this method to the study of how children classify using a block-sorting technique.  He discovered that the naming function of words undergoes deep changes as a child develops.
In another line of studies conducted in 1929, "significative" activity in children was studied - ways that children engaged in activities that gave significance to stimuli they were asked to master.  In these studies, children were asked to invent pictograms to help them memorize abstract phrases such as "The teacher is angry."
pg 51
There were other experiments.; Individually, all these studies were simplistic.  But the concepts behind them laid the methodological foundation for Vygotsky's general theory and provided a set of experimental techniques Luria used throughout his career.
Luria's work on the combined motor method was also an important aspect of the research work done by the Vygotsky school participants.  Although Luria began this work with an interest in studying the dynamic course of emotions, Vygotsky saw the combined motor method as a means for studying the relationship between voluntary movements and speech, especially the way speech served as an instrument for organizing behavior.

Steve Gabosh

 

Cole’s comments
Agree with the centrality of the method of using auxiliary stimuli as THE defining method/theory underlying the cultural historical approach, Steve. It is, not accidentally, the core of Yrjo Engestrom's current intervention research using the phrase, the "method of dual stimulation"
mike

 

 

Chapter 4: Cultural Differences in Thinking

 

Luria begins by noting that one way to study "man's intellectual function" is to compare intellectual activity in different cultures.  He cites speculative work in the area by Durkheim, Janet, Levy-Bruhl, Rivers, and Werner but notes that this work was "being conducted without the benefit of any appropriate psychological data" (59).  He notes that "a few studies on sensory processes, carried out by trained psychologist at the turn of the century, were available" (59) but this data did not address the higher psychological functions. 

 

Luria saw Vygotsky's work as a framework that unified the phenomenological (descriptive) branches and the natural (explanatory) branches.  Given this framework, in the 1930's Luria and Vygotsky planned a study of the "intellectual functions" of adults from "a nontechnological, nonliterate, traditional society".  Luria says they sought a society that was undergoing social and technological change.  They settled on Uzbekistan and Khirgizia in Central Asia, which were regions of the Soviet Union at the time.  Luria describes some of his work in Uzbekistan in this chapter. 

 

The Uzbekistan people he studied were traditionally villages who worked the land of others.  They raised cotton and practiced animal husbandry.  They were Muslims.  Women were isolated from the life of society.  In the period of the study, the Revolution of 1917 (and presumably the subsequent establishment of the Soviet Union in 1922) were causing significant changes in Uzbekistan.  One change was the introduction of schooling, another the "collectivization of agriculture" and another the emancipation of women.   The area was in a state of transition.  Luria studied five groups of people.  1) Illiterate women in remote village who were not involved in the modern ways.  2) Illiterate peasants in remote villages who were not involved with socialized labor.  3) Nearly illiterate women with some training in teaching kindergarteners. 4) Barely literate but experienced collective farm workers. 5) Modestly literate women with training as school teachers (these women had had only two or three years of schooling).  He considered that groups 1) and 2) were not exposed to modern ways but groups 3), 4) and 5) were. 

 

His methods "approached a full-fledged experimental inquiry" (62). Much time was taken to make "subjects" comfortable with the experimenters and the type of tests prior to "testing".  Field notes were taken during testing.  Tests were not standardized but were specially designed to be meaningful to subjects.  Each test had multiple possible answers. The following three higher psychological functions were tested: linguistic coding; categorization and abstraction; and verbal problem solving.  Luria says, "In each of these areas we discovered a shift in the organization of people's cognitive activity that paralled the changes in the social organization of their work lives" (64). 

 

Linguistic Coding:

Subjects differed in the way they named and grouped geometric stimuli.  Subjects who were not illiterate used geometric names like "circle".  Subjects who were illiterate used concrete names like "a plate".  This had consequences for all the other higher psychological functions.  Luria concluded that using geometric names "reflects historically developed and transmitted ways of classifying objects in the world" (66). 

 

Classification and Abstraction:

Luria also called these functions Categorization and Generalization.  "Non-modern" subjects tended to not categorize.  "Modern" subjects categorized in expected ways.  When presented with a set of objects "almost all subjects" tended to group according to practical considerations ("all these objects are needed for chopping") as opposed to using some characteristic of the objects ("all the yellow ones").  For "almost all subjects" grouping was idiosyncractic and resistant to change.  For this same group, grouping by abstract characteristics seemed "stupid" (69).  The tendency to practical grouping was strongest in the illiterate groups, groups 1) and 2).  Mixed results were obtained for groups 3) and 4).  Group 5 tended to employ categorical classification even though they had had only a year or two of school.  These results were understood to mean that "the primary function of language changes as one's educational experience increases.  When people employ a concrete situation as a means of grouping objects, they seem to be using language only to help them recall and put together the components of the practical situation rather than to allow them to formulate abstractions or generalizations about categorical relations" (72). Luria hypothesized that "when people acquired the verbal and logical codes that allowed them to abstract the essential features of objects and assign them to categories, they would also be able to do more complex logical thinking" (74). This was seen in Marxist terms as a "transition from sensory to rational consciousness" (74). Luria believed that theoretical concepts helped by creating a logical system of codes, which he believed develops with thought. 

 

Verbal problem solving/Syllogistic reasoning:

Luria links the same categorization classification approaches to the development of syllogistic reasoning e.g. If one hears "Precious metals do not rust." and   "Gold is a precious metal",  Syllogistic reasoning would lead one to say "Gold does not rust.".  Groups 1) and 2) saw each of the three parts of a syllogism as an isolated judgment even though Luria varied the syllogisms by crafting the statements so that they were about familiar or unfamiliar content.   Groups 1) and 2) (and I think also groups 3) and 4)) tended to not exhibit syllogistic reasoning.  He notes that groups 1) and 2) made "excellent judgments about facts of direct concern to them" and revealed "much worldly intelligence" but in theoretical thinking they were limited.  Three problems presented themselves: a mistrust of initial premises; a failure to accept the premises as universal; and syllogisms disintegrating into three isolated propositions devoid of logical structure. Group 5) responded to logical syllogisms in the expected way. 

 

Luria ends by referring to all the work that was not reported in this chapter -- what he calls the "anti-Cartesian experiments".  He also studied "problem solving and reasoning (not sure if this is distinct from verbal problem solving mentioned above or not), imagination and fantasy, and the ways in which informants evaluated their own personality" (80).  In these studies he found "critical self-awareness to be a product of socially determined psychological development, rather than its primary starting point" (80).  He concludes that "basic changes in the organization of thinking can occur in a relatively short time when there are sufficiently sharp changes in social-historical circumstances, such as those that occurred following the 1917 Revolution" (80).

 

This was a controversial study in its time and I think this is still true today. 

 

Chapter 5: Mental Development in Twins

In the 1930s, Luria was invited to set up a research program at the Medico-Genetic Institute, which gave him unlimited access to twins from all over the USSR. Luria's twin work was different because it did not assess mental functions using the standardized IQ test.  Also, he considered not just biological and cultural explanations for differences between twin performance but also explanations that considered age and the nature of the task being examined.  

 

Study 1:

Luria hypothesized that "As a child grows older, natural processes change quantitatively" (82) meaning that "mental processes increase in power but the basic principles of their action remain unchanged" (82).  However, "Cultural processes, …. change qualitatively" (83) meaning, I believe, that changes occur in the action of culturally-mediated mental processes over time.  Obviously, both natural and cultural processes will be influencing development simultaneously.  Giving memory as an example, Luria notes that "the child gradually learns to organize his memory and to bring it under voluntary control through use of the mental tools of his culture" (83).   Effectively, Luria distinguishes between "natural cognitive tasks" and "culturally mediated processes" and expects that biological factors have a stable relation to the former but that environmental factors have a changing relation to the latter. Luria considered that natural processes dominant up to 7 years and that cultural processes begin to dominant at 11 years. To test this relation he chose subjects who were 5-7 years old and 11-13 years old.  He also selected task that required either natural or cultural psychological processes.  He varied genetic factors by comparing identical and fraternal twins in the same environment.  Fifty pairs of twins were part of the study and they were evenly spread amongst the two age groups and two types of twins. For tasks, Luria used Leontiev's memory tasks, which tested 1) natural direct remembering of geometric figures, 2) the ability to remember very difficult words, and finally 3) the ability to remember a set of words given a set of signs (one per word).  Both quantitative and qualitative data were collected.  Generally, the younger children relied on natural abilities to perform the tasks whereas some of the older children used mediated means, particularly in the third task.  Regardless of the child's age he found that identical twins performed more similarly than fraternal twins on natural tasks (1 and possibly also 2 above).  The culturally mediated task (task 3) was performed similarly whether the twins were fraternal or natural although the older group was more similar than the younger group.

 

Study 2:

Luria also noted that identical twins "presented an interesting opportunity to study the effects of environmental variation" (88) by introducing a cultural difference into one twin's life.  He used educational games developed from a set of blocks to try to develop one twin's imagination, shape discrimination and ability to estimate visually.  Using blocks, he and his colleague Mirenova sought a task that was not too structured and not to open-ended.  They designed a learning activity that would give a child a goal but not the solution.  They gave children a model of a structure that provided the overall shape or outline of the final structure but didn't specify what blocks to use.  Luria felt that building from a partially specified model would develop "perceptual activity" in children (92).  He then asked the ten children in his study (5 pairs of identical twins) to build under the three conditions 1) very structured 2) goal-oriented and 3) open-ended.  Then one twin of each pair followed a two-month programme using strategy 1) above and the other using strategy 2).  When tested at the end of the programme, the twin following programme 2) was superior in building using either the fully-specified models for strategy 1 or the outline models used in strategy 2. Luria asks why and sought an answer by analyzing the model-building process itself.  He noticed that the twin in group 2 planned more, had a better sense of the relation of a block to the whole structure and were more articulate when identifying differences between their structure and the model they were working towards. 

 

Then Luria explored the "basis of the initial differences observed" (93).   He hypothesized those in group 1) above had "only exercised the children's elementary perception" (94).   In further tests he showed both twins in a pair were equally good at discriminating elementary figures and concentrating.  Eliminating these possibilities as explanations for the difference he then tested analytic skills.  Here he found differences between the ability to analyze the relation between "objects and their spatial configuration" (95), where group 2) was superior.  Luria concludes "this research was interesting not only for its general theoretical and educational implications but also for its demonstration that identical twins could serve as especially useful subjects in intensive work with small groups of children" (96). 

 

Study 3:

Luria became interested in the "private" speech of twins.  He studied one pair, Yura and Liosha, who had "a complex phonetic impairment" (97). At four they were speaking very little and interacting mostly with each other.  At five Luria observed their vocabulary was that of a normal two or three year old and their speech was tightly bound to action.  Luria wanted to "modify the level of the children's speech in a short time and by so doing produce a change in their mental functions" (99).  During a ten-month programme the twins lived apart.  Yura, the weaker twin, was "given speech training to discriminate and articulate sounds and to master adult speech" (99).   After three months he found that both boys speech had improved.  Both boys were heard uttering planning speech but Lioshas speech was rarely narrative whereas Yura's was not. Luria concluded "the development of planning speech grows out of the activities and interactions that normally occur in kindergarten, but the development of narrative speech seems to need special training at this stage in the child's language development" (101). However, Luria also noticed throughout the course of the study that Yura's planning and narrative speech was qualitatively different since it could apply to objects and actions that were not in the immediate environment. Luria concluded "the necessity to communicate led to the development of objective speech, but special training was required for the children to produce differentiated, well-developed sentences" (101). However, Luria was really interested in how speech affected children's thought processes.  Following Vygotsky, he focused on assessing the quality of the children's play.  After three months of separation he found "remarkable changes" in free play – games had "an agreed-upon objective" (102). Also, when playing with clay, both boys now worked towards a goal.  However, Yura led play requiring verbal formulations and Liosha led games based on motor activity.  In more detailed observations Luria noticed that Liosha could not label an object.  He had trouble classifying or finding absurdities.  Yura "demonstrated that he had learned to use culturally assimilated schemes to organize his thinking" (103). 

 

I love how the questions and direction of these studies were shaped by cultural-historical psychology.  I also greatly admire how each of the studies progressed to reveal more fundamental differences.  This reflects Luria's remarkable powers of observation.

 

 

 

 

Chapter 6: Verbal Regulation of Behavior

Pavlovian physiology was a popular explanation for animal and human behavior (by the late 1940s), however Luria found its lab models and
experimental methods to be more useful (than its explanatory power). However, Luria found Pavlovian methods to be too reliant on a simplistic,
mechanistic connection between stimuli and response (by over emphasizing techniques of reinforcement and conditioning); also, the Pavlovian terminology of excitation, inhibition, and plasticity could only describe, but not explain, experimental results.

In early 1950s Luria moved to Institute of  fectology, and worked with E.D. Homskaya on, among other things, the issue of how verbal regulation of behavior develop in children (or how children learn to regulate their motor functions based on verbal instructions). [Note: This research is summarized in _Problems in the Higher Nervous Activity of Normal and Abnormal Children_, 2-volumes, pub. 1956 and 1958 in Russian; also, it is
mentioned in _The Role of Speech in the  regulation of Normal and Abnormal
Behavior_, 1960 in English.]

The initial series of three experiments used combined motor method to test both normal of different age groups and abnormal children. All three experiments required the subject respond to visual stimuli (or refrain from responding) based on verbal instructions; the first involved motor
responses, the second required verbal responses, and the third involved both motor and verbal responses. The reason motor and verbal responses
were chosen is because each is presumed to be a lower level function and a higher level function respectively; the reason normal and abnormal
children were chosen is because the normal group provides results concerning developmental processes and the other group provides results
involving pathological conditions.

The experiments involved groups of normal children (2 to 2.5 yrs, 3 to 3.5, 3.5 to 4 yrs) and groups of mentally retarded children (one group
with low level problems and one group with high level problems; both groups somewhat older than the normal children). The results of the normal
children showed that at age 3.5 to 4 subject could begin to control their own behavior based on verbal instructions, while younger children could
not, or could only do so sporadically. The results for the abnormal children showed that those with lower level (motor) problems could use their verbal system to overcome their motor difficulties, while those with higher level (verbal and motor) problems did not improve their performance.

These experiments with verbal regulation of behavior in normal and abnormal children provided results apparently confirming a hypothesis about low and high level brain functioning within both developmental processes and within pathological circumstances. Specifically, one of the key findings of this research involves the internalization of verbal
commands for normal children between 3.5 and 4 years of age. The internalization of verbal  commands in normal children and the use of
verbal commands to overcome motor deficiency in some abnormal children showed that verbal processes are vital to motor coordination and possibly of extreme importance for complex mental functions in general. However, he warns against over-generalizing from these results due to the laboratory context of the experiments, and he suggests further studies involving observation of natural behaviors.


Luria, Chapter 7

 

From: "xavier cagigas" (xcagigas@ucsd.edu)

Luria suggests that there are two strategies toward understanding the nature of higher psychological functions:   tracing their development and following the course of their dissolution under conditions of localized brain damage.  He compares the localization theorists who attempted to relate each mental function to a specific cortical area with the holistic theorists who assumed that the brain functions as a whole to produce the psychological functions expressed in behavior.  Using the dialectical method, he goes on to form a synthesis of narrow localization and equi-potentiality with the strategy of investigating “functional systems.”

 

Using Broca and Wernicke as a foil, he introduces Hughlings Jackson’s view of the brain as a “complex vertical organization.”  Summarizing Jackson’s idea that each psychological function has a low (spinal cord/brainstem), middle (sensory/motor cortices), and high level (frontal/subcortical circuits) representation, Luria introduces the inspiration for what would later become his theory of the “3 functional blocks of the brain” as energizing, processing, and planning in function.

 

Having sketched the scaffolding for functional systems on which he would later draw, Luria launches into detailed descriptions of the functional systems of respiration and movement and the various different physiological ways of achieving the same end.  Luria summarizes the two most salient features of a functional system as:  a) the presence of an invariant task, performed by variable mechanisms, which bring the process to a constant invariant conclusion and b) the complex composition of a functional system always includes a series of afferent (adjusting) and efferent (effector) impulses.

 

Having given these examples in physiology, Luria then dives into Vygotsky’s theory that higher psychological functions represent complex functional systems which are mediated in their structure and incorporate historically accumulated symbols and tools.  Vygotsky assumed that his historical approach to the development of such psychological processes as active memory, abstract thought, and voluntary actions also held for the principles of their organization on a cerebral level.  This followed from the fact that research suggested that the role played by a cerebral region in the organization of a higher psychological process changes during the course of an individual’s development, and also by the fact that in its early stages complex thinking requires a number of external aids for its performance.

 

Luria gives many clinically rich examples of Vygotsky’s theory of higher cognitive functions guiding the rehabilitation of neurologically damaged individuals with Parkinson’s disease by reorganizing the theoretical functional systems using compensatory aids.  This section illustrates the tight coupling between both cortical and extracortical components of the functional systems which subserve higher order cognition and find their nexus in the historically positioned individual.

 

Luria also advances the idea that “Neuropsychology” represents a synthesis of the scientific method characteristic of his training as a psychologist and the detective-like syndrome analysis of behavioral neurology.  In doing so, he challenged the use of “standardized tests of intelligence” which don’t take into account the underlying neurocognitive functional systems and their social historical development, and therefore represent a reified view of “intelligence.”

 

Immediately following the short section on the shortcomings of either taking a purely “psychological” or “neurological” approach to the study of higher order cognition and on the tail of his short comment on intelligence testing, Luria ingeniously uses “aphasia” as a vehicle for deconstructing a reified construct.  By systematically using syndrome analysis to show that there are different forms of “aphasia” and that one must include both the underlying neurodynamic properties as well as the social context in which different symptoms present themselves, Luria fleshed out the unity of the functional systems which drive an activity both in the nervous system and in the social historical world in which individuals are called to act.  In the end, however, he states that perhaps if Vygotsky had lived longer he would have been able to expound upon t! his line of research in more depth.

 


Luria Chapter 8

 

Neuropsychology in World War II

 

During WWII, Luria was commissioned to organize a hospital to treat soldiers with brain injuries.  He started the project in the Urals, but after three years he was transferred back to Moscow where he continued his work after the war ended.  His team was required to diagnose brain lesions and treat complications, as well as develop a rehabilitation program for destroyed functions.  Several key points emerge in his discussion of this experience:

 

Luria emphasizes the close relationship between diagnostic theory and restorative practice (156).

 

His “intermediate” solution to viewing brain functions as either equally distributed or narrowly localized is “to think of the functional system as a working constellation of activities with a corresponding working constellation of zones of the brain that support the activities” (141).

 

The process developed to diagnose and treat patients accords with Vygotsky’s theory that a) higher psychological functions can be analysed by tracing the path of their dissolution, and b) these functions have sociohistorical origins (156).

 

Luria's primary example, which illustrates the above points, is writing – an activity which “clearly could not have been coded in the human brain in a purely organic fashion because it involves the use of man-made tools” (141).  Luria demonstrates that the task of writing involves a complex series of processes involving many different zones of the brain.  The injured part of the brain can be identified by analyzing the particular problem that manifests in the act of writing.  For example, when the tempero-occipital and parietal-occipital regions of the cortex are damaged, the “spatial organization of graphemes is disturbed” (142). 

 

Once the injury was diagnosed, Luria devised a treatment which would use the intact links, as well as external aids, to create a new functional system.  For example, in some patients with afferent motor aphasia, the partially-damaged functional system of articulated speech could be retrained by making the articulatory processes conscious: “to get the patient consciously to produce the sound of the letter p, the therapist gives the patient a lighted match, which he instinctively blows out when the flame reaches his fingers” (145).  His examples of treatment demonstrate the very concrete methods he developed to reconstruct a damaged function, utilizing such external devices as mirrors, index cards, and diagrams.  At the end of the retraining, the patient should be able to function without the external assistance.  While clearly there are variations in the degree of rehabilitation possible depending upon the injury, Luria’s innovative treatments helped his patients improve specific problems, such as articulation and spatial functions, and even more generalized disturbances, such as spontaneous thinking.    


 

Summary of Chpt 9.: Mechanisms of the Brain in The Autobiography of Alexander Luria.



After the Second World War, Luria was concerned with two lines of research: gaining a greater understanding of the neurophysiological mechanism underlying the brain structures associated with the specific syndromes and continuing to advance the general understating of higher cortical functions through the use of more sophisticated psychological analyses. The combination of these two lines of research into a single enterprise is represented, as Luria points out, in the term neuropsychology.

As part of this research project Luria studied the neurophysiology of the frontal lobes and the reticular formation. Research on the reticular formation had revealed that excitation of this structure spread in a gradual manner and therefore played a significant role in regulating activation of the cerebral cortex. This realization led to a shift in research focus from “horizontal” relations within specific levels of organization in the central nervous system (CNS) to “vertical” relations among between structures at the surface of the brain and those deep inside. (159) In addition to this change in research orientation, Luria notes a change in the way researchers understood the direction of excitatory influence in brain activation. Whereas earlier work in neurophysiology focused on the flow of CNS signals moving from lower to higher brain structures, later work highlighted the fact that signals also moved in the opposite direction, with higher structures exerting control over lower ones (afferent (ascending) vs. efferent (descending) connections).

To measure specific and non-specific activation by the frontal lobes of the brain as a whole, Luria and his colleagues employed electrophysiological techniques (electroencephalogram) in combination with tests of verbal comprehension. Participants were first habituated to listening to a list of common words. This elicited nonspecific activation. In order to elicit a specific response some words were imbued with “special significance either through being presented in conjunction with mild electrical shocks, or by asking the participants to listen for specific words (162). In this way Luria and his colleagues could examine their electroencephalogram recordings in order to distinguish between specific and non-specific activation in the brain. Confirmation of the central role of the frontal lobes in activation of the cerebral cortex came from studies demonstrating that frontal lobe patients who received instructions to listen for specific words did not showed any change in brain activation when presented with these specific words.

In the second half of the chapter Luria focuses his discussion on neurolinguistics. He introduces two significant influences on his thinking about the relationship between language and the brain. The first is the work of Trubetskoy who emphasized the importance of phoneme perception in language processing. The second was Saussurian linguistics. From Saussure Luria borrowed the distinction between “paradigmatic” (language as a tool for categorizing) and “syntagmatic” (language syntax) aspects of language and applied this distinction in trying to understand language organization in the brain. Luria and his colleagues found that lesions in specific parts of the cortex differentially affected patients’ abilities to speak fluently or to produce speech with coherent relations between individual words.


Luria ends the chapter by emphasizing the importance of motives and context in the organization of an individual’s speech. He cites Goldstein who argued that naming objects and word repetition did not form the basis of natural speech; rather “the basic form of speech communication is the formulation of ideas as whole propositions which are intimately bound up with the motives and conditions of the activity in which the individual is engaged,” (171).

 

 

 

 

 

 

 

 

 

 

 

 

 

 

 

 

 

 

 

 

 

 

 

 

 

 

 

 

 

 

 

 

 

 

 

Wk 2: Vygotsky

Vygotsky, Mind in Society, Ch 4: Internalization of Higher psychological Functions

Joanne Price


In this chapter, Vygotsky provides a distinction between tools and signs, the primary mediating artifacts that provide the evidence of higher psychological functioning. He believes that an imprecise use of metaphors associated with tools and signs might result in a blurring of the important distinction between them. And since the researcher’s task is “to uncover the real relationship, not the figurative one, that exists between behavior and its auxiliary means” (p. 53), such a blurring would be disastrous. Vygotsky accuses Dewey of making just such an error in referring to language as the ‘tool of tools.’

Of the two, tools and signs, tools have perhaps tended to receive more focus, more investigation since they are external, tangible. But Vygotsky seems to be intrigued with signs, ‘we seek to understand the behavioral role of the sign in all its uniqueness. This goal has motivated our empirical studies of how both tool and sign are mutually linked and yet separate…’

Vygotsky uses three ways to compare and contrast tool and sign:

1. They are similar in that either can play a mediating role in activity.

2. They are different in the ways that they orient human behavior:

a. A tool is externally oriented.
It is a ‘means by which human external activity masters nature.
b. A sign is internally oriented.
It s a ‘means of ‘mastering oneself.’

3. The real tie between these activities: the tie of their development in phylo- and onto-genesis.

‘The mastering of nature and the mastering of behavior are mutually linked, just as man’s alteration of nature alters man’s own nature.

I found this chapter shocking as it applies research in the workplace. There is so very little attention given to signs and their role in the development of activity and yet issues pertaining to ‘signs’ (because they are internally oriented) are perhaps the key stumbling-block to coordinated, distributed activity.

 

From: "david leitch" (dleitch@ucsd.edu)

Vygotsky begins Chapter Four of Mind in Society with a contention that, "the sign acts as an instrument of psychological activity in a manner analogous to the role of a tool in labor."(MiS 52) Here Vygotsky cautions against treating this analogy as an identity. On the one hand, treating the two as identical can lead, in Vygotsky's view, to meaningless expressions pretending to content: "The tongue is the tool of thought," for example. Once someone tries to interrogate this phrase for meaning, its vacuousness becomes clear. On the onther hand, treating sign and tool as identical can lead other psychologists, such as Dewey and other American pragmatists, to forget the important differences between them.

In order to avoid this, Vygotsky reaches back to Hegel's famous aphorism regarding reason:
"Reason is just as cunning as she is powerful. Her cunning consists principally in her mediating activity which, by causing objects to act and react on each other in accordance with their own nature, in this way, without any direct interference in the process, carries out reasons' intentions." (MiS 54 quoting "Encyklopadie, Eter Theil, Die Logik," which Vygotsky draws on from Marx's Capital [199])

Both tools and signs are like reason in that they cause intended actions without any direct interference in the process; they are subcategories of the general categoriy of mediated activity. The difference between them is that tools are used to mediate the physical world, and signs are used to mediate the psychological world. Therefore, tools are physical objects that mediate the physical world, such as wheels, pulleys, levers, and machines, and signs are psychological objects that mediate the mental world, such as mnemonics, gestures, and language. The physical and psychological worlds are analogous but not identical; note even the use of the term 'world' to describe what is psychologically mediated by signs; the term 'world' is itself a physical analogy, as there is no locatable psychological world. A crucial difference between the two is their orientation. The physical world is external to tool-user. The psychological world is internal to the sign-user. Therefore, ! tools orient the user externally and signs orient the user internally. These orientations are different things, but can take place together.

Indeed, this combination is the defining characteristic of higher mental functions. The development of pointing, for example, takes place in Vygotsky's model through the combination of tools and signs. The parent acts as a tool, in that they act upon the physical world to give the child the child reaches for, and the gesture towards the object and the failed attempt to grasp it is the beginning of a sign, in that it acts indirectly upon the world, mediating the child's desires through the rubric of the cultural significance of pointing. Let me explain this in a little more detail.

"We call the internal reconstruction of an external operation internalization." (MiS 56) In order to understand how internalization occurs, we must understand two things: what an external operation is and how it becomes reconstructed internally. Already from this sentence, however, we can see an important difference in Vygotsky's model from Piaget's. For Vygotsky, the higher mental functions begin externally, and move into the child, rather than vice-versa. This model opposes Piaget's conception of development as the increased external expression of internal development. This reversal is central for understanding Vygotsky because it encompasses the uniqueness of Vygotsky's thought. Modelling development as the external entering the internal emphasizes the need for a developmental model to account the transition from the external to the internal. For Vygotsky, this has two important effects, one of which I concentrate on in this account. First, this emphasis leads Vy! gotsky to the mediational aspects of his thought; signs and tools becomes central conceptual objects for Vygotsky because of the work they do in explaining the transition from the external to the internal. Second, Vygotksy's approach to signs and tools as conceptual categories forces Vygotsky to take account of culture in a much more nuanced, central way than previous thinkers had; as we will see, the innate sociability of man is the reason that operations begin externally. This accounting is not just a reaction against the Pavlovian Behaviorism that characterized the mainstream of Soviet psychology. Rather, it is the birth of a new way of approaching psychology, a way that takes an individuals' cultural memberships into account without denying the presence of the individual.

Vygotsky uses the example of a child pointing to outline the process of internalization. At first, a child sees and recognizes an object, and reaches out to grasp it. If the child is successful, the child grasps the object and, given the child's age when they begin to point, will frequently put the object into their mouth. If the child is unsuccessful, the attempt is either witnessed or not. If the attempt is not witnessed, then the child will either locomote over to the object and attempt to grasp it again, or the child will give up. If, however, the failed attempt is witnessed, by a parent for example, then the process of internalization can begin. (MiS 56)

The parent, seeing the child's failed attempt to grasp the object, understands that the child wishes to possess the object. Loving the child, the parent will frequently pass the object to the child. Consider this from the child's point of view. The attempt to grasp the object has succeeded, although through an unexpected means; rather than the gesture sucessfully interacting with the world directly, the gesture successfully acted on the wrold indirectly, mediated through a successful social interaction with the parent. "Consequently, the primary meaning of that unsuccessful grasping movement is established by others." (56) After some time, the child eventually comes to realize the primary meaning that has been established by the parent. At this point, the action of attempting to grasp an object becomes the action of pointing. The movement becomes simplified and oriented towards another person. So long as others respond to the gesture in the way the child has now come! to expect (by fetching the object), the social meaning of the gesture will be reinforced.

See what has happened here. The child no longer acts directly on the world in order to grasp an object, but rather indirectly upon the world through others. The child's interaction with the objective world has become mediated through the cultural convention of pointing. In this way, a cultural meaning has been internalized by the child through a series of three transformations:
An operation that initially represents an external activity is reconstructed and begins to occur internally
An interpersonal process is transformed into an intrapersonal one
The transformation of an interpersonal process into an intrapersonal one is the result of a long series of developmental events

From: "Matt Brown" (mjb001@ucsd.edu)

> Both tools and signs are like reason in that they cause intended
> actions without any direct interference in the process; they are
> subcategories of the general categoriy of mediated activity. The
> difference between them is that tools are used to mediate the
> physical world, and signs are used to mediate the psychological
> world. Therefore, tools are physical objects that mediate the
> physical world, such as wheels, pulleys, levers, and machines, and
> signs are psychological objects that mediate the mental world, such
> as mnemonics, gestures, and language.


I'm a little worried about this inference, but I guess that's because
I'm not sure what you mean by "psychological object." Surely, signs
aren't (always?) psychological objects in the sense of ideas or
mental representations. Traffic signs, words on a page, the sign on
the bathroom door, gestures and spoken words are all signs, but they
likewise have a material aspect. Similarly, tools also have an ideal
side. They aren't merely physical objects, because they are what
they are in virtue of having a history and being part of a culture.

From: "M Cole" (lchcmike@gmail.com)

These topics will be central to discussion today, David. Thanks for the
summary.
mike

From: "M Cole" (lchcmike@gmail.com)

I share your dis-ease Matt.
mike


L.S. Vygotsky, Mind and Society (1978), Ch. 5 The Problem of Method


Antonieta Mercado January 27, 2006

Problems of Method

Vygotsky argues that new approaches to scientific problems entail a
reevaluation of methodology. This has been the case in psychology, and th=e
introduction of stimulus-response experimentation was revolutionary at it=s
time, because it brought psychology closer to the natural sciences (p.
59).

Wundt saw the very essence of psychological method as the systematic
alteration of the stimuli that generate a change in the psychological
process linked to them (p. 59).


However, Wundt experimental method only applies to elementary processes o=f
psychophysiological character (p. 60). Higher psychological functions wer=e
out of the picture.

According to Vygotsky, experimental psychology has imposed the
stimulus-response framework as the main tool to study behavior and, he
argues, this approach cannot serve as the basis for the adequate study of
the higher, human forms of behavior (p. 60), because psychological
functions are processes not objects, and the stimulus-response framework
“fossilizes” a dynamic process.
    

Vygotsky talks about a method to assess higher mental functions as:

The search for method becomes one of the most important problems of the
entire enterprise of understanding the uniquely human forms of
psychological activity. In this case, the method is simultaneously
prerequisite and product, the tool and the result of the study (p. 65).

Vygotsky proposes three principles for the analysis of higher mental
functions and for the introduction of a developmental psychological
method, rather than only a experimental one. Those principles are:

a) Analyzing processes, not objects. Any psychological process
(development of thought or voluntary behavior) is undergoing constant
changes (even during experiments).

b) Explanation vs. description. Vygotsky cites K. Lewin when he contrasts
phenomenological analysis (based on phenotypes or external features), and
genotypic analysis (origins of phenomena). Vygotsky provides the example
of a whale: phenotypically a whale is a fish, but genotypically a whale i=s
a mammal. Thus, phenotypic viewpoints are descriptive in nature, whereas
genotypic viewpoints are explanatory (p. 62). “Two processes that are v=ery
close in their causal-dynamic nature may e very different phenotypically”
(p. 65).

Vygotsky has touched an important point for scientific inquiry: there is
mystery in the world, and science tools are out to discover, explain and
describe those mysteries. If phenotypical characteristics were the same a=s
genotypical, there would be no basis for scientific inquiry. Experience
solely would provide with knowledge about reality and nature.

c) The problem of fossilized behavior. Studying processes that have
already died. Examples of this are mechanized psychological processes
(some with ancient origins), that are repeated over and over (such as the
case of reflexes). Vygotsky posses that “inactive, rudimentary function=s
stand not as the living remmants of biological evolution, but as those of
the historical development of behavior (p. 64). In child development, thi=s
has been a big misunderstanding, because researchers have equated
historical as only past events, not processes that are continuously going
on before our eyes.

 

Vygotsky argues that in order to understand behavioral processes there is
no need for the mechanical decomposition of responses into their elements.
However, he argues that there is a risk that by not studying the elements
of a particular process, all what is left is a description of external
responses rather than an assessment of the dynamic process (refer to page
66 and the equation).

Vygotsky proposes a causal-dynamic study of choice reactions, where
complex reactions are studied as a living process, not as fossils. He
exemplifies it with an experiment on choice reactions in adults and
children. He observes that, when cued to push a button or respond to a
stimulus, adults acquire a stable response, while children generally do
not wait until directions are given and start a process of trial and erro=r
until they associate the correct way to approach a task. In doing this,
children may use external objects as signifiers, something that does not
have direct relation to the task. In this process, learning occurs as an
internal stimuli, that is hard to assess in a linear way, because it does
not occur linearly, but in leaps and set-backs. A experimental psychology
based solely on stimulus-response is unable to capture this very process
of learning that entails revolutions and evolutions.

 

Vygotsky Mind in Society, Chapter 5 Problems of Method

 

Anna

Hi all,
I’m too finally joining the inspiring discussion here!

Antonieta lists in her summary three main principles of Vygotsky’s genetic or developmental method for studying the higher mental functions. Of these I take up the two, focusing on process & explaining instead of describing.

This relates to what Steve and Mike pointed out in the last week’s Luria discussion: to the use of auxiliary stimuli as a central principle in cultural-historical research and its implications for practical research. Inspired by this, I am thinking of the following question:

What it means to apply Vygotsky’s principles of his method in the ethnographically oriented developmental research today?

The empirical research settings of Vygotsky and colleagues were mostly conducted in laboratory settings. One could say that these arrangements were often separate from real life contexts and set up for a research purpose only. However, since that teh focus has been on researching these similar issues in real life settings such as after-school clubs, workplaces, classrooms etc. Often in these cases the role of the researcher is something between an ethnographer and interventionist...

In these cases the empirical object of research is emerging and impossible to control and often very heterogeneous (a group of people developing their own work or a group of students and teachers developing their learning activity with the researcher). Also, what to focus on in an empirical analysis is hard to determine before the whole process is over - although development and change is what is often seen as the main focus of the study. And finally, the role of the research subjects themselves becomes crucial in this.

I find it challenging to think what actually does Vygotsky’s genetic method mean in these kinds of settings? And how to trace development so that one does not only describe what happened but can also trace the critical turning points, relations, etc., in which the subjects or the activity developed? (depending whether to concentrate on studying the development of activity or the development of an individual in that activity.. or both..)

Well, this is a very large topic and must be solved in each research separately. However, it would be interesting to hear how you people have struggled these issues in your own research settings / analyses?

Also Engeström discusses this issue in his paper presented in ARTCO conference in Lyon 2005, where he refers i.e. to van der Veer and Valsiner and the challenge Vygotsky himself was also very aware of:

“The notion of ‘experimental method’ is set up by Vygotsky in a methodological framework where the traditional norm of the experimenter’s maximum control over what happens in the experiment is retained as a special case, rather than the modal one. The human subject always ‘imports’ into an experimental setting a set of ‘stimulus-means’ (psychological instruments) in the form of signs that the experimenter cannot control externally in any rigid way. Hence the experimental setting becomes a context of investigation where the experimenter can manipulate its structure in order to trigger (but not ‘produce’) the subject’s construction of new psychological phenomena.” (van der Veer and Valsiner 1991, p. 399)

Who is to control what auxiliary stimuli is used and manipulated and in what way? This is first of all a question of agency…

Best wishes for all,
Anna


Vygotsky, Thought and Word Ch.7 Thinking, Sign and Inner Speech, Section 1:
Vygotsky Chapter 7 Thought and Word Sec 1

 

In this chapter Vygotsky begins by exploring the relationship between thought and word using a genetic approach.  He pursues two genetic paths:  phylogenetic (development of the species) and ontogenetic (development of the individual). He looks at the relationship between thought and word in the very early stages of the development of our species (the anthropoid stage) and the very early stages in the development of the person (childhood) (243).  He asserts that we do not come to be able to think and speak as a result of a skill that has developed over time in our species but that each generation acquires thinking and speaking skills in the course of their development as individuals, and, more specifically, as they develop human consciousness (243). (Here, I'd love to know what Vygotsky meant by human consciousness – I can only guess because I understand that the Russian concept of consciousness is very different from Western conceptions that no doubt invade my head).

 

Vygotsky says that in anthropoids thought and speech (meaning, I think, vocalizations) are not connected.  Also he says that in very young children you can observe speech without thought and also thought without speech (243).  So, Vygotsky asks, when and how do thought and speech become linked and what is the nature of the relationship between thought and speech over time?

 

One common pitfall he warns is thinking about thought and word as separate (243).  This happens especially when some researchers decompose verbal thinking (what Vygotsky calls the object of their study) into the two elements thought and speech, which individually do not possess the characteristics of verbal thinking.  Here he is making a point about the appropriate unit of analysis for studying verbal thinking.  He opposes decomposition into elements as an analytic technique that will reveal anything about the entire object of study. 

 

His approach is to partition verbal thinking into units (244).  Units are produced by the analytic process but their characteristics "relate to the whole" – this means that Vygotsky is seeking a unit that will relate to the object of study, verbal thinking, but is simpler.  The unit he suggests in this case is 'the meaning of the word' because 'the meaning of the word' is present in both thinking and speech.     He says speech without meaning is just sounds and that thinking produces meanings.  Vygotsky claims the relationship between speech and thought is dialectical and complex (my words).  Meaning is a phenomena of verbal thinking only if speech is connected with thought and thought is connected with the word (244). 

 

Having established his case for 'the meaning of the word' as the unit of analysis for studying verbal thinking, Vygotsky takes a developmental perspective and looks at how this unit changes ontogenetically.  In the rest of the section he critiques approaches that others have taken. 

 

He says word and meaning are not developed through associations that can only be reinforced or weakened.  Here I think he takes issue with word and meaning being related by an association – he believes this is not the nature of the relation.  He believes the field of semantics misses that the semantic structure of the word meaning relation changes over the development of the person's lifetime.  So, he says, initially we have primitive forms of generalization that relate word and meaning and later we have more complex forms (245).  Then he makes the link with consciousness saying that "reflection and generalization of reality [consciousness] changes with the emergence of abstract concepts in the process of the historical development of language [the child learning to speak]".  (245). 

 

For Vygotsky, the word has a place in the inner life of  thought (246), an assertion which challenges the thinking in the Wurzburg school and others of the same ilk. Vygotsky takes issue with the notion that there is a point when a concept is formed. What is incorrect with the notion, Vygotsky claims, is the assumption that "the initial moment and end point in the process of concept development coincide". (247)

 

Structural psychology, he says attempted to "remove not only thinking but speech from the domain of associative laws" (247).  Words in this framework are seen as functional (like a stick is functional) but still separate from thought.  Vygotsky has two other critiques of structural psychology.  One is that this approach doesn't bring out the unique characteristics of words – how they're different from sticks, what their structure is, how words represent things in consciousness and so on and it doesn't, like associative psychology, allow for the semantic structure of the word meaning relation to change over a person's lifespan.

 

So Vygotsky sees the associative approach and structural psychology as "like identical twins".  The words may be different but the thinking has not changed. Both assume that thought is independent of the word (248).  None of the approaches he reviews in part one "has grasped what make the word a word (249). "All have overlooked the generalization that is inherent in the word, this unique mode of reflecting reality in consciousness." (249) and they "consistently analyze the word and its meaning in isolation from development" (249).  

 

Section 1 is essentially a critique of the treatment of thought and language at the time and a suggestion for an alternative starting point for the study of verbal thinking.  We see here Vygotsky's preference to view research problems from a genetic prospective and his quest for an analytic approach that identifies units that reflect the whole.

 

Judy

 

Judy Brown


David Mather:

He says there is no primal bond between thought processes and language, but that a connection between them grows, changes and develops. This connection can be analyzed by using word meaning as the unit of analysis, since it is both spoken (external) and thought (internal). His analysis confirms that word meaning provides concrete measure of the development of verbal thinking, but, more importantly, it found that word meanings change and develop.

This new view of semantic structure (via individual development) contradicts other attempts to study language, such as the Wurzburg school, Selz (?), Ach (?), and Gestalt psychology. These other prevailing theories had two fundamental problems: they believe that words are connected to meanings via internal associations and that the all word meanings use the same structure. His theory overcomes those two errors by viewing thought and language as a developmental process that can account for different semantic structures.

[His examples of erroneous methods were quite extended no doubt due to the contentious nature of the historical context. He spends so much time framing his critique of their theories that he only barely begins to describe his own theory. Thus, my synopsis is rather concise.]

From: momoko kashima

 Hi all. My name is Momoko KASHIMA. I'm a graduate school student in Japan.

 First, I would like to comment on this chapter pertaining to my study theme.

 I've been working on children's play, especially on how they create a shared-new-meaning through their ongoing interaction. As Vygotsky says:

 The semantic structure has three basic characteristics: -- predominance of the word’s sense over its meaning, of phrase over word, and of the whole context over the phrase (277) -- agglutination (277) -- word sense is characterized by different laws of unification and fusion. Words are more heavily laden with sense than in external speech -- words are “a concentrated clot of senseE(278)

What kind of process happens for a word to get clot of sense? Right now, I'm analyzing this process by focusing on the context of ongoing interaction; toys,spaces and gestures.

Second, I'm interested in this comment:

The path of verbal thinking can thus be represented as: motive --> formation of thought --> mediation in internal word --> meaning of external words --> to the words themselves. or, if I understood correctly: motive --> formation of thought --> [Realm of Inner Speech: internal mediation of thought, first by meanings and then by words] --> to the words themselves

 In my understanding, word and thought are inseparable, so I guess it should be not separable process but more complicated. Although, I can't describe it in details.

That's all my comments for now.

Ana Marjanovic-Shane

 

Hi,
My name is Ana Marjanovic-Shane. I am one of the "external" non-UCSD
students in this course.

I would like to make a comment on Momoko's comments on Vygotsky's
Thought and Language, chapter 7. (I will address Momoko directly,
creating a dialogue, although my comments are directed to everyone else,
too).

It is interesting that you chose to focus on the part of the chapter
where Vygotsky analyzes the so called "inner speech". The semantic
structure you mentioned, where Vygotsky talks about predominance of the
word's sense over its meaning, agglutination and different laws of
unification and fusion when it comes to "sense"-- are all about
characteristics of inner speech. What is interesting is that you see a
connection between inner speech and play.

The most puzzling and least examined part of this chapter, in my opinion
is the analysis of "meaning" and "sense" as two different aspects or,
layers of meaning making. Vygotsky was partly inspired by Paulhan's
views on the relationships between word (phonetic aspect of language),
its meaning and sense. Vygotsky saw those three "layers" of language as
being in a constant dynamic process of change and simultaneously
changing the relationships between them. In his words:

"The relation of thought to word is not a thing but a process, a
continual movement back and forth from thought to word and from word to
thought. In that process the relation of thought to word undergoes the
changes that themselves may be regarded as development in the functional
sense. Thought is not merely expressed in words; it comes into existence
through them." (p.218)

In this chapter, Vygotsky's view of "sense" is somewhat puzzling in
itself. It is, at the same time, something, which is most characteristic
of inner speech ("In inner speech, the phenomenon reaches its peak. A
single word is so saturated with sense that, like the title "Dead
Souls", it becomes a concentrate of sense. To unfold it into overt
speech, one would need a multitude of words." p 247). Therefore, it
seems to be the most private zone of meaning. But, at the same time,
sense is that aspect of language which is the most dependent on the
immediate context ("A word acquires its sense from the context in which
it appears; in different contexts it changes its sense" p. 245).
Therefore it appears that sense is a function of dialogue, of
interaction, of situation, much more so than "meaning". Meaning is the
most stable "zone of sense" ("Meaning remains stable throughout the
changes of sense. The dictionary meaning of of a word is no more than a
stone in the edifice of sense, no more than a potentiality that finds
diversified realization in speech" p. 245).

The core of this discussion is, of course, the phenomenon of polysemy -
a capacity of a sign to have multiple meanings. If we agree with
Vygotsky that the relationship between word and thought (or word and
meaning) is a dynamic process, rather than a constant thing, then, the
question is, really "How is it possible for word to even develop a zone
of "constant" meaning?" In other words, if polysemy is the norm, then
how does a language ever acquire its "stable" zone of universal meanings.

What is interesting is that you chose to touch on this in the context of
studying play, especially how children "create a shared-new-meaning
through their ongoing interaction". I would like to know if you saw a
difference in the relationship between word and thought in play, and
their relationship out of play. And what are the criteria or the
phenomena that you take into account as critical when studying new
meaning making in play.

"That concludes my turn" (a phrase that in my inner speech has more
sense to me than you know :-) ).
Ana

Thinking, Sign and Inner Speech

 

From: gordon wells

When I first read Thought and Language in 1967 I was very excited by
Vygotsky's discussion of inner speech. I still think the 'concept' of
inner speech makes a lot of intuitive sense. But more recently I have
begun to worry about what V. means by 'thought' (or preferably
'thinking'), both in the ontogenetic developmental progression he
proposes and in his account of thinking and inner/outer speech in
Ch.7.

What exactly is pre-speech thinking? Is it essentially similar to the
thinking that might be attributed to other species, from dogs to
chimps; in other words, is preverbal thinking a 'lower mental
function', triggered by perception and motivation and leading to
external action of some kind? With the emergence of deliberate
communication in the first year of life (through gestures, systematic
vocalizations, etc.), the infant becomes able to get others to act
on her/his behalf and to share an interest in some aspect of the
environment. These are in fact the first functions that appear in the
infant's 'protolanguage' (Halliday, 1975) and, although not yet
'language', these are signs that seem to begin the process of
thinking together.

With the development into his/her first language, the child gradually
acquires both lexicon and grammar, making it possible to communicate
information as well as desires, and feelings. This is clearly what V.
is referring to when he writes about the beginning of verbal thought,
in which grammatical functions as well as word meanings develop as
the child engages in interaction with more mature speakers about
their shared situations. My question at this point is: How should we
think about the child's thinking? Is preverbal thinking changed as a
result of its interrelation with speech or does the preverbal
continue more or less before - perhaps at a level below conscious
awareness - while, at the conscious level thinking is essentially
sign-mediated thinking? In M&S Ch.3, V. talks about the natural
history of sign as being critical for an understanding of the
relationship between "two qualitatively different lines of
development," the biological and the sociocultural. But as has often
been mentioned, V. did not pursue the developmental nature of that
development. By the time he wrote T&L Ch.6, the relationship he
explored was between spontaneous and academic concepts. But both of
these are based in linguistic signs. So, is there any other sort of
thinking?

V puts the development of inner speech at the age of 5 years or so
and he argues that, as inner speech develops, it becomes both a means
of controlling one's own behavior and of mediating thinking. I am
happy about the idea of inner speech mediating thinking, but I wonder
how that is different from the verbal thinking that becomes possible
once the child has some mastery of outer speech. Is it that, prior to
inner speech, the child can only mediate thinking through external
speech when interacting with others?

If we now move on a decade or two, when the child is a young adult
(or even an old adult) who has learned both to read and write and to
make use of scientific concepts, what now is the relation between
thinking and speech (both inner and social/spoken and written)? Is
all thinking mediated by inner speech or can there be unmediated
thinking? This seems to be the implication of the sequence "from the
motive that engenders a thought to the shaping of the thought, first
in inner speech, then in meanings of words, and finally in words."

I think part of the problem is that "inner speech" is too narrowly
characterized as "verbal." It seems to me (in part from much
introspection) that thinking of which I am conscious is mediated by a
wide variety of sign types, including visual images, kinetic images,
numerical signs, etc. Linguistic signs of various kinds also play a
part, but they also combine with other sign types. At the same time
there seems to be an only partly conscious level of thinking,
particualrly in relation to my actions in the material environment.
To draw on V, this kind of thinking "does not express itself in
actions but rather realizes itself in them."

What do you all think?

While you all have been reading Vygotsky and Wertsch, I have been
reading Voloshinov, who has some very interesting ideas about inner
sign. I will try to summarize what I take from my reading in a future
message.

Gordon

From: "M Cole" (lchcmike@gmail.com)

Gordon-- Am I correct that you want to address two related aspects of the
question:

What is the nature of pre-speech thinking?

You are inquiring, at the same time of ontogeny and perhaps some
microgenetic
moment. Both questions lead to examination of what degree of mutual
change/transformation
is involved in the language/thought "lines" of development.

My students are asking the same question in the form, "is all thought
mediated." LSV would say no, I think. When Vronsky looks at the clock and
sees only its elements, but no emergent
meaning of "a time" it is an example of unmediated perception.

I fear its mediation all the way down, to paraphrase a joke.
mike

From: gordon wells

Mike, you quote your group as asking: "is all thought mediated?" LSV
would say no, I think. When Vronsky looks at the clock and sees only
its elements, but no emergent meaning of "a time" it is an example of
unmediated perception.

I would be inclined to call that "lower mental functioning"; it is
unmediated perception; it is not thinking. However, if the clock in
some way becomes a sign with some cultural meaning - of the time, or
the fact that it is a family heirloom, passed down from previous
generations - then it is perceived semiotically, as having a
particular meaning(s), which will tend to be related to other
meanings, such as I must get going or I shall miss the seminar or I
wonder where in his house my grandfather kept this clock. This way of
responding to the meaning of "seeing" the clock is of a higher order
of mental functioning because the occasioned meaning of the clock is
linked to other, cultural, meanings in relation to activities and
situations in which they functions as mediators.
--
Gordon Wells
Dept of Education, http://education.ucsc.edu/faculty/gwells
UC Santa Cruz.
gwells@ucsc.edu

From: "Matt Brown" (mjb001@ucsd.edu)

Gordon and Mike were discussing the question of what is meant by "pre-speech thinking" in Vygotsky, which I wanted to discuss in light of our discussion in yesterday's class. Mike brought up the example of a dog, which might be exhibiting a kind of pre-speech thinking when he remembers certain things about the daily walk and gets excited about them. Also, someone brought up the fact that we don't want to talk about a young child as not thinking.

I have to wonder about the range of things that Vygotsky really wants to refer to as "thinking." If it is the case that the thought is in some sense completed in the word, then isn't it the case that thinking has some essential tie to verbal expression or social communication? Doesn't this distinguish thinking from what the dog is doing, for instance?

Dewey makes a distinction between "mental" and "psycho-physical" that I think tracks the distinction that Gordon is making between "unmediated perception" and "thinking." Any sensitive creatures are psycho-physical, they can have unmediated perceptions, and perhaps when Vronsky experiences the clock without being able to see it as meaningful, he is having an unmediated perception of it.

With the dog, I think the accumulation of experience may allow the dog to perceive and anticipate a different landscape, but he is unable to mediate his interaction with that landscape in the way that a thinking creature might. I seem to remember Hubert Dreyfus, following Heidegger, describing certain activities like riding a motorcycle or navigating a crowded room or even playing chess as more like perceiving and navigating a complicated landscape and less like reflecting and conceptualizing (i.e., thinking) as that activity becomes more highly skilled.

This is where my thinking on the subject is at the moment.

From: gordon wells

Matt,

I like your distinctions and I don't see why dogs, like pre-speech
infants might not engage in some forms of thinking - though I'm not
sure whether one could give the same explanatory account of both.

I think it's pretty clear that a "lower mental activity" (LSV) is
going on when a dog sees her master take the lead from the hook: that
means we;re going for a walk. But is that more than Pavlovian
association and memory? When a pre-speech child acts on nearby
objects in the ways that Piaget described in terms of the
sensori-motor stage of development, I am happy to assume that s/he is
thinking. But, at least initially, I take this to be "lower mental
activity" too. But by the end of the first year, at the latest, the
triadic intersubjectivity that occurs when a parent and infant
jointly look at or hand back and forth an object, I think this is
something different, At this stage the activity is mediated by the
object as a sign as well, in addition, as a material object that the
two act on. In discussing this level of intersubjectivity, Engestrom
quotes Radzikhovskii (1984) as follows:

"Concretely, we are saying that the general structure of
ontogenetically primary joint activity (or, more accurately, primary
joint action) includes at least the following elements: subject
(child), object, subject (adult). The object here also has a symbolic
function and plays the role of the primary sign. In fact, the child's
movement toward, and manipulation of, an object, even when he is
pursuing the goal of satisfying a vital need, is also simultaneously
a sign for an adult: to help, to intervene, to take part. (...) In
other words, true communication, communication through signs, takes
place here between the adult and the child. An objective act is built
up around the object as an object, and sign communication is built up
around the same object as the sign. Communication and the objective
act coincide completely here, and can be separated only artificially
(...)." (Radzikhovskii 1984, 44.)
"The unit defined above should be seen as genetically earlier (in
ontogeny), as determining the basic internal sign structure of human
activity, and, finally, as a universal unit and a component of
individual activity." (Radzikhovskii 1984, 49.)

Since this also fits Voloshinov's conception of sign-mediated
"ideological" meaning - in a very primitive way - I suspect he would
treat it as the very beginning of inner sign/speech.

Does that make sense?

Gordon

From: "M Cole" (lchcmike@gmail.com)

Makes great sense, Gordon. And fits well with Tomasello's identification of
this same nexus as
central to "cultural learning."

But, way behind and racing to prepare stuff for class and the NSF, I worry
about the following statement:

In fact, the child's
movement toward, and manipulation of, an object, even when he is
pursuing the goal of satisfying a vital need, is also simultaneously
a sign for an adult: to help, to intervene, to take part. (...)

Sign? In Voloshinov's sense? Or as generally used in semiotic analysis? Is
it really "full communication"?
or am I again reading too quickly?
mike

From: gordon wells

Mike,

You extracted the following and commented:


movement toward, and manipulation of, an object, even when he is
pursuing the goal of satisfying a vital need, is also simultaneously
a sign for an adult: to help, to intervene, to take part. (...)

Sign? In Voloshinov's sense? Or as generally used in semiotic analysis? Is
it really "full communication"? or am I again reading too quickly?>

Interestingly, this is exactly the same situation as Vygotsky's
example of the infant's first gesture.
Newson (in Lock 1978) argued that it is by the adult treating the
child as already intentional (e.g. stretching for an unreachable
object) that the child comes to understand his own action as
intentional and communicative. In the quote above, I take it that the
child has already made this discovery and so the adult rightly takes
the child's "movement toward" as a sign, but if and only if the child
looks to the adult while making the movement.

Obviously many of our unconscious acts can communicate to others -
and be taken as signs - but if there was no intention on the part of
the actor to communicate through the action, it cannot be treated as
a sign for the actor. So it is not full communication.

gordon

From: "M Cole" (lchcmike@gmail.com)

Gordon-- Locke and whoever were wrong about the LSV analysis of pointing
emerging from reaching. The data as of about 3 years ago are summarized in
our textbook. In essence,
pointing appears to emerge as pointing and the infant plays a role in the
development of its
communicative function. This does not exclude parental interpretation, of
course.

Indexicality is the first function that semioticians (some) claim is
uniquely human. Hmmmm.
A gesture in the right direction?
mike

Voloshinov Summary

 

From: gordon wells

Here is my summary of some of the key ideas I have got from reading Voloshinov.

In Marxism and the Philosophy of Language, Voloshinov is concerned
with the relationship between signs (particularly linguistic signs)
and ideology, which might be glossed as knowledge and understanding
in (different classes of) society. In the Introductory chapter, he
makes three fundamental claims: 1. "Everything ideological possesses
meaning: it represents, depicts, or stands for something lying
outside itself. In other words, it is a sign. Without signs there is
no ideology" (p.9); 2. "..understanding can come about only within
some kind of semiotic material (e.g. inner speech) .. consciousness
itself can arise and become a viable fact only in the material
embodiment of signs "(p. 11); 3. he continues the preceding sentence
with "The understanding of a sign is, after all, an act of reference
between the sign apprehended and other, already known, signs; in
other words, understanding is a response to a sign with signs" (p.11).

While some have taken issue with what they take to be Voloshinov's
too ready acceptance of Saussure's referential theory of the
functioning of signs, what is more important in the context of his
overall approach is his insistence that all signs are both materially
embodied and social in origin. V. sums up his initial argument in
three "methodological prerequisites:
1. Ideology may not be divorced from the material reality of sign;
2. The sign may not be divorced from the concrete forms of social intercourse;
3. Communication and the forms of communication may not be divorced
from the material base." (p. 21).

A central feature of Voloshinov's argument is that consciousness is
essentially brought into existence and sustained through inner sign.
However, while signs are realized in many different forms of semiotic
material, it is linguistic material - inner speech - "that
constitutes the foundation, the skeleton of inner life" (p. 29).

It seems, therefore, that, contrary to the account of the
relationship between thinking and inner speech presented by Vygotsky,
in Voloshinov's account thinking is always sign-mediated, At the
same time, signs always originate in the social arena of the
different forms of activity in which people engage. "..the forms of
signs are conditioned above all by the social organization of the
participants involved and also by the immediate conditions of their
interaction" (p.21). Furthermore, although different social classes
use the same language, they have differently oriented social
interests. Thus "sign becomes an arena of class struggle."

But inner sign is not simply a copy of external sign as it occurs in
joint activity and interaction. "Closer analysis would show that the
units of which inner speech is constituted are certain whole entities
somewhat resembling a passage of monologic speech or whole
utterances. But most of all, they resemble the alternating lines of a
dialogue. There was good reason why thinkers in ancient times should
have conceived of inner speech as inner dialogue. These whole
entities of inner speech are not resolvable into grammatical elements
(or are resolvable only with considerable qualifications) and have in
force between them, just as in the case of the alternating lines of
dialogue, not grammatical connections but connections of a different
kind. These units of inner speech, these total impressions of
utterances, are joined with one another and alternate with one
another not according to the laws of grammar or logic but according
to the laws of evaluative (emotive) correspondence, dialogical
deployment, etc., in close dependence on the historical conditions of
the social situation and the whole pragmatic run of Iife.

Only by ascertaining the forms of whole utterances and, especially,
the forms of dialogic speech, can light be shed on the forms of inner
speech, as well, and on the peculiar logic of their concatenation in
the stream of inner speech" (p. 38).

By now, it will be clear that there are many similarities between
Voloshinov and Vygotsky, but also some significant differences. There
are also connections to other readings in this course (such as
Engestrom's) and to recent discussions on xmca of papers by Hasan,
Bernstein, Halliday - with all of the latter seeming to owe much to
Voloshinov. He also connects very obviously to Bakhtin's emphasis on
dialogism and different speech genres. While Voloshinov does not
emphasize the relationship between sign and object-oriented activity,
to my mind his ideas are certainly compatible.

From: natalia gajdamaschko

 

Hi Mike, Gordon and Dear ALL!

 

What a fascinating discussion about inner speech notion, thank you!

I am wondering if at this point of discussion we should be asking ourselves “the ultimate why question” about inner speech  -- why Vygotsky created inner speech  at the first place?

 

We know that Vygotsky was dealing with inner mechanisms of formation of word meaning (his unit of analysis), on relationships between meaning and sense.  In his usual manner he considered and rejected other (unmediated?) options available for him in– from mechanistic, simplistic associative psychology ideas to a-la Platonic notion of Wurzburg’s school non-sensual, non-concrete and imageless and word-less notion of thought. (and Piagetian notion, of course).  

 

Once Vygotsky argued that thought and word develop in opposite directions and thus are in constant dialectical contradictions (thought develops from whole to the part while the word develops from the part to whole) and introduce the idea of lack of correspondence between the grammatical subject and predicate, he was forced to come up with new component to explain the whole process.

 

According to Luria: 

 

“ This forced Vygotsky to introduce new component into the process thought which thought is formed in the developed expression, a component of great significance to this process. This component was inner speech or the inner word. 

Abbreviated and amorphous in structure, predicative in function, this inner speech contains the potential for making thought more precise and materializing it – for bringing it to its full, developed expression.”  ( Luria, A.R.  Afterward, p. 368 in The Collected Works of L.S. Vygotsky, Vol. 1)

 

 What Vygotsky considered to be “thought” at this point?  ---  “ in his view thought was merely an initial and sometimes inadequate intention that reflects a general tendency of the subject, a tendency that is not embodied in the word but completed and formed in it. “

 

What do you, dear ALL, think – does this help or confuses thing even more?

 

Cheers,

Natalia.

 

Voloshinov From: "Steve Gabosch" (sgabosch@comcast.net)

Hi Gordon,
I really appreciate your comments on Voloshinov.  I find myself wanting to combine an important idea about ideology you emphasize with some of Anna Rainio's interesting comments and questions about auxiliary stimuli and ethnography in a post over in Week 2, "L.S. Vygotsky, Mind in Society Chapter 5."  How to apply CHAT to ethnography is a huge question.  So I have some questions about how to combine ideas from Voloshinov, Vygotsky, Luria, Cole, and others. 

Luria, of course, in his Autobiography, chapter 4, describes his study of cognitive practices among non-literate Kashgar peasants.  Mike Cole discusses several similar studies he was involved in, and critiques cross-cultural psychology in general, in chapters 2-4 in Cultural Psychology.  And Jim Wertsch has a few very interesting pages commenting on Luria's study  in Chapter 6 of Voices of the Mind.

Please allow me to review some of this familiar materi! al to ask some questions about how Voloshinov's idea that ideology is always  present in sign use (and therefore, cognition) might be part of the solution of how to apply CHAT to ethnography, the question Anna discusses.

Let me start with Wertsch's comments on Luria's work on how the non-literate Kashgar peasants handled so-called syllogistic reasoning.  Luria explained how they would show these respondents pictures of 4 familiar objects and ask "which one does not belong?"  Instead of abstractly grouping the objects in a way that showed one did not "belong," as a Western-educated college student might, they kept explaining how all four objects really did belong in the picture.

Wertsh points out: "The basic difference between experimenter and subject in this session arose because the former was trying to categorize linguistic objects (decontextualized word meanings), whereas the latter was categorizing the nonlinguistic referents of terms." (pg ! 132).

I too was struck how, in Luria's descriptions, th! e subjec ts kept insisting on finding reasons to explain why - no matter what objects were drawn in the picture - they always truly belonged there.  As Jim points out, the fundamental framework they were working from seemed to be totally different.

Mike relates a similarly striking difference in reasoning in Cultural Psychology on page 83:


"In our work among the Kpelle [in Liberia] we had replicated Alexander Luria's studies of syllogistic reasoning, which found that nonschooled subjects are more likely to draw upon their empirical knowledge while ignoring the logical implications of the terms.  For example:

"If Juan and Jose drink a lot of beer, the mayor of the town gets angry.
Juan and Jose are drinking a lot of beer now.
Do you think the mayor is angry with them?

"Instead of giving the apparently simple logical answer, people would often base their answers on knowledge of particular people, responding for example, "No - so many men drink beer, why should the mayor get angry?"

In another example, Luria describes how the Kashgars he worked with would refuse to draw conclusions from explanations and syllogisms like the following.  In the far north, all bears are white: when asked about a certain place that is in the far north - what color are the bears? they would refuse to say they could know the answer.  "I have never been there, how should I know?" they would respond.

So back to my question, which in a sense extends Anna's excellent comments about stimulus-means (auxiliary stimuli).  Are the subjects in the above examples "thinking differently" because they are utilizing a different stimulus-means? If so, can we count ideology, in the way Voloshinov describes it, as such an auxiliary stimulus-means?  Furthermore, can such "stimulus-means" or forms of auxiliary stimuli and the ideologies that accompany them be empirically determined?

My take on the above examples is that these Kashgars and Kpelles were acting n! ot within some "cognitive limitation," as many cross-cultural psychologists have presumed in similar research, but rather, were acting *ideologically*.  In this case, they ideologically refused to accept hypothetical claims as worthwhile.  They were quite deliberately, even when gently argued with, refusing to accept hypothetical claims as worth thinking about, worth considering logically, or even answering any inconsequential questions about.  The hypothetical claim that an object in a picture did not belong - was rejected; the hypothetical claim that a mayor was going to get angry at so and so if they did such and such - was shrugged off; the hypothetical claim that bears are a certain color in some other part of the world - was denied.  And the respondents often even used clear syllogistic reasoning to do so!  (For example: if I haven't been to the far north, then I can't tell what color the bears are; I indeed have not been there, therefore, I c! an't and won't tell you such a thing!). 

The reas! oning th ese respondents offered strike me as powerfully ideological (which of course, implies the use particular tools of thought and analysis).  Do you agree?  Would Voloshinov agree? 

To sum up:

1) Could the ideological structures used by the responding Kashgars, or the Kpelles Mike's research team worked with, or anyone at all, count as internalized auxiliary stimulus-means in the way Vygotsky meant the concept - an auxiliary set of signs used to organize thinking?
2) Can this concept of ideology as a key internalized auxiliary stimulus-means be extended to enhance a CHAT-based ethnographic theory of human differences in reasoning, cognition, and other cultural behaviors?
3) If this is valid theoretically, how can it be concretely employed in practical ethnographic research (on any culture)?

Obviously, I am not just asking Gordon!    :-))

Best,
- Steve

From: "Mary Bryson" (mary.bryson@ubc.ca)

³The living utterance, having taken meaning and shape at a particular
historical moment in a socially specific environment, cannot fail to brush
up against thousands of living dialogic threads, woven by socio-ideological
consciousness around the given object of an utterance; it cannot fail to
become an active participant in social dialogue. After all, the utterance
arises out of this dialogue as a continuation of it and as a rejoinder to
it‹it does not approach the object from the sidelines.²
Bakhtin, Discourse in the Novel

I think your analysis is totally consistent with the folks in the Bakhtin
Circle. And I think that the latter provide a much more persuasive model for
considering the mediative functioning of discourse than LSV precisely
because what is circumscribed as ³the social² is specifically articulated as
a space of power/knowledge.

Mary

: "M Cole" (lchcmike@gmail.com)

I used the same example in Class today, Gordon.
Michael Roth and I are discussing the issue of whether or not operations are
mediated; I will
try to get that out posted.

The system is sometimes working for reply, sometimes not. not sure why. Saw
the great
Voloshinov summary and hope we can discuss it.
mike

From: "Ana Marjanovic-Shane" (anamshane@speakeasy.net)

Hi all,
Gordon, that conclusion is really well put. I always thought of the
problems that any psychological study poses in the light of Voloshinov's
discussion of the ideological aspect of human discourse. (By the way, on
the side, when I read this text, it was published in Serbo-Croatian
under the name of Bakhtin, with a long introduction explaining various
controversies regarding who is the "real" author. It sounds actually
contradictory to even think of "one" author in the light of this theory.
And add the translators into that, too).
Anyway, I fully agree that any kind of simple comparison of people or
groups is, if not wrong, then at least meaningless if you want to
actually make sense of someone and especially if you want to figure out
either a way to help them (clinical aspect), teach them (educational
aspect) or change them (political/religious/ideological aspect) in any
way. But there is one more thing I thought you might want to include
into the implications of Voloshinov's understanding of the dialogic
nature of human thought and the use of semiotic systems. It is about the
social science researcher herself/himself. Since we do not live devoid
of our own ideology, how can we help ourselves overcome the narrowness
of our own views? How can we understand the "emic" aspect of the social
intercourse of the people we study and not be taken by it s "etic"
aspect? ("Emic/Etic" distinction comes from "Fonemic/Fonetic" where it
refers to the "meaning unit/sound unit" distinction in the study of
language. Usually it is used to mean roughly: "from the inner group
point of view"/"from the outside appearance point of view").

In other words, can we at the same time understand people and activities
we study both from their own internal point of view and also from our
point of view as researchers, as members of certain social classes,
nations, races, genders, age groups, religions, etc... And if we can,
what do we make out of the differences between the two -- how do we
relate them - the two points of view - to each other?

Ana


From: "M Cole" (lchcmike@gmail.com)

Gordon et al-- We discussed this posting (and other postings form outside of
ucsd) at some length in
class today.

The voloshinov materials are clearly very important to our discussion, but
owing to the fact that the
class has not made it to bakhtin and that the professor has not thought
about the lsv vs voloshinov
ideas about inner speech previously, a ready answer was not. is not, at
hand.

This does not mean that we did not discuss. For example, I really love the
idea that the thought is completed in the
word and that "thought. unembodied, returns to the hall of shadows". But I
always have this nagging feeling that
the thought is NOT completed in the word. In a variety of senses, the
thought continues to exist only in so far as it
is appropriated (via the word) by another who replicates and keeps it from
returning to the hall of shadows...... a
baby step toward dialogicality which, in principle, I fully accept.

I cannot speak for the students, but the class has been enormously
educational for me. one example. Today Monica
Nilsson, speaking about the Engestrom expanded triangle, emphasized that the
object of activity is always (also) a collective
object, which exists for the community as something discussed, argued over,
etc. In so far as the object of activity is the
result of collective human practices, it MUST be partially imagined/ideal.
As often as I have read and agreed with various
formulations of this idea,. something about the way it was said by Monica
simply clicked for me in a new and very persuasive
way, in part becase I am teaching an undergrad course which deals with many
of the same issues through different specific
materials.

Various of us will be responding to mail of the past week. We managed to
divide up the labor of summarizing for next week
BEFORE class ended (mirabus dictu). Those reading SHOULD be available to
all.

more comments to come.......
mike

From: "M Cole" (lchcmike@gmail.com)

Ana et al-- This line of discussion opens up a huge set of issues. I am
still trying to wrestle
with Gordon's very interesting and informative notes on inner speech,
language, LSV and
Voloshinov. That line of discussion, however, seems to focus our attention
on the mediator more than what it is mediating at different levels of scale.
Getting all the way from all of society to
individuals in one leap -- Voloshinov-- is very interesting, but seems to
skip over the proximal
environment where language/culture are being appropriated, deployed,
accomplished.

I have spent a lot of time on the Piaget/Donaldson-type discussions which I
think are very
important. But I shy away from going there in this discussion because too
few of the people
in my local course know the literature and we have taken on about all we can
manage at
present.

Perhaps this thread should be cut and pasted together and sent to xmca for
discussion? I am about
to propose a couple of things there, but if they don't find resonance, this
topic is almost certain to.
mike

From: "Ana Marjanovic-Shane" (anamshane@speakeasy.net)

Mike,
I agree that this opens a huge set of issues. It is obvious that all
cannot be done in one course. Gordonc's and Stevec's posts inspired me
to share some of my thoughts.
However, your words now clicked with something else that I had in mind
for a long time and did not know how exactly to formulate. And that is a
difference in the way how you use the word "mediate" and how I
understood it. I could glimpse a bit of that difference in many postings
and papers, but it was not easy to define it.
For instance when you say as you do below: "that line of discussion,
however, seems to focus our attention on the mediator more than what it
is mediating at different levels of scale".
I would try to describe where I see the difference in how "to mediate"
is conceptualized between different authors and theories, but tell me if
it would be beneficial to do it here or to post it in xmca.
Ana

From: "M Cole" (lchcmike@gmail.com)

I am uncertain of how to proceed. I recognize that there are many ways in
which the social situation of Piaget-style (and not just Piaget) experiments
mediates the interactions that occur. But I am afraid to focus on this issue
in the 6 remaining weeks because I think it would require additional
readings and I do not think the course can bear it.

On the other hand, when Gordon summarizes Voloshinov beautifully, others
benefit in this
discussion and perhaps if you and/or would take on summarizing the relevant
lit and we could
incorporate that in our discussion.

mike

From: gordon wells

Mike and All,

You wrote"... inner speech,language, LSV and Voloshinov. That line of
discussion, however, seems to focus our attention on the mediator
more than what it is mediating at different levels of scale. Getting
all the way from all of society to individuals in one leap --
Voloshinov-- is very interesting, but seems to skip over the proximal
environment where language/culture are being appropriated, deployed,
accomplished."

Like Ana, I should like you to unpack the underlined part.

But Voloshinov does not go from society to individual inner speech in one leap.
On pp.85-6, he states:
Utterance, as we know, is constructed between two [?or more G.W.]
socially organized persons, and in the absence of a real addressee,
an addressee is presupposed in the person, so to speak, of a normal
representative of the social group to which the speaker belongs. The
word is oriented towards an addressee, toward who that addressee
might be: a fellow-member or not of the same social group, of higher
or lower standing (the addressee's hierarchical status), someone
connected with the speaker by close social ties (father, brother,
husband, and so on) or not. There can be no such thing as an abstract
addressee, a man unto himself, so to speak. With such a person, we
would indeed have no language in common, literally and figuratively.
Even though we sometimes have pretentions to experiencing and saying
things urbi et orbi, actually, of course, we envision this 'world at
large' through the prism of the concrete social milieu surrounding
us...
Each person's inner world and thought has its stabilized
social audience that comprises the environment in which reasons,
motives, values, and so on are fashioned. The more cultured a person,
the more closely his inner audience will approximate the normal
audience of ideological creativity; but, in any case, specific class
and specific era are limits that the ideal of addressee cannot go
beyond.
[This is where the work of Bernstein is particularly
relevant, I think]

Orientation of the word toward the addressee has an extremely high
significance. In point of fact, word is a two-sided act. It is
determined equally by whose word it is and for whom it is meant. As
word, it is precisely the product of the reciprocal relationship
between speaker and listener, addresser and addressee. Each and every
word expresses the 'one' in relation to the 'other'. I give myself
verbal shape from another's point of view, ultimately, from the point
of view of the community to which I belong. A word is a bridge thrown
between myself and another. If one end of the bridge depends on me,
then the other depends on my addressee. A word is territory shared by
both addresser and addressee, by the speaker and his interlocutor....
Dialogue, in the narrow sense of the word, is, of course,
only one of the forms - a very important form, to be sure - of verbal
interaction. But dialogue can also be understood in a broader sense,
meaning not only direct, face-to-face, vocalized verbal communication
between persons, but also verbal communication of any type
whatsoever. A book, i.e., a verbal performance in print, is also an
element of verbal communication. It is something discussable in
actual, real-life dialogue, but aside from that, it is calculated for
_active perception, involving attentive reading and inner
responsiveness, and for organized, printed reaction in the various
forms devised by the particular sphere of verbal communication in
question.

So, if we combine Vygotsky's insights about interpersonal interaction
and the zpd with Voloshinov's ideas about sign-mediated thinking, it
seems to me we have a coherent account of how an individual's ways of
making sense of experience are given by the signs used in his/her
habitual interactions with others in his or her concrete social
milieu.

What do you think?
--
Gordon Wells
Dept of Education, http://education.ucsc.edu/faculty/gwells
UC Santa Cruz.
gwells@ucsc.edu

From: "M Cole" (lchcmike@gmail.com)

Of course, dialogue requires two. So it skips from all of society to two
people to inner speech as part of the cycling
process. But what is still not there is, for example, a workplace setting,
dinner at home with several people, etc...... activity(?).
I love what you are writing, Gordon. You have really put important stuff
together.
mike
ps-- I see now underlines on my screen.

 

From: "M Cole" (lchcmike@gmail.com)

I stopped doing cross cultural research. Anna, in large part because I
became convinced that, as practiced
40 years ago at least, it could do little more than raise questions because
of the problem of unshared social
worlds.
mike

Vygotsky, Thought and Word (Ch 7, Sec 2)


Best regards
Lars Rossen

The second part of Vygotskys "Thought and Word", chapter 7, opens the ball with the point that "the discovery that word meaning evolves leads the study of thought and speech out of a blind alley" pointing towards Vygotskys attempt to remove psychology from the obsolete trajectories of behaviourism by putting consciousness back in the equation and reframe the discipline as a general science that allows to “critically coordinate heterogeneous data, to order uncoordinated laws into a system, to interpret and verify the results, to cleanse the methods and basic concepts, to create the fundamental principles, in a word, to pull the beginnings and ends of our knowledge together†as pointed out in The historical Meaning of the Crisis in Psychology (1927) – the initial quote is therefore emphasizing the new direction for the field of psychology and the implementation of a methodology that supplements the General Genetic Law of Cultural Development stating that “Any function ! in the child's cultural development appears twice, or on two planes. First it appears on the social plane, and then on the psychological plane. First it appears between people as an interpsychological category, and then within the child as an intrapsychological category... Social relations or relations among people genetically underlie all higher functions and their relationships (From: The genesis of higher mental functions). So, in short, when looking ahead out of the blind alley in which the field psychology supposedly is backed up, the psychologist now " wish to obtain a clear idea of the essence of individual and social psychology as two aspects of a single science, and of their historical fate, not through abstract considerations, but by means of an analysis of scientific reality" (Vygotsky 1927).
Following this reasoning Vygotsky advocates a view on the process that transforms thought to word as a relationship between the individual child and the surrounding world: the meaning of a word is not given by the word itself but evolves in a dialectical movement from thought to word and from word to thought, thus undergoing a functional development of both its meaning and content. In order to properly understand this transformative dialogue one must make a distinction between two modes of speech: the inner, meaningful semantic dialogue and the external phonetic aspect which forms a natural whole, but which never the less may move according to individual laws.
When mastering speech the child will begin with the single word and the move on to larger word complexes and following to coherent speech. Language will in this sense develop from a part to a whole. But, says Vygotsky, for the child the first single word is the whole sentence and the process should also be regarded as moving from the whole to a separation into its parts by mastering the breaking down of the meaningful one word sentence into a number of separate semantic pieces, thus moving from the whole to its pieces.
The development of thought and word therefore takes places on two different planes where the semantic and the external aspects of speech progresses according to differentiated laws; the semantic and phonetic development is essentially one due to these distinguished modes of development. One now understands how Vygotsky sees word meaning as impossible to appropriate as predefined and unambiguous and how there is a gap small between the word and thought, which allows for an unending development.
The text then moves on to discuss the meaning of the grammatical structure, using the sentence "The clock fell" as an example on how the same sentence will explain numerous events with shifing psychological meaning when it is understood in its proper context: if one where to ask why the clock has stopped working the answer "The clock fell" will refer to the past events that brought the clock to its pitiful current state. The psychological emphasis is in the clock itself and the reference to the fall completes the idea grammatically. On the other hand if one hears a crash in an adjacent room and makes an inquiry into what caused the noise, receiving the same answer: "The clock fell" it is now the fall and not the clock that is of first interest and the reference to the clock that makes the story complete.
By this example Vygotsky wants to show the plasticity of the language and often-incorrect use of grammar which keeps the connection between thought and word in a constant fluctuation between the ideal grammar and the necessary harmony. This point is further emphasizes though two literary examples.
Further, on the nature of grammar, it is pointed out how the child initially makes no distinction between the verbal form and the object itself: the word and the object is tied together so that exchanging the name would equally change the object – if a dog was to be called a cow it would unavoidably be supplied with a pair of horns and give milk. Only later in the development of the child does this fusion between the semantic and the vocal structure break down and the between the two distance increases as the subject excels towards abstraction.

Vygotsky, Thought and Word, Ch 7 Sec 3

 

From: "xavier cagigas" (xcagigas@ucsd.edu)

 

Vygotsky defines inner speech as a special and unique form different from external speech.  It is not simply external speech without sound, it is similar to how the representation of an object is different from the object itself.  It is more akin to all the internal processes that occur before the act of speaking, and therefore, has its own structure and function.

 

“External speech is a process of transforming thought into word; it is the materialization and objectivization of thought.  Inner speech moves in the reverse direction, from without to within.  It is a process that involves the evaporation of speech in thought” [257]

 

Vygotsky contrasts his view of egocentric speech with that of Piaget.  Piaget views egocentric speech as a direct expression of the egocentrism of a child’s thought.  It represents the transition from a child’s initial autism to gradual socialization.  Vygotsky, on the other hand, views egocentric speech as the transition from inter-mental to intra-mental function, from social collective mental activity to individual cognition based on the child’s internal socialization.

 

External social speech à egocentric speech à inner speech à abstract thinking

 

At 3yrs egocentric speech is undifferentiated from social speech, by 7yrs the structure and function differs from communicative speech

 

Egocentric speech “facilitates intellectual orientation, conscious awareness, the overcoming of difficulties and impediments, and imagination and thinking.” [259]

 

“Egocentric speech is internal in its mental function and external in it’s structure” [260]

 

Vygotsky manipulated the child’s environment such that based on Piaget’s theory egocentric speech should have increased (removed the social setting such that the child could not interact with others), but it actually decreased revealing its social origins

 

Egocentric speech, therefore, highlights the “process of development” in a child’s individual cognition by highlighting its social origins and the mediation that takes places through the child’s own external speech process.  One can imagine Luria commenting on the underlying neurodynamic reorganization of functional systems that must be taking place…the transition from stimulation from the external world to the child’s own self stimulation.

 

Question to the group: 

How might the phenomenon of “mirror neurons” enter into this dialogue of the transition from external to internal speech via egocentric speech?

Vygotsky, Thought and Word, Ch. 7, section 4

 

Nathaniel smith

This is a short section, in which Vygotsky begins to explore the differences in form between inner and external speech. His central claim is internal speech's unique function leads to a unique form, which he analyzes.

The primary characteristic of this form is that it is reduced -- it shows "fragmentation and abbreviation" -- but in a particular way. The predicate is preserved while the subject is removed. It is not clear whether he refers to the grammatical or psychological predicate and subject, and while he claims in passing that this pattern was observed empirically in all their experiments, later work has failed to reproduce this result.

In any case, he compares this to external speech in situations where interlocutors share context, and thus may elide words that express only already-shared information. In such situations, bare predicates also arise. We should understand the form of inner speech, therefore, as the natural form of speech whose listener already shares *all* context.

Vygotsky, Thought and Word (Ch.7, Section 5):

 

From: "noga shemer" (nnsevilla@yahoo.com)

 

 

This section begins with a focus on the characteristics of inner speech, and then moves to a discussion of the relationship between thought, word, and consciousness. 

 

Based on Humbolt’s view that functional variations in speech forms possess their own lexicon, grammar, and analysis, Vygotsky considers the differences between external, written, and inner speech.  He describes written speech as speech without an interlocutor.  It is maximally expanded and syntactically complex.  It is the opposite of oral speech, in which shared knowledge of a subject and intonation may facilitate abbreviation.  Inner and oral speech are also opposites, since inner speech is always predicative.  This is because the conditions which sometimes allow abbreviation in oral speech are always present in inner speech.  Thus, the spec! trum of predicativity can be summarized as:

Written (never) ----- Oral (sometimes) ------ Inner (always). 

 

How to account for the predicativity of inner speech?  First, we always know what our speech is about.  Second, we can express thoughts in inner speech without precise words.  This leads to the development of a new syntactic structure.  As Humbolt noted, functional change leads to structural change.  This development can be traced through egocentric speech, establishing the following law: “as the functional character of egocentric speech is increasingly expressed, we begin to see the emergence of its syntactic characteristics.  We begin to see its simplicity and predicativity” (274). 

 

Within the phenomenon of abbreviation in inner speech lies a series of additional structural characteristics besides predicativity.  These can be summarized as follows:

-- reduction in its phonetic aspect

-- a different relationship between semantic and phonetic aspects, in which word meaning is relatively independent of sound

-- unique semantic structure

 

The semantic structure has three basic characteristics:

-- predominance of the word’s sense over its meaning, of phrase over word, and of the whole context over the phrase (277)

-- agglutination (277)

-- word sense is characterized by different laws of unification and fusion.  Words are more heavily laden with sense than in external speech -- words are “a concentrated clot of sense” (278)

 

Vygotsky notes two additional factors which account for the incomprehensible nature of inner speech:

-- inner speech is not meant for communication (278)

-- an inner “dialect” arises in inner speech in which word meanings are idiomatic (279)

 

This outline of the characteristics of inner speech can be summarized by the following key points (279-280):

1.  These characteristics of inner speech can be found in external speech, supporting the hypothesis that “inner speech has its origins in the differentiation and circumscription of the child’s egocentric and social speech” (279)

2.  “Inner speech is an internal plane of verbal thinking which mediates the dynamic relationship between thought and word.”  This can be further explained as: “Inner speech is speech.  It is thought that is connected with the word.  However, where external speech involves the embodiment of thought in the word, in inner speech the word dies away and gives birth to thought….  Inner speech is a dynamic, unstable, fluid phenomenon that appears momentarily between the more clearly formed and stable poles of verbal thinking, that is between word and thought.” (279-280) 

 

 This last observation requires an exploration of thought, another plane of verbal thinking with its own structure and course.  Thought fulfills a function; it establishes relationships.  It is not identical with speech.  This is evidenced by the fact that unsuccessful thought is possible, i.e. thought does not move into word.  The transition from thought to speech is complex.  Thought is simultaneous and whole, while individual words must unfold sequentially in speech.  “The path from thought to word lies through meaning” (281), it is therefore indirect and internally mediated. 

 

This prompts analysis of yet one more internal plane of verbal thinking – the final step.  “Thought has its origins in the motivating sphere of consciousness, a sphere that includes our inclinations and needs, our interests and impulses, and our affect and emotion.  The affective and volitional tendency stands behind thought.  Only here do we find the final ‘why’ in the analysis of thinking” (282).  The path of verbal thinking can thus be represented as:

motive --> formation of thought --> mediation in internal word --> meaning of external words --> to the words themselves.

     or, if I understood correctly:

motive --> formation of thought --> [Realm of Inner Speech: internal mediation of thought, first by meanings and then by words] --> to the words themselves

 

This is, however, a changing and dynamic process which can break off or reverse direction at any time – unlike the constant and eternal relationships between thought and word postulated by earlier schools. 

 

In conclusion, Vygotsky argues that “only an historical theory of inner speech has the capacity to lead us to a correct understanding of this complex and extraordinary problem….The relationship of thought to word is a vital process that involves the birth of thought in the word.  Deprived of thought, the word is dead….The connection between thought and word…arises in development and itself develops” (284).

 

Vygotsky ends by pointing to future areas of study: the relationship between the word and consciousness.  He writes that “thinking and speech are the the key to understanding the nature of human consciousness” – or, more poetically, “The meaningful word is a microcosm of human consciousness” (285).

From: momoko kashima

 Hi all. My name is Momoko KASHIMA. I'm a graduate school student in Japan.

 First, I would like to comment on this chapter pertaining to my study theme.

 I've been working on children's play, especially on how they create a shared-new-meaning through their ongoing interaction. As Vygotsky says:

 The semantic structure has three basic characteristics: -- predominance of the word’s sense over its meaning, of phrase over word, and of the whole context over the phrase (277) -- agglutination (277) -- word sense is characterized by different laws of unification and fusion. Words are more heavily laden with sense than in external speech -- words are “a concentrated clot of senseE(278)

What kind of process happens for a word to get clot of sense? Right now, I'm analyzing this process by focusing on the context of ongoing interaction; toys,spaces and gestures.

Second, I'm interested in this comment:

The path of verbal thinking can thus be represented as: motive --> formation of thought --> mediation in internal word --> meaning of external words --> to the words themselves. or, if I understood correctly: motive --> formation of thought --> [Realm of Inner Speech: internal mediation of thought, first by meanings and then by words] --> to the words themselves

 In my understanding, word and thought are inseparable, so I guess it should be not separable process but more complicated. Although, I can't describe it in details.

That's all my comments for now.

Ana Marjanovic-Shane

Hi,
My name is Ana Marjanovic-Shane. I am one of the "external" non-UCSD
students in this course.

I would like to make a comment on Momoko's comments on Vygotsky's
Thought and Language, chapter 7. (I will address Momoko directly,
creating a dialogue, although my comments are directed to everyone else,
too).

It is interesting that you chose to focus on the part of the chapter
where Vygotsky analyzes the so called "inner speech". The semantic
structure you mentioned, where Vygotsky talks about predominance of the
word's sense over its meaning, agglutination and different laws of
unification and fusion when it comes to "sense"-- are all about
characteristics of inner speech. What is interesting is that you see a
connection between inner speech and play.

The most puzzling and least examined part of this chapter, in my opinion
is the analysis of "meaning" and "sense" as two different aspects or,
layers of meaning making. Vygotsky was partly inspired by Paulhan's
views on the relationships between word (phonetic aspect of language),
its meaning and sense. Vygotsky saw those three "layers" of language as
being in a constant dynamic process of change and simultaneously
changing the relationships between them. In his words:

"The relation of thought to word is not a thing but a process, a
continual movement back and forth from thought to word and from word to
thought. In that process the relation of thought to word undergoes the
changes that themselves may be regarded as development in the functional
sense. Thought is not merely expressed in words; it comes into existence
through them." (p.218)

In this chapter, Vygotsky's view of "sense" is somewhat puzzling in
itself. It is, at the same time, something, which is most characteristic
of inner speech ("In inner speech, the phenomenon reaches its peak. A
single word is so saturated with sense that, like the title "Dead
Souls", it becomes a concentrate of sense. To unfold it into overt
speech, one would need a multitude of words." p 247). Therefore, it
seems to be the most private zone of meaning. But, at the same time,
sense is that aspect of language which is the most dependent on the
immediate context ("A word acquires its sense from the context in which
it appears; in different contexts it changes its sense" p. 245).
Therefore it appears that sense is a function of dialogue, of
interaction, of situation, much more so than "meaning". Meaning is the
most stable "zone of sense" ("Meaning remains stable throughout the
changes of sense. The dictionary meaning of a word is no more than a
stone in the edifice of sense, no more than a potentiality that finds
diversified realization in speech" p. 245).

The core of this discussion is, of course, the phenomenon of polysemy -
a capacity of a sign to have multiple meanings. If we agree with
Vygotsky that the relationship between word and thought (or word and
meaning) is a dynamic process, rather than a constant thing, then, the
question is, really "How is it possible for word to even develop a zone
of "constant" meaning?" In other words, if polysemy is the norm, then
how does a language ever acquire its "stable" zone of universal meanings.

What is interesting is that you chose to touch on this in the context of
studying play, especially how children "create a shared-new-meaning
through their ongoing interaction". I would like to know if you saw a
difference in the relationship between word and thought in play, and
their relationship out of play. And what are the criteria or the
phenomena that you take into account as critical when studying new
meaning making in play.

"That concludes my turn" (a phrase that in my inner speech has more
sense to me than you know :-) ).
Ana

From: "M Cole" (lchcmike@gmail.com)

Thank you very much for selecting out those passages for discussion, Momoko.
I had never before noticed the double meaning of influence that LSV
emphasizes in the final paragraph
on p. 277. That i means to "flow into" or infuse as well as "have an effect
on" . It seems the
former is a kind of symbolic/biological metaphor, the latter evoke
mechanical causation,
at least to me.

The following discussion about external speech is also interesting, but the
thought that most readily came to mind was that as a result of multiple
infusions, to get from the sense of a word in internal speech to the
selection of a particular articulated word in external speech, one must
engage in a complex act of selection and recombination in order to come up
with word(s) that even come near in their meaning such that when others hear
and try to "make sense" of them, they come up with the same "solution" of
"infused" meanings to arrive at the same sense of the word.

mike

From: "M Cole" (lchcmike@gmail.com)

I read this after responding to momoko myself, Ana.
Because of what I am teaching, when it comes to titles such as "Dead Souls"
or "Strike"
(a movie by Eisenshtein, I am teaching about montage and my thinking about
sense is
influenced" by that teaching and accompanying reading, I think of a
"perfect" montage
film (there is not such thing) as one in which the entire ensemble of
juxtaposed images brings to life in the mind of the spectator the "theme" of
the movie as a whole. A vaulting ambition if
there ever was one, but it is not accident that Eisenshtein refers to Luria
and Vygotsky and vice versa, a least vis a vis Luria.
mike

Mike, Everyone,


To me, the key in understanding "sense" is in the  following passage
[which you also touch upon in your reply to Momoko. ("it means to "flow
into" or infuse as well as "have an effect on")]:
"The third basic semantic peculiarity of inner speech is the way in
which senses of words combine and unite - a process governed by
different laws from those governing combination of meanings. When we
observed this singular way of uniting words in egocentric speech, we
called it "influx of sense". The senses of different words flow into one
another - literally "influence" one another - so that the earlier ones
are contained in, and modify, the later ones."
Maybe we can think of it as "syntagmatic" combinations as opposed to
meaning as "paradigmatic" combinations. Here to illustrate: think of a
word "butterfly" -- its meaning which can be found in dictionaries is a
concept and as a concept it is part of a paradigm that includes our
knowledge of biology, evolution, different species, etc... For each one
of us -- it is a bit different depending of the particular knowledge we
have, but it is still part of the same universal paradigm.
But, is I say something like: "remember the butterflies last spring? The
ones that suddenly appeared in your garden? You wrote about them on
XMCA. ..." Then, in that context -- which is syntagmatic -- because it
puts "butterflies" into a picture, a definite event -- "butterflies"
starts getting a peculiar sense, a particular sense which includes many
more "things" in your memory including a feeling of suspense about why
is that particular event mentioned again, what is its purpose in the
ongoing dialogue...
To rephrase: "sense" is both a history of the previous uses of the word,
a particular shape of the building of the meaning of a word, AND it is
also the MOMENTARY use of it which puts it (and everything that it
stands for) into yet another particular syntagmatic constellation. That
is how I understand "influx of sense".
It is interesting that you mention Eisenstein -- the technique of
montage and creating meaning by juxtaposition. It is the same solution
or the same way of thinking about it.

Ana

From: momoko kashima

Hi, all.

Thank you Mike and Ana for your comments on my comment.

Both comments give me very good idea.

 

Double layers of words (sense and meaning) are becoming one of the most important topics to me who study children play.

 

In Mike words, like each piece of montage photo doesnft give a particular character of someonefs face before becoming ensemble, word meaning remains stable until it comes into context in the form of external speech.

 It seems to me what happen in the process of creating varying sense (for example, creating play drama in play, daily conversation) is transforming meaning in accordance with context. Context may give the speaker-hearer hints and it also function as medium for them to arrive at the same sense of the word.

 What I must struggle as a student is to demonstrate this process through observational data of play, which is very puzzling for me.

 

Let me try to solve this puzzle a little while answering Anafs question:

 

>I would like to know if you saw a
> difference in the relationship between word and thought in play, and
> their relationship out of play. And ! what are the criteria or the
> phenomena that you take into account as critical when studying new
> meaning making in play.

 

First, Ifll answer the first question if I saw a difference in the relationship between word and thought in play, one point come to my mind is that children allow to think and speech about fantasy. It must be very wired for adults when someone speaks about fantasy but not for children. Also, I think fantasy composes one of the peculiar features of play, which makes play conversation (interaction) different from other interaction. (I wonder if I can take your question correctly. Please rephrase it for me if I donft get your point.)

 

To answer second question, the critical phenomena when studying new meaning making play is the process of transforming object meaning into quality of meaning (Leontfev, 1981). In other words, children transform object meaning according to play context. Objects seem to provide meaning, which perform as medium for children creating new elements into play.

Maybe we can discuss this topic during week 5,6 and 7 where wefre going to read symbolic interactionism and dialogical approaches to mediation.

 

 Thatfs all for now.

 

Best wishes,

Momoko

 

Wertsch on Vygotsky, Voices of the Mind
Summary of Chpt.1 Prerequisites

In this introductory chapter Wertsch lays out some of the fundamental tenets of the sociocultural approach to the study of mind. He begins by framing this study as a choice between two research agendas that need to be addressed, “and, where possible, integrated”: the study of universals of mental functioning vs. an emphasis on the relationship between “mental processes and their cultural, historical, and institutional settings,” (7). Whether one adopts the former or the later stance, one must still grapple
with the question of what counts as an appropriate description or explanation of the phenomena being studied – i.e. what is the unit of analysis?

ACTION: Wertsch believes that human action, specifically mediated action, is the appropriate unit of analysis. In shifting the research focus from the individual and the environment to action, “human beings are viewed as coming into contact with and creating their surroundings as well as themselves through the actions in which they engage,”(8). This perspective contrasts with two traditional approaches to conceptualizing the relationship between individuals and their environment: the Lockean perspective which holds that the individual is a “passive recipient of information,” and the Cartesian perspective which focuses on the individual while viewing the environment as “secondary, serving merely as a device to trigger certain developmental processes.”(8)

Habermas’ categories of action form the basis from which Wertsch develops his notion of action. Habermas’ concept of action is in turn derived from Popper’s “three-world theory” which distinguishes between physical objects and states, mental states (states of consciousness), and the world of “objective contents of thought.” There are four forms of action in Habermas’ typology: teleological, dramaturgical, normatively-regulated, and
communicative action. Teleological action refers to the idea that there is an agent that can decide among a variety of means in order to bring about a desired state. Dramaturgical action relates to what Goffman calls “impression management.” This is the notion that an individual, conscious of the image s/he is portraying to those around him/her, strategically stylizes this image in order to attain certain goals. Normatively-regulated action relates to Popper’s “world of objective contents of thought.” In this form of action individuals in social interaction orient their behaviors according to the norms that are appropriate to the context in which they find themselves. Finally, communicative action refers to the attempt by two or more individuals to “reach an understanding about the action situation and their plans of action in order to coordinate their actions by way of agreement,” (11).

MEDIATED ACTION: For Wertsch human action does not take place in isolation. It involves the use of mediational means such as tools and language. In studying mental function the focus is not just on the person in action but the person engaged in action in conjunction with mediational means.

VOICE: Wertsch’s use of “voice” in this book is borrowed directly from Bakhtin. Voice here refers not only to speech, but to the expression of personality, of consciousness. Wertsch’s use of this notion of voice reflects ideas found in both Vygotsky and Bakhtin’s work: a.) an understanding of “human mental action” requires an understanding of the semiotic tools that mediate this action; b.) an awareness that some forms of human mental functioning are “fundamentally tied to communicative processes”; and c.) an understanding of human mental function requires some from of analysis that is genetic or developmental in nature.

VOICES: Werstch’s use of the plural form is meant to reflect his belief that when engaged in research reality can be represented in multiple ways. We must try and understand what makes one or several voices dominate in certain contexts.

MIND: Wertsch employs the term mind (vs. cognition) in order to try and encompass the “wide range of psychological phenomena” – not just cognition, but also emotion, and self-identity. Furthermore, borrowing from Geertz and Bateson, Wertsch wishes to advance a notion of mind as socially distributed.

SOCIOCULTURAL: Finally Wertsch explains his use of the term sociocultural. Again he reiterates what he said at the beginning of the chapter – his central concern is with understanding the relationship between mental action and its situatedness in cultural social and institutional settings.

From: "tamara ball" (tball@ucsc.edu)

Hello everyone... Tamara here from UCSC
Mike's recent nudge worked ( although threats of very unsatisfactory
will not work since I am already up to my ears in credits.... but no,
not lurking intentionally... just distracted by everything going on
here)

As I understand it, understanding that dialectics are happening
everywhere, and always: agent [subject]- agent; agent-position (
identity/role); agent- object; agent-instrument [artifacts and cultural
("artificial" )context] ; agent-institution (including legitimacy norms
and formal rules) is the understanding that moves past Lockean
(individual's as passive recipients) and Cartesian (environment as
separate and secondary) perspectives. I also understand that Wertsch's
, among others, ( I think very useful) insistence on ACTION as the
most appropriate unit of analysis is to insist that we "live in the
middle" or continually train our focus on the dialectic. Of course we
must then spend considerable time defining and agreeing on the
parameters of that unit - ACTION. And then comes joint-attention,
communication thus mediated action. So I am getting more and more
comfortable accepting that yes indeed mediated action and auxiliary
stimuli are key to the way past the mind-body or mind-environment
divides . I am comfortable accepting that this mutual constitution
goes on - I just wish I had a better idea of how it all got started in
the first place.

So here I have two questions about trajectories :

?1. as this developmental process continues over time, do each of these
dialectical constituents become increasingly interdependent? So that
each moves increasingly towards becoming what Dot Robbins (ISCAR, 2005)
explained as a hologram? ( where each part is really nothing less than
a microcosm of a whole) If not what kind of trajectory do we imagine
results from the persistence of dialectics over time?


?2. And another query: while I am comfortable accepting that this
mutual constitution is the way of the living world - I get caught
wondering if we really do get to continue to talk about person engaged,
individual, or agent? Even Wertsch talks about the "irreducible
tension" between agent and instrument - which leads me to assume that
even he assumes it is at least worth keeping a place-holder for the
individual. But if individual minds and identities, consciousness
and/or self-consciousness really are only the products or outcomes of
continuing dialectical inter-action.... (rather than something like
free will impelling the process) then maybe we are just bluffing
ourselves by keeping that place-holder.... On the other hand I am
quite enamored by the idea that I am one... and so was Rosa Parks.

(Ok but you can probably ignore those last two because they really are
stupidly leading to metaphysics and the pragmatist in me is fairly
sure it is not worth drawing them out... this next one might be more
palatable)

?3. Robert L.'s comment about Wertsch's use of mind rather than
cognition, because it encompasses a notion of mind as "socially
distributed" reminded me of a discussion we had in the UCSC CHAT
Class with Gordon (earlier this fall) about situated cognition vs.
distributed cognition - I would be interested to know what the UCSD
contingent understand as the important things to recognize here? For
instance - The blind man's sense of where he is - is distributed
across his mind, hand, walking stick and the "environment" the walking
stick encounters... altogether and at once... or we might say the
man's sensemaking is mediated by all this... but what is the
difference between conscious awareness being distributed or situated
by both tools we use and the particular environment?
Tamara Ball
doctoral studies
Education Department UCSC
email: tball@ucsc.edu
home phone: (831) 420-1080

From: "M Cole" (lchcmike@gmail.com)

Sorry for slow uptake. Tamara--

We discussed some of the issues you raised in our meeting yesterday, others
on Wednesday at the Dist Cog lab.
We discussed the issue of action as unit of analysis (vs activity) and also
the significance of talking about sociocutural/cutlural
historical. And we also discussed how it seems that on the one hand CHAT
gets criticized for not incorporating organizations
and social class into its analysis (on the one hand) and for eliding the
individual on the other. Two strikes!

The readings for this coming week take up these same topics, but now with
activity rather than mediated action as the unit
of analysis. And, note, that in the LSV’s division between tool and sign he
has them as different kinds of mediated ACTIVITY, not
ACTION. We wondered if he was the one who used activity or if he got some
help in the 1950's when this was first published
in the USSR from his students in the psych dept at Moscow State U which was
headed by Leontiev.

Hard to say.
mike

From: "Steve Gabosch" (sgabosch@comcast.net)

I have not infrequently wondered why the "conscious act" is not a, or even the, primary unit of analyis for human higher psychological functioning.  On the surface, it seems to follow the same logic that word-meaning does in LSV's analysis of verbal speech (inner and external), that the reflex follows in elementary functioning, and that the water molecule follows in hydrochemistry.

- Steve

From: "M Cole" (lchcmike@gmail.com)

We discussed this at length in the seminar on Friday, Steve.
Seems like one of the other seminar members might summarize that aspect of
the discussion, but if that proves a problem, i will try to help out.
mike

Wertsch Voices of the Mind
Chapter 2 “A Sociocultural Approach to Mind”

 

From: "emma johnson" (ekjohnso@ucsd.edu)

So sorry to post last minute—night before class (for those of you attending the class). Life just takes over sometimes—more often than I usually bargain for.

Wertsch begins by pointing out (reminding us?) that sociocultural and universalist approaches are not mutually exclusive. He also explains that most sociocultural studies have been done in a comparative (anthropological) mode. The chapter is focused on introducing three basic themes in the Vygotskian approach. Wertsch closes with a short discussion of the linkages between Vygotsky and Whorf. The chapter is close to 30 pages, so this summary is kind of an overview of an overview.

The three basic themes from Vygotsky are:
1. Genetic/Developmental analysis
2. The importance of social life
3. Human action is mediated by signs and tools.

These three themes are interrelated, and their power comes from this interrelation. However, Wertsch chooses to separate them for clarity.

1. Genetic Analysis (begins on P19)
On page 20 Wertsch quotes “it is only in movement that a body shows what it is” (this quote reminds me of the changes that accompany early photographic motion studies on bodies—the shift from anatomy to physiology). But the gist is that Vygotsky’s approach is concerned with process, not product. Many different trajectories can lead to the same outcome.
There are several distinct genetic domains. Vygotsky focused mostly on individual development during childhood—partly for political reasons. But his work also touches on phylogenesis (he uses Darwin and Engels), sociocultural history, and microgenesis (an example is that he didn’t want to throw out the data when training subjects for experiments in lab settings). There are different ways to conceive of the relationships between these domains. Haeckel and Hall are cited as theorizing that the individual recapitulates the history of the species. Werner instead proposes a parallelism—where there are general formal rules but not a recapitulation. Vygotsky rejects all this and insists that the domains are unique. While there are formal similarities, there are different kinds of developmental forces at work in the domains. This kind of "critical point theory” is criticized by Geertz and Sinha.

2. Social Origins of Mental Functioning in the Individual (begins on P25)
In particular, the emergence of labor and speech move the important genetic domain into the sociocultural, rather than evolutionary. This is where he takes up some pretty general statements from Marx and, it what Wertsch says was a normal practice at the time, combines Marx with other thinkers to provide a Marxist framework for his research. So, everything appears first on the social or intermental plane first, and then it is internalized—although not as a direct copy.   ********
An idea from Vygotsky that has been widely taken up is the Zone of Proximal development, in which adult-child (or expert-novice) dyads work together to complete tasks which are to difficult for the child/novice to complete unassisted. Vygotsky argues that potential functioning is as important as actual—and that both teaching and assessment should be aimed at the potential.

3. Mediation (begins on P28)
Vygotsky made a big contribution in his focus on psychological, rather than technical tools. He looks at how language mediates action, so he doesn’t see it as a self-contained system (example of the “forbidden colors task”—and the importance of the combined motor method). Wertsch points out that Vygotsky has a real focus on verbal mediational means—which is perhaps due to his intellectual roots. This can seem very natural in western settings, but it is clear that there are probably other kinds of memory tasks, and problems that are best solved with other strategies.
Wertsch points out that Vygotsky’s focus on intermental functioning doesn’t give such a clear picture of the ways that larger social/institutional forces can shape mediational means. He gives some examples to counter a kind of folk idea that mediational means are designed to best meet the needs of users. The first example is QUERTY vs. Dvorak keyboards—the QUERTY having been designed by a typewriter manufacturer to slow down typists—and therefore prevent the keys from sticking. And yet, it remains dominant, even though it is pretty easy to switch. A second example is of sorting children in schools (from Mehan), and the kinds of bureaucratic decision-making that then do far more than describe the children who are sorted—but serve to constitute (part of) their identities. Mediational means can end up shaping the setting.
Semiotic Potentials: because of the social construction of mediational means, language can end up being better suited to some uses over others, and this is not necessarily consistent with the ideal individual mental functioning. This section of the chapter discusses two major areas of “semiotic potential” that Vygotsky investigated: decontextualization (in scientific or academic language use) and increased contextualization. Language in scientific and academic settings can become decontextualized when it becomes an object of reflection or study in itself, and where the relation of words to each other is foregrounded—rather than the relation of words to actions. Inner speech and egocentric speech represent speech becoming its own context—which Vygotsky calls increased contextualization, and Wertsch proposes could also be considered a “recontextuatlization. (Other readings deal with inner and egocentric and inner speech in more detail so I won’t do that here).

*Vygotsky and the Whorfian Hypothesis (begins on P43)
Following Boas and Sapir, Whorf develops ideas about how spoken language affects thought. Boas rejects a unilinear development—an assumed progression from primitive to modern, which Vygotsky assumes. Whorf and Vygotsky choose very different units of analysis, proposition/sentence and word meaning, respectively which lead to very different processes and conclusions (also may partly reflect the structure of the languages they studied). In this area Wertsch argues that they can be possibly complimentary. But they are very different in assumptions about language- Whorf assumes it is referential, and Vygotsky that it is functional.

-Emma

Comments on Unit of Analysis from I don’t know where

 

From: "Mike Cole" (mcole@ucsd.edu)

All-- this discussion appeared on a yahoo group devoted to preparations for AERA seminars on chat. I believe it started here so I am posting it here. Would those who wish to post it on AERA chat group please cc this group?
mike
-------------------------------
Gordon, Ana and All,
So if I understand, we are talking not so much a typology of activity
(eg. reflexive, mechanical, reflective), but more an identification of
of attributes (eg. context such as time, space, engagement)... and is
that vs the attributes of mediated action (eg. tool-and-result)? Are
there different attributes that correspond in terms of identification
(potentially variables?) as different units of analysis for different
purposes? Which units of analysis are more readily available for
causal types of analysis - including different kinds of causality, (eg
Maxwell's how vs why)?
Also, I am still troubled by conversation, play, etc. however. What is
the appropriate unit of analysis when the activity overtakes the
participants? For me this would also include working in the ZPD.
~ Em

--- In MC-CHAT-2006@yahoogroups.com, Ana Marjanovic-Shane
(anamshane@...) wrote:
>
> Gordon and all,
> this is a great point you make: "the biggest problem is the issue of
> Time and the various time-scales on which Activities, Actions and
> Operations are enacted ".
> It is important to think of these "units" as being exactly what you and
> Emily say: dependent on the scope and the purpose of the analysis.
> However, it will be also very interesting to discuss how they relate to
> each other. As you said, one can think of the Engestrom's triangles as
> somehow depicting some of those relations and I would love to hear more
> thoughts regarding representations like that - and especially as they
> relate to concrete research issues and settings.
> Another point that I would love to address in the course is the use of
> terms "activity", "action" and "act" ("operation") in this theory as
> opposed to the intuitive everyday use.
> And since Emily also thinks that "mediation" point of view is not
always
> pertinent to the analysis, I would also like to hear some discussion on
> what does "mediation" mean for different people and in different
> research contexts.
> Ana
>

 

 

 

 

 

 

 

 

 

 

 

 

 

 

 

 

 

 

 

 

 

 

 

 

 

 

 

Week 3

 

Section 3.1 of Leont'ev's, Activity Consciousness and Personality

From: "david leitch" (dleitch@ucsd.edu)

Punchline: Psychology must stop modeling the internal and external worlds in opposition, and instead use activity as a category to mediate between the two.

Leont'ev begins this chapter with a claim that Marxist (and Feurbachian) materialism understood the importance of human activity, not just contemplation, and it understood that activity in a concrete, rather than abstract way (as opposed to idealism). This renders the psychological subject too passive in non-Marxist psychological theories; man reacts to the environment in ways determined by his training. This has caused psychology to split into two fields, one philosophical and one based on the natural sciences model.

This split has encouraged two disappointing tendencies in modern (circa 1978) psychology. On the one hand is the reductionism of the natural sciences model. On the other is a bland form of interdisciplinary studies, in which family resemblance takes the place of actual conceptual rooting, and the disciplines themselves are unaffected by the outcomes of the studies. Only a new psychological system, which draws on a variety of different disciplines but finds the actual concrete common grounding, can overcome these problems.

The very core root of this split is the reigning binomial method of analysis: “action on receptor systems of the subject -> resulting response – phenomena (subjective and objective) evoked by the given action.” (ACP 46) This method reaches its logical conclusion in behaviorism's distillation of it into the stimulus/response (SR or S -> R) formula and the postulate of directness. This formula removes life from psychology entirely; it, “excludes from the field of research the cogent process in which real connections of the subject with the object world, his objective activity, are made.” (ibid) In short, it models psychology around a false opposition between the objective, external world of activity, and the subjective, internal world of intention.

Thus, behaviorism and other pre-Marxist psychologies to create mysterious forces that their theories could not fully account for in order to explain directed human behavior. Even sophisticated intervening variables were insufficient, in that they were unable to account for the uniqueness of humans; for example, Tolman et al and Rubinshtein's writings on subjective interpretation were unable to add anything meaningful to SR, because they only show that expression occurs differently depending on conditions, something true of both the animate and inanimate. Theories of cultural mediation, such as White's in The Science of Culture, also fail to add sufficiently, because they treat cultural as a mediator that does not interact with either the subjective or objective worlds. SR still holds, but both the stimulus and responses are cultural, rather than material; the binomial still holds, though. The only difference between man and beasts is the replacement of bodily condition w! ith culture, according to White. American models of cultural mediation remove will from man, treating him as a node of cultural expression (to me, seeing American psychologists, rather than French, accused of this is surprising; in political theory, the place of will in Foucault's work is a major question).

Cybernetics, which had originally seemed promising, cannot overcome this split on its own, because certain psychological realities (Leont'ev lists psychic image, consciousness, motivation and purpose here [ACP 49]) could not be explained in a way that was both scientific and cybernetic. Since these realities were the same ones that Tolman et al, Rubinshtein, White, and American psychologies could not grapple with, this left cybernetics as it is unable to move forward.

All of these theories have in common a belief in SR, and their 'advancements' come in the form of complications or add-ons to the basic oppositional model. The development of a category of object activity into psychology, however, is a rejection of SR, and therefore has some promise for moving beyond it. The basic model of activity is trinomial, rather than binomial, as theories of behavior or its neuropsychological underpinnings are. The introduction of a third term, “the activity of a subject and, correspondingly, conditions, goals, and means of that activity,” allows for a mediative, rather than oppositional model of the relationship between the internal and the external. (ACP 50)

This formulation alters the basic psychological question of, “Who or what am I?” into two possibilities: consciousness is determined either by the objects around a person, or by their social life. Interestingly enough, Leont'ev never considers the Kantian possibility that consciousness is characterized by its self-determining nature, and, at least in this section, he dances around the possibilities that Vygotsky's view of consciousness as 'active reflection,' which Vygotsky borrowed from Lenin's lectures on Hegel opens in developing a model of consciousness as something beyond an epiphenomenon.

Section 3.2 of Leontiev’s activity consciousness and personality

 

From: "emma johnson" (ekjohnso@ucsd.edu)


There are some things in this (short) selection that I am just not understanding. Perhaps some of our ethereal participants, or more embodied class members, can clarify? Or is this just an example of Joanne’s point about the difficulty of boundary (or was it border?) crossing.

In this segment, Leontiev gets down to business, describing what psychology should be doing by studying activity (rather than its errors). Activity is a system, with structure. “…whatever kind of structure it assumes, [activity] must not be considered as isolated from social relations, from the life of society” (p51). Activity is not a narrative about the individual struggling AGAINST society, because society is not exterior to the individual, it is on the inside too.

 

The concept of objectivity is contained in the concept of activity, and the prehistory of human activity begins with the acquisition of objectivity—and as evolution continues, the objective world intrudes further into activity. Psychic reflection develops in order to regulate the activity. Activity precedes psychology?

Activity has a circular structure—and this vocabulary is obtuse for me (afferator, effector, etc) but I think its basically describing the way that the world impact the person, who regulates that psychically, and then in turn corrects/changes the environment. But have I gotten this wrong?
Luckily (since I am a bit confused), the circular structure is not the point, rather, the way that psychic reflection is not directly generated by forces, but rather by the practical contact with the world. Needs and emotions are part of this category of activity too. For instance, hunger itself merely gives greater urgency to getting food—it doesn’t direct the activities. A need must “meet with” an object before it can direct activity. A need must be “filled” with content from the world around it, objectifying it. This explains the development of new needs—the s that fill needs are produced, therefore needs themselves can be produced.

So, in addition to requiring clarification on the circular structure, I can’t seem to wrap my head around the following quote (at the top of p52):

“Thus, the object of activity is twofold: first, in its independent existence as subordinating to itself and transforming the activity of the subject; second, as an image of the object, as a product of its property of psychological reflection that is realized as an activity of the subject and cannot exist otherwise.”

Does this mean basically that activity has as its object both what we would think of as something “out there” that directs and bounds activity, and yet is simultaneously also “ideal” in the mind of the subject? I keep reading this sentence, and can’t quite separate these two objects. Or is that partly the point—that they can’t be separated?

Emma

 

Leont’ev, Ch 3 - The Problem of Activity and Psychology Section 3.3


Joanne Price



In Sections 3.1 and 3.2 Leont’ev describes the problem of the ‘bionomial plan of analysis’ (the stimulus-response, or S-R) approach to psychology:

“The inadequacy of this scheme is that is excludes from the field of research the cogent process in which real connections of the subject with the object world, his objective reality, are made.” (p. 46)

And by Section 3.3, Leont’ev has established the necessity of including external activity as part of the study of psychology. In this rather short section, he begins to consider if there are certain types of external activity that should be the focus of psychology research (while other types should be excluded). He characterizes the ‘old psychology’ as one that studied only ‘what goes on in the mind of man.’ However, in a manner somewhat similar to Bateson’s decrription of the cane (though, for me, far more abstruse), Leont’ev points out that objects shape what goes on inside the head and people shape the objects; thus, they cannot be artificially separated in research:

“This means that it is incorrect to think that although the external, objective activity presents itself for psychological investigation, it does so only to the extent that it includes internal psychic processes and that psychological investigation advances without studying external activity itself or its structure.” (p. 56)

Reflections on the Reading:

More than anything else this reading was instructive of just how frustrating it is to listen to ‘insider’s talk’. Had this not been an assigned reading, I would have long since pulled off it. Happily by Section 3.3, (my section) Leont’ev started to speak less referentially and somewhat more directly. I had the feeling that Leont’ev was aware he was involved in what I will call “writing to the inner circle,’

“Of course, I am omitting here [in Section 3.2] any statement of the concrete, scientific basis for the theoretical positions referred to, just as I have in the evaluation of the problem of their internal connections with the teaching of I. P. Pavlov about the signal function of conditional stimuli and about orientating reflexes; I have explained both of these points in other papers.” (P. 52)

Let me be quick to note that I am not criticizing the writing, but simply pointing out what I believe to be a very serious problem in ‘boundary crossing.’ I have a relatively high tolerance for ‘struggling through the vocabulary’ to find my footing. And for me, reading this piece was excruciatingly frustrating.

My recent thoughts about workplace learning have centered around the fundamental importance of organizational artifacts—all the various types of reification of ‘organizational knowledge and experience—required for organizational learning. While there is considerable formal ‘reification’ of knowledge and experience, particularly in the form of reports that come from the outside (from consultants), as well as informal artifacts that never enter the organizational stream, I have noticed there is relatively little access to that knowledge. Using the metaphor of an individual to portray the entire organization, this would be akin to each organ of the body, ‘doing it’s own thing…with little reference or organization of the mind…in terms of this week’s reading, activity without mind!

Since the organization tends to be perceived more in terms of its individual members, or individual functioning units (like the organs of the body), there seems to be little focus on the coordination of these functioning units (collective activity). It is no wonder, therefore, that there is not much concern about incorporating these artifacts of organizational knowledge and experience.

My further question, in line with the experience this week’s reading, is when there are attempts of one group trying to access another group’s information and experience, I wonder just how accessible that information is to an ‘outsider.’ The paradox is that in order to make progress in highly specialized areas, the language must of necessity become ‘packed,’ but it is that very characteristic that makes ‘boundary crossing’ a formidable prospect.

Using the same ‘body’ metaphor, we do not expect the stomach to ‘speak directly’ to the feet (‘hey feet, go get me some food’)…our brains do that. Organizations have organs, in these days of high specialization and the attendant specialized languages…we have ‘lost our heads.’

Leontiev – The Problem of activity and Psychology  3.4 The relationship of Internal and External Activity

 

From: "Robert Lecusay" (rlecusay@ucsd.edu)



In this section of the chapter Leontiev introduces the concept of interioriazation, which we later learn refers to Vygotsky’s notion of internalization. The definition of internalization we encountered last week (“The internal reconstruction of an external operation,” (Vygotsky, Mind in Society, 56)) is expanded:

“a transition that results in processes external in form, with external material objects, being transformed into processes that take place on the mental plane, on the plane of consciousness; here they undergo a specific transformation - they are generalized, verbalized, condensed, and most important, they become capable of further development which exceeds the boundaries of the possibilities of external activity.” (Leontiev, 58)

Leontiev then goes on to note that Vygotsky distinguishes between two “principal interrelated features that must be considered basic to psychological science” which Leontiev describes as the “instrumented”, or equipped, structure of human activity and the incorporation of this structure into the system of social interrelationships. (More on this in my questions below.)
Leontiev reiterates the primacy in the process of internalization of external representation (“in a form of action or in the form of external speech,” (59)) in social interaction. It is in this form of interaction that higher psychological processes originate. He continues with one final qualification regarding the relationship between “internal psychological activities” and “practical activity.” In this perpetual transformation of internal and external structures “simultaneously there takes place a change in the very form of the psychological reflection of reality: Consciousness appears as a reflection by the subject of reality, his own activity, and himself. But what is consciousness?” Yes, what is it? Leontiev goes on to offer some answers. He describes consciousness as “co-knowing” (“in that sense that individual consciousness may exist only in the presence of social consciousness and of language that is its real substrate”); as originating in society; as the consequence! of internalization.
Here Leontiev brings us back to the discussion in “Thought and Word” where Vygotsky argues for word meaning as the unit of analysis for studying consciousness. (I am unclear as to the meaning of his articulation of Vygotsky’s position: “Not meaning, not consciousness lies behind life, but life lies behind consciousness.”) Leontiev argues that this approach, though essential for understanding the process of internalization, does not paint the whole picture: “Meanings in themselves do not give rise to thought but mediate it -just as tools do not generate activity,” (60). Wertsch’s Vygotsky and the Social Formation of Mind further examines this critique (in fact, his discussion of activity theory in this book helped keep me from feeling totally lost while reading this chapter!)

Leontiev continues referencing ideas contained in “Thought and Word.” He notes that Vygotsky “saw the last remaining ‘secret’ plan of oral thinking in its motivation, in the affective-volitional sphere;” This, I think, refers to the “final internal plane of verbal thinking” (Thought and Word, 283). I loose Leontiev in his subsequent argument: “The positive program resulting from this [the deterministic view of psychic life], having preserved the active function of meaning and thought, requires that the problem be considered once again. And for this it was necessary to turn to the category of objective activity, applying it also to internal processes, the processes of consciousness.”
This, and other claims in this section and in 3.5, raise some questions for me (be warned, I’m posting this without yet having read the other readings assigned for this week. I may get my answers in these other readings.):


- What are the differences and similarities in the way Leontiev and Vygotsky (in “Thought and Word”) use the term “motivation?” And if there are differences, how do these tie into Leontiev and Vygotsky’s conceptions of internalization? I ask because Leontiev indicates that loss of motivation is one factor that contributes to the continual transformation of activity, which in the scheme he lays out in this chapter, as far as I am able to understand it, includes both activity as an “external” and an “internal” phenomenon. Which brings me to another question:

- What is the difference between “psychic reflection,” (Leontiev, 67) “internal activity” (Leontiev, 61- 62)? How does this compare with Wertsch’s “mental action,” (from the intro to Voices of the Mind)?”

- What does Leontiev mean by “cognitive motive?” (68)

- In our seminar meeting last week we discussed the notion of thought being completed in the word (from Vygotsky’s “Thought and Word” chapter), and, if I remember correctly, Mike argued that thought isn’t really completed, that it moves from person to person, gets interpreted and reinterpreted. Is this what Leontiev is referring to when he advocates an analysis of an activity’s internal systemic connections in order to understand activity, “as a process that is characterized by continuously proceeding transformations”?(67) Is viewing “thought as completed in the word” simply a distinction that should be made for analytic purposes only? Internalization refers to a process which suggests at least a stage in which the interpsychological function is represented as a transformed (not identical - “external and internal activity have a similar general structure,” (Leontiev, 61)) intrapsychological function. So there is completion in the sense of a completed change. Is thought incom! plete when it is not articulated? Again, I think this requires clarification of how Leontiev applies the term “activity” when he is talking about “internal” vs. “external.”

Confused,
robert

 

From: "david leitch" (dleitch@ucsd.edu)

(Note: I'm at least as confused as you are.)

Isn't "activity" supposed to break down the binomial of internal v external, anyway?


Engestrom, The Emergence of Learning Activity
as a Historical Form of Human Learning (First Lineage)

Antonieta Mercado February 3, 2006



The Learning Paradox

The discussion of the “learning paradox” is an attempt to reconceptualize learning theory from a developmental perspective. The learning paradox, according to Bereiter is the attempt to grasp not only the child learning a theory, but the child acquiring a theory to be tested. Engestrom argues that at the metatheoretical level, the problem of the learning paradox is how a structure generates another structure more complex than itself, and at the theoretical level is that how the development of complex mental structures are represented by mechanisms that are not necessarily highly intelligent or full of knowledge?

Bereiter questions how to explain intuition and creativity?

Engestrom mentions that Bereiter cites Vygotsky when clarifying that learning depends on cognitive structures that are situated in the culture, rather than the individual. Children acquire those structures through the interaction with adults. With adult guidance the child “internalizes” cognitive structures. However, Bereiter criticizes Vygotsky explanation, because this internalization is where the “learning paradox” resides, and it is not accounted for in Vygotsky (p. 2).

Engestrom questions if it is true that the learning paradox resides in “internalization”. Bereiter discusses higher forms of learning as “creation”, but creation seems to refer only to new cognitive structures, subjectively, inside the head of the individual (p. 2).

Is learning internalization? Is creation only internalization?

Klix questions the laws of learning, and finds that it is not an invariant process. Klix points that there are two classes of learning in humans and animals, the first is conditioning while the second refers to cognitive learning. With cognitive learning comes a particular adaptive behavior. Some primates even have early forms of cognitive learning, like hypothesis checking (as dogs).

Human learning is learning within the head of the individual (Klix). Engestrom formulates the following questions to assess human learning and internalizations:

Is the evolution of learning the story of progressive enlarged capacity for internal individual processing of information?
Is man finally leaving behind the restrictively specific influence of environmental properties?
Is man’s crucial feature simply the fact that he thinks more than his evolutionary predecessors?

Engestrom cites Zinchenko’s assessment of involuntary memory, and points out that Zichenko explained that involuntary memory and mechanical memory are not the same. He talks of involuntary memory when the act of remembering occurs within an action of different nature, that is not directly directed to the task or remembering (Engestrom, p. 4). Involuntary memory changes with the subject’s activity or actions, it is a byproduct, but it is not a mechanical one. Voluntary memory on the other hand is not necessarily non-mechanical. It involves an action directed to remembering, where the subject is aware of what is being called to remember. In this process, the subject is not passive, and logical memory can be both, voluntary and involuntary. Thus, the division of memory between mechanical and logical is false (p. 5).

According to Zichenko it is illegitimate to treat earlier types of learning as “conditioning” and higher or later types as “logical”.

Involuntary learning takes place in action, it is also a byproduct of action. Conscious learning are also higher formation. For learning to take place, we should have conceptual means to analyze activities (p. 6).

Triangles of Activity

In the 19th century biology, philosophy and social sciences experienced important breakthroughs. In philosophy, Hegel was the first to bring attention to material, productive activity and the instruments of labor in the development of knowledge. The Spirit for Hegel, was the accumulation of knowledge in humanity. The Spirit was the consciousness, which was the reflection historical knowledge represented in individual action (p. 6). In biology, Darwin discoveries, lead to the elimination to the appeal to supernatural forces in scientific explanations. Marx and Engels, on the other hand, changed social science by introducing the idea that human beings are not only products of evolution, or assimilators of culture, but have the capacity of transforming and creating (p. 7).

The sciences in the 20th century have not meet the challenge of constructing theoretical instruments to assess activity: where the circumstances are changed by men and the educator himself is educated (p. 7).

Engestrom proposes

• Activity as the simplest, genetically original structural form, as the smallest unit that still preserves the essential unity and quality of any complex activity.
• Activity must be analyzable in its dynamics and transformations, evolution and historical change.
• Activity must be analyzable as a contextual and ecological phenomenon.
• Human activity must be analyzable as a cultural mediated phenomenon.


Engestrom argues that human activity is mediation.

Prerequisites of a theory of activity go from Peirce, Ogden and Richards and Popper’s evolutionary epistemology.


First Lineage: from Peirce to Popper

Peirce one of the founders of semiotics built his theory of mediation on the idea of a triadic relationship between an object, a mental interpretant and a sign.

In Pierce, mediation is purely mental and intentional. Pierce suggests a triad with a sign (representamen), comes first and is related with a second thing named its object, and the relationship of the two previous ones, is capable of determining a third, or Interpretant. The mediating signs loses its cultural quality and reverts to individualism and rationalism (p. 10).

Ogden and Richads formulated the following diagram to explain the meaning of meaning:


Thought or reference


Symbolizes Refers to



Symbol Referent

Stands for

Meaning is a triad of thoughts, words and things.

Ogden and Richards point that symbol (word) and referent (thing) are not connected directly, but indirectly around the two sides of the tringle (p. 10). This means that there is not direct correspondence between the symbol and the thing it symbolizes. This relationship is always constructed by human beings. Since meanings are constructions, the construction of meaning is a human form of activity (p. 11).

Engestrom points out that for, Peirce and Ogden and Richards, meaning is a construction of the individual thought process, seeing meaning only embedded in language and not in material things (p. 11).

Engestrom argues that symbols are socio-historically produced and transmitted artifacts, to learn a word is to use its meaning. Word is a man-made implement (p. 11). Things are also used and produced by human beings in collective activities.

Popper conception of three worlds

• World I: Physical world
• World II: World of mental states (consciousness, psychological dispositions and unconscious states).
• World III: World of the contents of thought, and the products of human mind


When Popper explains that language is non-material but appears in physical sounds, he implies that the three worlds that he refers to are independent and of different nature each.
Engestrom offers the example of Hellen Keller, and suggests that Popper may have thought that the early, gestural language of Keller was immaterial (p. 14). Popper postulated discontinuous relations between the three worlds when affirming that World I and II can interact, and World II and III also can interact, reducing the triangle to two dyads.

Engestrom criticizes this dyadic reductionism, arguing that it destroys the interaction of the system. Instead of mediation being dynamic, the three worlds live autonomous existences. He also argues that by dividing the framework into dyads, activity loses its continuous triplicity, and the theory loses strength and becomes static.

 

Engestrom: Learning Activity article (part 2)


[continuing from "The Second Lineage" to "Evolution of Activity"]

2nd Lineage: Intersubjectivity (Mead and beyond)
Mead's 'social behaviorism' tries to overcome individualism and intellectualism of previous theories (Pierce and Popper) by positing a dynamic social process, wherein the individual is preserved, but as part of a larger social unity. Social process presents new objects in an individual's field of experience, object to which it responds and adjusts. New objects are created through symbols and symbolization, which grow out of gestures. "The function of the gesture is to make adjustment possible among individuals. . .with reference to the object or objects with which the act is concerned; and the significant gesture. . .affords far greater facilities for such an adjustment. . ." (17). Engestrom observes that the way significant gestures arise is not explained by Mead.

Contrary to Mead's verson of constructed objects, Leont'ev and Tran Duc Thao both believe that communication and symbolization derive from production. While Leont'ev believes gestures originate in work movements and while Tran Duc Thao traces gestrures to indicative signs of prehominids, both concur that there is a unity between instrumental and communicative aspects of activity. Joas, a proponent of Mead, admits that Mead's concept of action is oriented toward adaptive intercourse and not toward "material production of the new" (19).

Trevarthen observes secondary intersubjectivity in infants, and he establishes a relation between communicative and instrumental aspects of activity, unlike Mead. Nonetheless, Engestrom notes that he doesn’t pursue the "praxic mode of action", nor does he refer to symbols. Morss says neo-Meadians do not quite understand Mead, because Mead's notion of social process uses a general, abstracted other, whereas his followers substitute the dyadic relation of mother-child for this more general relationship (21). Morss says that for Mead social knowledge is public, impersonal, while for neo-Meadians this knowledge is interpersonal (shared between mother and child only), and it thus takes the form of individualism, which Mead sought to overcome.

Summary of 2nd lineage: It extends the idea of activity to social process, and symbolic exchange, though it leaves out practical material construction.

Third Lineage: Cultural-Historical Psychology (Vygotsky to Leont'ev)
Vygotsky describes signs as a new relation between stimulus and response, which has been actively created and which permits the individual to control her behavior. Vygotsky differentiates between tool and sign on the basis of lower (just tool) and higher psychological functions (where tool and sign combine). Engestrom interprets this as concurring with the emergence of significant gestures (Mead) and with intersubjectivity (Trevarthen). Wartofsky makes the distinction between primary and secondary artifacts, the later of which correlates with Vygotsky's notion of psychological tool (24). The relation between physical and psychological tools was conceived but not developed by Vygotsky, who had an intellectualist approach that focused on higher level functions. In trying to correct this bias, later action-oriented Soviet researchers neglected the issues of signs.

Zinchenko says that Vygotsky's word meaning cannot be the analytic unit, since it only expresses cognition, but not motive, emotion, and creation; he suggests using tool-mediated action. Engestrom notes that this doesn't resolve the problems he identified with word meaning; both Vygotsky and Zinchenko pursue goal-oriented cognition: "The problems with motivation, emotion, and creation seems to be unanswerable on this level. They belong to a higher, collective, and—paradoxically—less conscious level of functioning (25).

Liberated action was an idea put forth by Zinchenko, and later Bartlett, to account for an individual's psychological shift during a dangerous situation (piloting an aircraft); liberated from a self-conscious, situated time, they were said to move outside time. This issue of levels of human functioning was also mentioned by Leont'ev in relation to a beater during a hunt. Engestrom points out that in both case (pilot and beater), the individual obscures the social relations at work during functioning. In reality, the social relations of an activity controls the individual's action.

Leont'ev foresees the psychological core of what will be the concept of learning activity; he sets up the relationships among activity, action, motives, conscious goals, and operations (28). "Activity appears as a process in which mutual transfers between the poles of 'subject-object' are accomplished" (29). Also, "The notion of an individual, a child, who is all by itself with the world of objects is a completely artificial abstraction" (30). Instrumental and communicative aspects of activity were not brought into a unified model by Leont'ev, so it did not supercede Vygotsky's model of instrumental action.

Radzikhovskii points out that the genesis of activity is not understood by keeping communicative (social) action separate from instrumental (objective) action. For him, the use of the sign in the primary joint action is nonconscious and fused with the action. El-konin says that developmental psychology needlessly distinguishes between need-motivation and cognitive-instrumental spheres.

3rd lineage summary: It gives birth to activity based on material production, mediated by technical and psychological tools and within social process; the next task is to derive this from genetic analysis.

The Evolution of Activity:
The basic model of animal survival methods involves three aspects of a changing system: individual, population, and environment. The "direct links" among these aspects, produces acting alone, acting with others, and being together. However, with evolution of these relationships, there emerge three "ruptures" which depend on specific species and environmental characteristics; they are constructed between the 3 aspects above: tool-making, collective rules and rituals, and division of labor. While in some species they may be isolated ruptures, they should be understood with human evolution to have been integrally linked ("unified determining factors"). "A theory of evolution should ask how emergent tool-using capacities become integrated into the domain of intentional social action" (35).

Leakey and Lewin mention how digging stick and carrier bag can be thought of as making life easier for early hominids, but the bag ushered in a new lifestyle of collecting food and sharing. The social requirement of handling these food surpluses transforms/mediates all aspects at once (individual, population, environment) and effects the 'ruptures' (tools-making, rules, division of labor) to create new forms of social actviity: production, exchange, distribution, and consumption (which is what Marx articulates). Marx says these social activities are dynamically interrelated, not static and separate; Engestrom states this as the triangles in the model develop subtriangles to signify how production, distribution, and exchange are inherent in each of the other three, and they all come together for consumption. In short, "there is no activity without the component of production" (38). The antithetical definition of production refers to the total practice of the entire society an! d to the specific practices within a society.

Engestrom concludes (this chapter) with an overview of how this model corresponds with the original four criteria of a root model of activity (part 1 of this article). This is the smallest unified model without oversimplifying that can analyze concrete activities, and it can accounts for inner dynamic relations and historical change.

From: "judy brown" (mmjbrown@connect.carleton.ca)

Hello everyone. My name is Judy Brown and I’m posting from Canada. I have been lurking for a while and enjoying all the synopses, which I have found very useful. I’m a PhD student at Carleton University in Ottawa. I’m part of a human-computer interaction laboratory within a psychology department. For my thesis I am using a cultural-historical approach to study artifact-mediated interaction. I’m interested in the user-interface design process and how artifacts mediate this activity. I wish I could come to UCLA for the Friday lectures to meet people face to face and join in the discussion more fully. I find there are very few people in my midst who are knowledgeable about cultural-historical psychology.

I haven’t yet caught up with all the readings, having gotten off to a slow start – but I only have two more to go and then I’m there. What a fantastic reading list, not the type of material that can be skimmed. Some of the papers I’d read a couple of years ago but I found I had forgotten much because I had not understood what I was reading. I hope I am taking in more now. The parts that mostly didn’t make sense to me were the Marxist sections where explanations were given in terms of production, exchange, consumption etc.

To begin somewhere, I’ve decided to comment on the Engestrom post. While I have found Engestrom’s model very useful I am troubled with his assumption that there must be one unit of analysis. In a book I read by Yaroshevsky published in 1996 (p. 170) Yaroshevsky said that Vygotsky never did settle on one “unit” but that candidates (arranged chrornologically) included: “ ‘an instrumental act,’ ‘the highest function of the psyche,’ ‘meaning’, ‘sense,’ and ‘experience.’”. I think activity is a critical unit analysis but I think there are others and that one of the tasks of the researcher is to select that appropriate unit for the research they are doing. In fact, I think that it may be important to select multiple units of analysis within one research project so that various processes can be studied over time and something can be said about how two trajectories interact. So, while it is probably essential to track activity over time (because of the fundamental rule that all! actions appear on the social level first before appearing on the individual level) it may also be useful to track how meaning or sense or experience or action changes over time. I think that Engestrom even admits that not everything can be described as organized activity, hence he has been introducing new concepts like “knot-making” to describe other ways that people organize themselves collectively.

This post is essentially asking – need there be only one unit of analysis? Also, are there cases where individual actions are not part of an organized activity? I am very interested in hearing other people’s views on this topic. I pose the questions because I do not know the answers myself and I am hoping this is a place where such questions can be asked.


Judy

From: "Ana Marjanovic-Shane" (anamshane@speakeasy.net)

Hi Judy and all,

Your question about the "unit of analysis" is very thought provoking. In
a way it has been asked several times, but not in such a clear form. I
guess, every time we ask about interaction between different functions
or variables, we also ask about the relationship between different units
of analysis. I believe, like you do, that there should be different
units of analysis depending on a particular aspect, or a particular
focus of research.
One of the most intriguing relationships that was discussed several
times in the past is the relationship between an "individual" and the
"society" -- or in other words, what would be units of analysis on the
two different planes (zooms?) or levels of processes. I think that your
other question regarding "production, exchange, consumption" and other
economic terms used by Marx, is also a question of how to understand
these Units in terms of an individual. What kind of intra-psychological
process can be conceived in the light of these economic processes?
This is also an issue which interests me: how to put together
macro-social processes with inter-personal processes with
intra-psychological processes? How do they compare to each other or
affect each other?

Ana

From: "Anna Rainio" (anna.rainio@helsinki.fi)

Hi Ana and Judy and others!



I also find the question of the unit of analysis very challenging. I even must say that it is not always clear to me what is actually meant with this term (in a practical research) in the first place. Can it be understood as a level of analysis? Or the focus of analysis?

What seems necessary is that the unit of analysis is seen in relation to what is the research interest or the questions we want to find answers to. Perhaps for one analysis we must define one unit. But there can be many levels of analyses in one research..

For example, it seems clear that when we are interested in individual development (for example how a child develops through participating in a classroom activity with her/his peers or with adult) we must focus on the individual as a part of some activity or in relation to others but however the unit of analysis cannot be the activity itself (perhaps this is what Vygotsky in his problem of method refers to as focusing on process).

But what I find difficult is to think what is the role of individual actors and their actions when the research interest (and unit) is on the development of the whole activity itself (say, of the classroom practice)? (this is actually the very same problem as Ana posed in her reply)

Very interesting was also Judy's question: "are there cases in which individual action are not part of some organized activity?"
Looking forward to people's comments on this!

Best,
Anna

Cole, “Putting Culture in the Middle,” Part 1 (pp. 116-130)

 

From: "noga shemer" (nnsevilla@yahoo.com)  

 

This chapter develops a “conception of culture adequate to the theories and practices of a second, cultural psychology with the phenomenon of mediation” (116).  There are four main points from Cole’s summary of the chapter (144) which pertain to this first section.  These are reproduced below, with additional comments based on the main text.

 

1.  Artifacts are the fundamental constituents of culture.

2.  Artifacts are simultaneously ideal and material.  They coordinate human beings with the world and one another in a way that combines the properties of tools and symbols.

3.  Artifacts do not exist in isolation as elements of culture.  Rather, they can be conceived of in terms of a heterarchy of levels that include cultural models and specially constructed “alternative worlds.”

4.  There are close affinities between the conception of artifacts developed here and the notions of cultural models, scripts, and the like.  Exploitation of these affinities requires conceiving of schemas and scripts as having a double reality in the process of mediation.

 

Cole begins by elaborating on the Russian approach to tool mediation.  Tool becomes a subcategory of artifact.  Artifacts are “simultaneously ideal (conceptual) and material.  They are ideal in that their material form has been shaped by their participation in the interactions of which they were previously a part and which they mediate in the present” (117).  This point was also elaborated in our last seminar meeting, in which Mike pointed out the word “art” in artifact, and discussed the “ideal” aspect of a simple tool, such as a hammer.

 

This dual nature of an artifact also “asserts the primal unity of the material and the symbolic in human cognition” (118).  This addresses the debate between those who would place culture outside the individual, as products of prior human activity, and those who locate culture internally, as a collection of knowledge and beliefs (118).  The concept of artifacts, as both material and ideal, has the potential to resolve this debate. 

 

Culture is literally placed in the middle in Figure 5.1, a basic mediational structure in which subject and object are connected directly (naturally, “unmediated”) and, simultaneously, indirectly through “a medium constituted of artifacts (culture)” (119).  Actions and artifacts do not exist independently, however, and a model must be developed that can address “aggregations of artifacts appropriate to the events they mediate and to include the mediation of interpersonal relationships along with mediation of action on the nonhuman world” (121). 

 

Cole develops a three-level hierarchy of artifacts, elaborating on Wartofsky’s scheme.  Then, he addresses the ways in which artifacts are woven together in the process of activity.  The question of patterns, coherence, and cohesion in culture is another source of ongoing debate in anthropology.  Cole argues that extremes which focus on either complete isolation of events or total uniformity must be avoided.  Instead, he points to the unequal distribution of culture across individuals – “no two members of a cultural group can be expected to have internalized the same parts of whatever ‘whole’ might be said to exist” (124). 

 

Cole then turns to schema theory, discussing schemas (selection mechanisms), cultural schemas (patterns of schemas that make up a cultural group’s meaning system), and cultural models (intersubjectively shared cultural schemas).  While these structures are often seen as existing in the head, Cole argues that, like artifacts, they must be seen as “inner” and “outer.”  He writes, “I prefer to think of cultural models as systems of artifacts in order to emphasize the dual materiality/ideality of both cultural models and artifacts…” (358 n.6).  He cites several scholars who have also tried to overcome this divide – Hutchins argues that culture “should be thought of as a process,” and Bruner and Nelson “treat scripts as dual entities, one side of which is a mental representatio! n, the other side of which is embodied in talk and action” (129). 

 

The limitations of scripts and schemas to determine our thoughts and actions lead to the discussion in the next section, which addresses the “circumstances” in which thinking occurs.

 

From: "David Mather" (dmather@ucsd.edu)

I found that, in comparison with the Engestrom and even the Leont'ev texts for this week, Cole's piece was refreshingly explanatory. The overall argument just make sense to me (intuitively), and it clarifies some points that confused me in the other pieces.

While it did not address what I see as contradictions in the Engstrom model of production, consumption, exchange, and distribution (if they are always interrelated, why cordoned them off at all?), I do believe it incorporates in the initial (survivalist) model in an acceptable manner. Since his inclusion of schemas and scripts extends the argument, I would simply ask what happens to the categories of production, consumption, distribution, and exchange. Do they need to be added back in or perhaps jettisoned altogether?

Davydov

 

From: "xavier cagigas" (xcagigas@ucsd.edu)

 

I found this to be a very challenging article to understand...perhaps because of my lack of exposure thus far to activity theory.  I decided to highlight aspects of the text that seemed to crystalize what I perceived to be the essence of Davydov's argument and interspersed my own comments.

 

 

Part I:  Structure of Activity

 

Davydov offers a “new” approach to activity structure and content by adding “desire” to the structure of activity in addition to Leontiev’s needs, tasks, actions, and operations

 

“…nothing can be said about activity unless one understands spiritual or organic desire and how it is transformed into a need.” [41]  “emotions are inseparable from need” [42]

 

“Activity is, in my view, a phenomenon of social-public nature, of human nature.” [42]

 

Davydov illustrates the intimate and simultaneous connection between the internal and external aspects of activity: 

 

“…a task is a unity of goal and the conditions to achieve the goal.” [42]

 

“…the achievement of any human goal is invariably connected with transformation of certain fields of material or social reality” [42]

 

“…actions, motives and means can be included as constituent elements in the activity structure alongside task.” [42]

 

The structural elements of activity consist of:  desires, needs, emotions, tasks, actions, action motives, means used for actions, planes (perceptual, mneumonic, thinking, etc.) which refer to cognition and will.  All of these operate on multiple planes simultaneously [43] 

 

-- Davydov’s incorporation of desire and emotion in driving all action and activity (cognitive or otherwise) is actually physiologically plausible in view of Luria’s neural functional systems which are coordinated with shared social/historical activity in the real world

 

Again, Davydov uses Galperin’s specification of the orientation, executive, and control elements of action which correlate with activity to illustrate the primacy of desire in shaping activity.  Control = Attention and Will is control through attention

 

Attention “is a control mechanism functioning in the process of realizing the plan to achieve a goal and it relates to will.” [43]

 

Types of activity:

1)      historical =  productive (work, artistic), moral (law, religion) & sport

2)      ontogenetic = object manipulatory activity, play activity, learning

 

Davydov asserts that there are no broad activities such as perceptual, mneumonic, or thinking activities…”there are thinking actions—because they are aimed at fulfilling thinking tasks.  There are motives to perform those actions, but the very thinking tasks stem from human needs and emotions…” [44]

From: "Steve Gabosch" (sgabosch@comcast.net)

Steve Gabosh

"I do not understand the Davydov so you are going to have to help!"
- mike

LOL!  I am not so sure how well I understand the Davydov article myself, but I can tell you why I like it.

First, this keynote address to ISCRAT 4 in June, 1998 was the year Vasily V. Davydov (1930-1998) passed away.  From looking over old xmca archives, many were shocked and saddened by the unexpected loss.  That speech strikes me as a high water mark in Davydov's important career as an international figure in cultural-historical psychology, CHAT, activity theory, and especially, how to apply these ideas to education.  I get the impression he used that forum in Aarhus, Denmark to offer some of his most interesting and advanced ideas regarding CHAT.

Second, I agree with Mike and Xavier that this article is very challenging to understand.  I don't find it cohesive, and in an important way, Davydov himself explained that it wasn't going to be.

A! t the beginning of the article he says his report will consist of two parts, a description and grounding of the "structure of activity," and a presentation of the "cell of activity content."  But then he explains:

"I have not succeeded yet in connecting the structure I am going to tell you about with the cell because to do this requires a new and original approach in understanding activity. I am searching for this new approach but until now I have not found any.  Therefore, I will present the two new ideas - the structure of activity and the cell of activity content - separately (and you will realize they are separate)."

I found myself reading the article more as a collection of interesting insights and provocative ideas rather than a theoretical narrative.  I like Davydov's approach because it relieves me of having to critique his overall argument. He isn't really making one.  Instead, I feel treated to a somewhat rambling but pleasant pre! sentation of intriguing ideas that he had been accumulating fo! r many y ears.  Instead of critiquing these ideas one by one, I find myself letting them sink in and provoke me.  It isn't so much that I agree with Davydov's thinking at every turn as that I find his imaginative thinking process agreeable.

So here are some of the points Davydov makes in this article that make an impression on me and give me food for thought.

1. Davydov sees Rubinshtein and Leontiev as the classical architects of activity theory, with important advances made by Bibler and Brushlinsky.  He also praises more recent work by Gromyko, Schedrovitskii, Engestrom, and Elkonin.

2. Davydov bases his hypothesis about activity structure on Leontiev and others but goes farther.  Leontiev did not consider "desire" as an element of activity structure, but it should be so considered.  Nobody has investigated the transformation of desire into need.  However, nothing can be said about activity unless one understands spiritual or organ! ic desire and how it is transformed into a need.  A desire is the core basis of a need.

3. Practically nothing is known about real human emotions, either.  Emotions are inseparable from needs.  We cannot discuss one without the other.  When we discuss a certain emotion we can always identify the need the emotion is based on. 

4. Emotions are more fundamental than thoughts.  They are the basis for all the diverse tasks a person sets for themself.  The general function emotions perform is they enable a person to set a certain vital task by enabling that person to decide whether the physical, spiritual and moral means that are needed to fulfill that task are available.  If the emotions say "yes," the analytical apparatus is set in motion and action proceeds. If they say "no," the person refuses to take up the task.

5. Needs and desires make the basis on which emotions function.  Human activity is based on desir! e, need and emotion, not just needs and motives.  In fact! , motive s are not fundamental to activity, as Leontiev argues.   Motives, based on the needs created by desire and guided by emotions, generate actions.  Davydov is suggesting that rather than being fundamental to the activity structure, motives appear farther down in the chain of events. 

6.  In summary, Davydov urges that desire and emotion be seen as fundamental to the formation of needs, motives and actions.  He lists the structural elements of activity as the following: desires, needs, emotions, tasks, actions, action motives, means used for actions, and planes of cognition and will.

7.  Neither Davydov's analysis of tasks and will nor his typology of activity types make much of an impression on me.

8.  Davydov in "Part 2," as Xavier neatly puts it, discusses, in the form of four theses and commentary, his ideas about the basic cell or unit of activity.  He explains what he means by the general concept of an act! ivity cell, or unit, as "a certain kind of integrity ... which is always present in any activity."

9.  The first thesis that Davydov lists is about the collective nature of activity.  There is no individual activity that does not stem from collective activity, and collective activity is a process performed by a collective subject.  Davydov cites Bibler as having analyzed in detail the individual and collective subject along these lines.

10.  The second thesis is about the ubiquitous character of "appealing" in all human activity.  No one appeals to stones.  But people constantly appeal to one another, as Xavier quotes, "for true cooperation, for control over [their] ... individual actions, and for evaluation of those actions." 

11.  The third thesis is that "appealing" by individuals to others in collective activity presupposes an individual's awareness of the positions and potentialities of others, an awareness! Davydov refers to as the "ideal plane," following Ilyenkov's ! theory o f the ideal.  Appealing also presupposes an individual's reflections on himself or herself and their own actions.

12.  The fourth thesis is that in order to understand how individual activity stems from collective activity, a special study of "interiorization" is needed - a study of the transformation of collective activity into individual activity.  This task is a key interdisciplinary scientific task to fulfill, and goes well beyond just the discipline of psychology.  Davydov stresses that activity in general, as an object of research, is fundamentally interdisciplinary.
 
13.  Davydov also makes some interesting observations about the differences between animals and humans and the transition from one to the other.  He identifies the "psychic point" in the activity cell as precisely the transition from animals to humans.  The essence of the psyche, found in both human beings and animals, Davydov explains, is the attempt ! to identify what is possible.  Both humans and animals search and try, but the uniquely human way of trying is to appeal to others in the process, to fulfill tasks on the ideal plane.  [Comment 1: pets and domesticated animals learn to appeal, don't they? - is the socialization of animals the exception that proves this rule?  Comment 2: I believe Davydov's analysis is consistent with the Marxist labor theory of human origins].   Davydov goes on to speculate that animals may possess a rudimentary natural function of the psyche, one which humans certainly possess, in an interesting generalization:  the operations of animals (possibly) and the actions of humans (certainly) always undergo a transition from a "successive" mode of performance to a "simultaneous" one.  This law, at least concerning humans, has been studied by Lomov, Galperin, Podolsky, Zinchenko, and Brushlinsky.


This article is the best look I have had so far into Dav! ydov's creative thoughts, and I appreciate it especially for t! hat.&nbs p; Vasily Davydov left us many interesting ideas to consider, which we will have to put together and critique ourselves since he is no longer with us to help us search for "new and original" approaches.  That he was so boldly and honestly looking for them is enough to get my deep respect.

- Steve Gabosch

From: "Peter Moxhay" (moxhap@portlandschools.org)

I have a problem reading this Davydov paper because of the translator's use of the term "need" for the Russian word "potrebnost'" and the term "desire" for the Russian word "nuzhda."

Normally, I would translate;

potrebnost' -> want, demand
nuzhda -> need

For example, elsewhere Davydov write about how we have an organic "nuzhda" (need??) for oxygen, but this gets transformed into a "potrebnost'" (want??) when we are in a stuffy room. As a result, we become conscious of our need -- it becomes a want -- and we go to the window and open it.

It seems like this point is worth understanding, since it seems to be one of the focuses of Davydov's attempts to reinterpret the structure of activity. That behind "wants" there lie "needs." It doesn't make sense to say that behind "needs" there lie "desires."

I may be wrong, of course! But I would be interested if other who know Russian (Mike, Natalia, others) could comment on this point.

Thanks,

Peter Moxhay

From: "judy brown" (mmjbrown@connect.carleton.ca)

While I would be very interested in the answer to Peter's question about translation issues above, Davydov's article still has meaning to me because it shows that there are many perspectives on the important attributes ,or should I say structure, of group activity. For me, at least, it is easy to be convinced by one writer, and it is very valuable for me to have a glimpse at other Russian perspectives on this topic.

Leontiev, I believe, identified group activity as a central concept in psychology. For him, the main attributes of activity are the object, which can direct activity (and which is also linked to human need), but also important to him are the motives, form, methods, emotional intensity, time requirements, space requirements, and the physiological mechanism of the activity system. To me it seems he successfully integrated perspectives that were both individualistic and social.

For Engestrom, the structure of activity is quite different: subject, tools, objects, rules, community and division of labour. I value Engestrom's work for the way it builds on Leontiev's work and the way it can organize my understanding of the workplace.

Davydov, it appears, distinguishes between structure and cell of activity. The structure of activity, is desires, needs, emotions, tasks, actions, action motives, means used for actions, and planes of cognition and will. Davydov, to me has added a whole new dimension by addressing matters of the heart.

One question I have (and there are many), is what is meant by structure? Does it mean essential attributes or characteristics or is it something different? Also, are their aspects that are non-structural? Any help here would be appreciated. I think knowing what structure meant would help to evaluate these different views on activity.

From: "Robert Lecusay" (rlecusay@ucsd.edu)

Hi all,

I am writing with some comments and questions that stem from Davydov’s discussion (pp.47-48) of the relationship between the “ideal plane of man” and the structure of activity.

In this section Davydov seems to be putting the capacity for (and the drive, desire for?) intersubjectivty at the center of the structure of activity. He begins by presenting Ilyenkov’s definition of the “ideal in man” (which Davydov renames “the ideal in plane in the individual”): “an individual’s awareness of all other’s positions and potentialities.” He goes on to note that Ilyenkov “in his writings stated directly that imagination is the only thing that enables a person to conceive others’ positions and potentialities. It is imagination that enables a person to look at himself through other people’s eyes.” He then goes on to note that for Ilyenkov imagination is consciousness. Davydov then goes on to qualify his definition of the cell of activity as “conscious activity . . . there can be no other.” Am I making a simplistic leap here in connecting consciousness and conscious activity?

If I’m reading Davydov correctly then it seems we should keep an eye on the role of intersubjectivity in activity when we engage in our upcoming discussion on “interobjectivity” and distributed cognition. When I took the distributed cognition course last quarter there was a moment in the seminar when the idea arose that all artifacts in a joint activity have the potential to be recruited by the subjects engaged in the activity as signs for developing an interpretation of the mental states of others involved. I think this ties into our previous discussion of the object of activity as always being a collective object. Doesn’t our understanding of an object’s meaning and sense initially derive in part from our understanding of what that object means to another person?

As a graduate student I’m constantly being reminded in many of my readings not to give primacy to one influencing factor (can’t think of a better term) or another when studying behavior; however, whenever I drop my guard I succumb to the idea that intersubjectivity drives an activity, that it leads from one reaction to the next. It’s the unreachable end that people in interaction move towards. Believing that there is shared understanding or the desire to have shared understanding seems to propel interaction. “The evidence that each participant has successful communication is the flow of joint activity itself,” (Hutchins, Cognition in the Wild, 237).

Thoughts? Corrections?

(I did have another somewhat related question: What is the relationship between Ilyenkov’s “ideal plane of man” and ideology in Voloshinov? I’m thinking of the questions raised in the Steve-Ana-Anna (Hi Anna!) thread concerning the interpretations of tester intentions made by subjects in cross-cultural research; however, at the moment I can’t remember what else I wanted to say about this . . .)

-Robert

 

 

 

 

 

 

 

 

 

 

 

 

 

 

 

 

 

 

 

 

 

 

 

Week 4 Readings

 

Interobjectivity, Ideality and Dialectics
Yrjö Engestrom


by Joanne Price



In this paper Yrjö Engström critiques Bruno Latour’s paper entitled, “Interobjectivity,” presented in Mind, Culture, and Activity (3), 228-245.

Engeström points out that there are similarities and differences between Latour’s ideas and those of cultural-historical activity theory (CHAT). Engeström does this in four subsections:

1. Interobjectivity Embraced

Latour argues that human interaction, unlike simian interaction, uses material frames, including even walls and clothes. Engström notes that activity theory typically refers to artifacts as tools [and signs], but they are not as inclusive as Latour’s material frames viewpoint. CHAT would consider some of these frames as infrastructure, and not attached to specific purposes and uses.

Consideration of Latour’s ideas “force us to rethink our habitual tendency to freeze and categorize objects into one fixed role and place in the flow of activity.” Engeström then provides an example of how ‘wall’ moves over time within the activity system: wall is first an object to be created; then it is a product created; then it is a mediating artifact (tool used in rearranging his living space); then it is no longer a tool but part of the community infrastructure; with respect to the family, it is part of the division of labor in the family; it is part of the rule structure for the family (e.g. children are not allowed to play in this room).

Activity theory uses the concepts of internalization and externalization. Latour speaks about the movement of objects in terms of localization and globalization. Engeström notes that localization and globalization constructs provide additional perspectives from which to analyze the development and success or failure of an activity system.

2. Action and Activity, Object and Tool

Latour notes that something (some ‘operator’) must weave together ‘the properties of the object with those of the social.’ Neither Latour, nor activity theorists, believe that individuals (subjects) act on an object independently from mediation by the social world. In fact, each (the individual and the social world) is shaped by the other. And in a statement reminiscent of discussions of distributed learning, the whole is greater than the sum of its parts: “Collective activity is realized through individual actions, but it is not reducible to a sum total of those actions.” (p.262)



3. Object and Commodity

Latour and activity theory agree that objects are also commodities whose values are defined by the social world.

4. Dialectics and Contradictions

“Latour is a determined critic of dialectics.” Engeström finds this amusing as he notes that Latour’s analyses follow a dialectical approach.



Reflections on the Reading

One focus of this piece was on the mutually constitutive influence between individual actions and the social world. This discussion brought three concepts to mind for me: (1) automaticity; (2) scripts; and (3) cultural capital.

Automaticity
Gregory Bateson’s illustration of the man and the cane succinctly illustrates the concept I have of automaticity. Over time, with use, the cane becomes an unconscious extension of the body. Through automaticity, manipulation of the tool requires almost no conscious consideration. The tool has thus been internalized and is more a part of the body than of the external world. (Disruption in the normal routines with the cane causes a rupture in automaticity and jolts the cane back into the external world for a time until the disruption has either passed or has become a new part of the routine.)

The idea of automaticity pertains to the individual subsuming parts of the external world. Scripts and stories and cultural capital pertain to ‘relatively invisible’ influences on the individual from the social world.

Scripts and Stories
Roger Schank began his career as a researcher of artificial intelligence at Yale University in the 1970s and 1980s. In Schank's view on the other hand, we accomplish this because we have access to a stored 'schema' based on previous experience of what it is like to walk to the store, and we don't need rules to describe this.

In Roger Schank's first book, Dynamic Memory (1982), he described how computers could learn based upon what was known about how people learn. After further work in the field, Schank changed his focus from artificial intelligence to human intelligence. In a second book, Dynamic Memory Revisited (1999), Schank describes the role of scripts and stories in how people learn. Dr. Schank was one of the foundational theorists pertaining to case-based reasoning and the role of social scripts and stories in the learning process.

Since many of these scripts are formed during a child’s early years, in interaction with family and friends, they are often invisible to the individual, although very influential to the formation of cognitive constructs, expectations, and social interaction.

Cultural Capital
The idea of cultural capital by Pierre Boudieu seems to be another of these ‘invisible’ influences on the individual and actions within any given activity system.

Investigations into both personal and social community contexts helps shed light on actions within the activity system. This is the importance of historicity emphasized by cultural-historical activity theory.

Latour “On Interobjectivity”

 

From: nathaniel smith

Latour is concerned in this paper with the question of what social behavior in primates can tell us about building an appropriate theory of sociology. The key claim he makes about primates is that firstly, they live pervasively social lives, and secondly, that this sociality is different in key respects from that studied by sociologists looking at Homo.

The key differences are that in (e.g.) baboon groups, social relations are highly unstable, and must be constantly re-checked and re-created. In addition, all interactions take place in a single, mostly homogenous sphere -- all individuals are present (though not actively involved) in all interactions.

He will then use the former claim to argue that theories that reify social structure cannot be correct, and the latter to attack traditional interactionist theories.

Finally, to justify the application of this data to human sociology, he argues that since social life must have been continuously present since before Homo speciated, we should take this seriously, and expect that this was as much a shaping factor in evolution as anything else.

Latour – middle

From: "xavier cagigas" (xcagigas@ucsd.edu)

 

Must sociology remain without an object?

 

Latour begins by contrasting how chimps need to interact directly with one another (proximity in space and time) to maintain social life whereas humans can go beyond the immediate surroundings purportedly through the use of symbols.  Latour, however, asks:  then what do symbols hold onto?  “…how could the brain alone stabilize that which bodies cannot?” [8]  The key word here is “alone” à the brain in coordination with artifacts in the world seems to be where he is headed (again, Luria’s functiona! l systems which includes the extracortical)

 

“In order to get from a complex social life to a complicated one, we need to be able to timeshift, dislocate, make lopsided and delegate the present interaction so as to make it rest provisionally on something else, while waiting to take it up again.”

 

Here Latour points out the flaws in relying solely on social interaction (as in chimps) and also in relying on a strictly disembodied symbolic explanation.  The symbol processing he seems to invoke deals with immaterial representations; however, he hints at the dual nature of artifacts which he will introduce with the following:

 

“When [symbols] are sufficiently sustained, when cognitive capacities are sufficiently instrumentalized, heavy enough - then it will be possible to provisionally attach meaning to them; but not before.”  [9]

 

From here, Latour turns to objects as artifacts embedded in social activities as the solution.  However, he repeatedly points to the error of focusing on the objects themselves and refocuses on the space between them – mediation.  Objects create and are created, they form part of an all encompassing web of the social which is always present.

 

One cannot point to social forces or human agency, to causal relations, to history, culture, or anything else because they all share a recursive relationship and co-construct one another.  The focus is, therefore, on interobjectivity [inter-object-activity?]

 

“Not action, nor the actor, nor interaction, nor the individual, nor the symbol, nor the system, nor society, nor their numerous combinations can be redeployed.”  [12]

 

“The new cognitive capacities owe their extension less to the powers of symbols than to those of the instruments that hold them.”  [12]

 

Xavier’s take:  It seems that what sets humans apart is there ability to coordinate their activity with artifacts in the real world.  Put a different way, the "dual" nature of artifacts is crucial -- the material aspect seems to be covered by Latour and so is the immaterial in terms of the cultural-historical, but what of the brain as mediating this duality and in a sense unifying it?

Latour – Final Section: From the study of the soul of society to that of its body

 

From: "noga shemer" (nnsevilla@yahoo.com)

 

Latour argues that sociology, in spite of its “will to be materialist,” created a new form of spiritualism in which only the “soul” of the social body is considered.  Humans are seen as monkeys surrounded by material things -- things which are taken to be mere intermediaries instead of mediators.  As a result, sociology lost the practical means to understand localization and globalization – creating the apparent abyss between social structure and individual interaction.  Latour advocates a study of the “body” of society instead, in which we need: “a) to treat things as social facts; b) to replace the two symmetrical illusions of interaction and society with an exchange of properties between human and non-human actants; and c) to empirically follow the work of localizing and globalizing” (15).

 

Key points:

 

-- Human interaction is localized by a frame of non-human actors. (12)

Latour uses the example of the post office interaction again, tracing the physical structuring of the space, the rules for the interaction, etc. not to “society” but to the architects who anticipated that moment in their category of user, which he actualizes with his own body.  Then, moving in another direction, he traces the frame to those who constructed the actual building which mediates his interaction with the postal worker.  This demonstrates that incorporating objects involves circulating in time, space, and across levels of materialization (13).

 

-- “Any time an interaction has temporal and spatial extension, it is because one has shared it with non-humans.”  Therefore, “‘Inter’-action… signifies that action must be shared with other kinds of actants in other spatio-temporal frameworks” (13). 

This ability to endure beyond the present is difficult for non-human primates.  Here, Latour uses the example of a shepherd delegating a task to a fence – a view which looks to the fence as an actant in its own right.

 

-- Aggregates of interaction must also be viewed as framed by other actants dispersed in time and space (14).

Latour uses the example of “Parisian life” to reconsider the means of constructing the social world.  The accumulation of statistical ‘I’s do not create a social structure above the individual actor.  The control rooms, panoptica, formulae, etc. are themselves localized and blind, but together, they can explain structuration effects.

 

-- “If you set yourself the task of following practices, objects and instruments, you never again cross that abrupt threshold that should appear, according to earlier theory, between the level of ‘face to face’ interaction and that of the social structure….  The work of localization, like that of globalization, is always carried out by bodies in times and places far apart from others” (14). 

 

From: Johannes Knigge

I find that the theory of actants as important interactive elements have helped me understand and explain how a certain setting can influence the possible actions  in a given situation.

The fact that artifacts and outer circumstances influence our actions and possible actions seems self-explanatory, but are often omitted from theories of social interaction.

I also find, that being able to shed light upon the actant-networks "within" a person - thus seeing the factors / actants influencing the decision- and action-making is most important.

When we are further able to share some of our actant-networks with other individuals through learning and interaction, I find the actant-theory very potent. If my peers learn from my actions in a given setting, they too acquire some of the basis / actants influencing my possible choices - and thus they achieve a future reference to the given network of actants, giving them new possibilities of action in the future.

An actant can be both a new theoretical approach, or the knowledge of alternative possible actions.

The fact that actants in this way can mediate personal growth is most rewarding - and the theory holds great explanatory power when talking to lay-men in the social sciences.

Greetings from Denmark

Johannes Knigge

From ?

Latour is concerned in this paper with the question of what social behavior in primates can tell us about building an appropriate theory of sociology. The key claim he makes about primates is that firstly, they live pervasively social lives, and secondly, that this sociality is different in key respects from that studied by sociologists looking at Homo.

The key differences are that in (e.g.) baboon groups, social relations are highly unstable, and must be constantly re-checked and re-created. In addition, all interactions take place in a single, mostly homogenous sphere -- all individuals are present (though not actively involved) in all interactions.

He will then use the former claim to argue that theories that reify social structure cannot be correct, and the latter to attack traditional interactionist theories.

Finally, to justify the application of this data to human sociology, he argues that since social life must have been continuously present since before Homo speciated, we should take this seriously, and expect that this was as much a shaping factor in evolution as anything else.

From: lars rossen

This might (though coing being pastedin from a different context) serve as somewhat of a commentary on the ongoing discussion - it's an elaboration on Latour, regarding a discussion on the agency of artifacts: My thought was that the purpose that gets build into a given artifact obviously gives it a certain intended agency - a specifically intended goal and ways to obtain it by it's use - which changes the way of interactions in society and again the agency that the user meets the artifact with re-shapes it due to the various other yet similar tasks that artifacts is applied to in the "natural" setting. The agent might use it in unintended ways that will backfire towards the designer and change the agency of the system that produced the artifact - so the development of artifacts in an industrialized society has the dimension of shaping according to an imagined demand and use that might or might not hit the spot - or even hit more spots, opening up for refinements. So the ! fact that we have engineers to do the dirty design work adds an extra space of artifact mediation in the form of the agency build into the damn things we use , which we can peek into: the multidirectional agency of the things that gets directed toweards towrads us and we direct towards the things. Long story short: Latours idea does not seem that far fetched on the particular case... or, as far as I'm concerned - most of his other ones :-)

Lars

From: "Steve Gabosch" (sgabosch@comcast.net)

 

Interesting points, Lars.  The idea of building agency into artifacts is sometimes formulated as “stored labor.”  The computer age – with its engineers and everyone else - has certainly expanded this process of building agency into artifacts to amazing levels.  I agree with the point you make following Latour that when we are mediated by artifacts (especially, but certainly not just, artifacts that are “programmable”), along with all the other properties these artifacts contain, we can be mediated by a kind of “built-in agency” or “stored labor” contained in these objects.

 

As an NC mill operator, I am constantly confronted by the work of programmers who control the motions of the machines I run.  Some of these motions – “built-in agencies” - are not quite right and create problems, and operators such as myself may have to create “work arounds” to accomplish the desired outcomes.  If the program creates too many such problems, we “squawk” (critique) the “tape” (the program, which once upon a time was stored on a punched tape) and get the programmers to write a new and hopefully improved program.  A wide range of “agencies,” positive and negative, get built into these artifacts of stored labor.

 

So Lars, returning to the underlying issue in this thread, what is your take on this question of the origin and genesis of the “ideal half” of an artifact?  Does this idea of “built-in agency” shed light on the more specific question “do brain processes or social relations create ideality?”

 

-       Steve

-        

From: "Donna Russell" (russelldl@umkc.edu)

>David Shaffer has written an article that will appear in MCA
>at some point that bears on our discussion of Latour and AT.
>You might be interested in checking it out.
>
>http://coweb.wcer.wisc.edu/cv/papers/toolforthoughts-sub1.pd
>f

this article defines many of the issues that i have been considering as i design research of online learning environments- is understanding the mediationality of the tools  as important as understanding the potentiality of the learner - and how does this impact design of SC research and design of tech-mediated learning environments?

 

From: "Mike Cole" (mcole@ucsd.edu)

Donna-- How could/should anyone want to separate properties of person and properties of mediational tools, or either of those from consideration of the activity?

If you are interested, a grad student and I, independently, have written a paper on the same general problematic, but we are less enthused about Latour's symmetry assumptions.
mike

 

Hutchins: How a Cockpit Remembers its Speed

 

From: "emma johnson" (ekjohnso@ucsd.edu)

 

How a Cockpit Remembers Its Speeds" by Edwin Hutchins

In this article, Hutchins applies the cognitive science approach to a unit of analysis broader than the individual person. This methodology has the benefit of including things that can be directly observed (unlike much individual cognition). The cognitive properties of systems are not simply reducible to the sum of all the individual processes. Hutchins studies concrete situations from actual observations, which he argues are the only way to successfully study cognitive systems. In this summary, instead of explaining all of the details (which Hutchins does well) I am trying to synthesize the main points.

Hutchins studies the cockpit system of commercial airlines. In the cockpit, there are two pilots who, in order to safely land the plane, need to calculate and remember a set of corresponding airspeeds and wing configurations. Pages 3-10 are pretty much devoted to describing in general what goes on and why, and I will only describe it briefly here. Hutchins makes clear both the time In order to land the plane safely, the pilots must decrease the speed of the plane as they approach, but in order to maintain lift of the plane as speeds decrease, they must change the shape of the wings using moveable flaps and slats. Hutchins describes a set of things that take place: calculating the correct speeds based on the weight of the airplane, then setting physical markers, called "speed bugs" on the instruments displaying the airplane’s speed. These bugs are placed on the dial of the instrument in order to correspond with the values on the speed card that the pilots have already selecte! d based on weight. There are also a series of verbal exchanges between the pilots during the landing, in which the pilot who is not actually flying reads off information—and warnings if the airplane deviates too much from the safety zone.

Hutchins then explains why these speed cards and speed bugs and the verbal exchanges are not merely memory aids. For example, on page 17 he writes "Speed bugs do no help pilots remember speeds, rather they are part of the process by which the cockpit system remembers speeds."

There are a couple of key points that Hutchins makes about the way these artifacts and processes function in the cockpit system. Perhaps most obviously, they change the temporal character of the tasks that must be completed for a safe landing. Calculation and crosschecking take place in the relatively calm period before the airplane begins to descend for landing, when there are not many demands made on the two pilots. Displaying the selected speed card also allows for crosschecking of this calculation at any point by comparing it to the weight displays.

The speed bugs allow conceptual (computational) tasks to be implemented by visual perception. By representing the speeds visually on the dial, the pilots can look at the needle position in relation to the markers, and instantly know whether they are within—or close to—the danger zone. Also, by what appears to be a coincidence, the width of the marker itself in relation to the speed dial corresponds to the critical 5-knots on either side of the target speed, which further facilitates this transformation from a calculation into judging spatial proximity.

And Hutchins points to the way that the formalized verbal exchanges during the critical landing period allow the pilot to employ a different perceptual system by transforming visual information into aural cues. So, this distributes tasks socially, spatially, and across perceptual systems.

Hutchins points to the way that this system includes a lot of redundancy, which makes sense considering the literal life-and-death nature of the decisions being made. By distributing the tasks, including cognitive, perceptual and memory tasks between people and things, the demands made on the individuals are more easily managed. Also, the tactic of employing a series of artifacts and media, makes the cockpit "memory" more robust over time.

 

 

 

 

 

 

 

 

 

 

 

 

 

 

 

 

 

 

 

 

 

 

 

 

 

 

 

 

 

 

 

 

 

 

 

 

 

 

 

 

 

 

Week 5: Links to Pragmatism and Symbolic interactionism

 

(i.e. Activity Theory?)

 

Synopsis for Miettinen's "Artifact Mediation in Dewey and in Activity Theory" (2001)

 

From: "David Mather" (dmather@ucsd.edu)

In discussing a Garrison article that contrasts Dewey and other theories of activity, Miettinen outlines a number of comparisons among these theories, as well as activity-network theory. Regarding dualism and monism in theories of interactionism (activity theory), Miettinen shows that Vygotsky and Leont'ev have a nondualistic (viz. non-Cartesian) approach to the subject and the object. The concept of internalization is a specific case in which internal activity bears a close relation to external activity (and tool use) and in which externalization (as artifacts) reflects internal dynamics that include social relations.

Miettinen develops the point that Marx's category of praxis can be found within Dewey's tripartite structure of functional coordination and also within activity theory's tripartite structure of mediation (among subject, object, and artifacts). According to Miettinen, Marx says "all social relations are mediated and produced through natural things" (300), but that "nature cannot be reduced to the historical modes of its appropriation in practice" (301). This complex character of nature arises as its resistance to cultural and social use. Resistance, a concept that arises in various theories, is one result of human activity that manifests physically: "Activity theory, Dewey, and actor-network theory all agree that artifacts carry intentions and norms of cognition and form a part of the agency of the activity, expressing their agency also as resistance" (301) Also, for Dewey, Latour, and Lektorsky, artifacts are objects with agency: "Ways of doing and properties of things are o! bjectified in tools and cultural artifacts" and "culture created by humanity constitutes an external, objective medium of and basis for the mental activity of the individual" (302).

Dewey's theory of cultural emergence of meaning, mind, and self is strikingly similar to activity theory's discussion of mediation (303). Then, Miettinen compares how each handles the concept of motive in relation to activity. Since motives can be seen as arising from socio-cultural systems, the individual habits illustrate distributed agency, and, likewise, disturbances of habits "are increasingly seen as an important source for change and development" within individuals and within culture (305).

Miettinen connects Dewey's transactional philosophy with practical research by showing how each tries to use a nondualistic analysis of social relations. Activity theorists model activity as a culturally and socially mediated activity system, which relates to both conjoint activity (Dewey) and the collective (Latour). Dewey suggests a practical transformation of the world (in _Democracy and Education_), which is a correspondence between pragmatism and activity theory to be further developed.

From: "Steve Gabosch" (sgabosch@comcast.net)

 

Below are the five paragraphs of a section in Meittinen’s article Artifact Mediation.  I like Reijo’s writing and clarity of thought – he reminds me of Keith Sawyer that way – but especially paragraphs two and three in this section have some ideas that puzzle me.  I have bolded and used colors to emphasize the parts I am trying to understand.

 

1.  In paragraph 2 Reijo says: “We are interacting with two kinds of material things: the means and objects of activity. That is why practical activity has a tripartite structure, envisioned by Vygotsky (1978) …”

 

Can we really simplify all material things to “two kinds” this way?  I would find it intriguing if we could.  And if we can, how does this relate to the tripartite structure:[stimulus --- artifacts/tools/signs ----response] envisioned by Vygotsky?

 

 

2.  In paragraph 3 Reijo says: “Under pre-industrial conditions, the natural moment is dominant. Nature (fruitful soils, waters teeming with fish) is primarily a means of subsistence, existing naturally and independently of man. In industrial society, the moment of subjective intervention asserts itself in increasing measure over the material provided by nature.”

 

What does he mean by “industrial society” and “pre-industrial conditions”?  What stages in human development is he referring to?  Soils imply agriculture, and fish imply barbed hooks, both relatively recent innovations (barbed hooks go back to maybe 75,000 BC – although that of course is always subject to change - and of course, there are nets, etc. – and agriculture, maybe 11,000 BC, again, give or take).  Such tools and associated planning processes – my point in bringing up the technical aspects - imply uniquely human higher mental processes requiring considerable social and cultural mechanisms for them to be possible.  How is the “natural moment” “dominant” in principle in these two examples?  Surely, modern industrial society greatly increases the dominance of humans over nature, but hasn’t that process in principle been increasing since humankind first began to use tools to socially produce its means of life?  How do the principles of activity theory – externalization/internalization, extrinsic/intrinsic stimuli, materiality/ideality, natural/social, praxis/passiveness, etc. etc. not apply to all humans at all stages of social development?  Isn’t the “social moment” ultimately dominant in all human activity?

 

3. Paragraph 4 uses a number of terms I find puzzling, which I highlighted, but they lead back to the above questions.

 

The paragraphs in question follow for reference.

 

- Steve

 

_______________________________

from ARTIFACT MEDIATION page 299

 

 SUBJECTS AND OBJECTS AS PRODUCTS OF ACTIVITY

 

Garrison (this issue) started his critique of dualism by stating—citing Dewey—that behind the Cartesian dualism of subject and object, psychic and material, there is a deeper dualism, that of the dualism of inner and outer. Although this reformulation makes it possible to discuss directly the language used by Vygotsky and Leont’ev in their the psychological texts, I can’t see any significant difference between these formulations of dualism. In most of his philosophical work, such as Experience and Nature (1925/1988), Dewey dealt with the Cartesian dualism between the subject and the object, between mind and the matter. Philosophical works on the foundations of activity theory discuss as well the dialectics between the subject and the object (Ilyenkov, 1977; Lektorsky, 1980; for a contemporary Western interpretation, see also Bakhurst, 1991). Therefore, I speak in this article mostly of subjec! t–object dualism.

 

 1About the shift from the paradigm of internalization to the paradigm of externalization or artifact creation, see Engeström (1999, pp. 26–27).

 

 

ARTIFACT MEDIATION page 300

 

The dialectical materialist tradition is not dualist in any Cartesian sense; that is, it does not postulate two distinctive and opposing realms of reality. To substantiate this, I briefly characterize the idea of activity or praxis in Marx’s work. He formulated the category of praxis (transformation of nature, artifact creation) to explain the simultaneous emergence and transformation of the subject and the object (Marx, 1964; Marx & Engels, 1968). The human being (humankind) creates him or herself by changing nature and by producing the world of cultural objects. This leads to a particular conception of the nature of the environment, something that activity theory shares with Dewey. We are interacting with two kinds of material things: the means and objects of activity. That is why practical activity has a tripartite structure, envisioned by Vygotsky (1978), and, as Garrison (this issue) told us, also by Dewey, in a three-term structure of functional coordination.

 

As Schmidt (1971) noted, Marx conceived that the unity of the subjective and objective is realized in changing configurations. He analyzed this philosophical problem in historical-economic terms (Schmidt, 1971, pp. 121–122). Under pre-industrial conditions, the natural moment is dominant. Nature (fruitful soils, waters teeming with fish) is primarily a means of subsistence, existing naturally and independently of man. In industrial society, the moment of subjective intervention asserts itself in increasing measure over the material provided by nature. Nature ceases to be a natural entity, independent of man. Labor appears to be ever more clearly a factor of nature. Marx expressed, according to Schmidt, the epistemological signif! icance of this historical development in the first thesis on Feurbach.

 

Marx could not accept Feuerbach’s materialist conception of nature as a homogeneous material substratum. His solution was to dissolve this homogeneity into a dialectics of the subject and the object (Marx&Engels, 1968). In the Theses on Feuerbach, Marx called attention to the ideal, the subjective aspect of reality, namely human, creative, transformative practice. Nature is something already transformed by humans and hence understandable only through human mediation. Pre-human nature is no longer accessible. Natural history is indivisible from human history, which is characterized by transformation of nature by human work, the creation of objects by humans. Marx re! garded it necessary to retain the non-identity of subject and object within their unity.

 

Garrison (this issue) stated that the inter-actionist theories of Vygotsky and Leont’ev first postulate two opposing entities, subject (agency) and environment (things, nature), and “then struggle with the problem of putting them back together via activity.” This is not the case. Following Marx, activity theory starts from the unity of nature. In studying the emergence of the human consciousness, it also studies the historical emergence of the subject-object distinction, based on practice that transforms nature and produces cultural artifacts. This activity is the basis of monism: both the subject and the object owe their possibility to activity through which their inescapable unity is constantly reproduced.

 

<end>

 

From: "Robert Lecusay" (rlecusay@ucsd.edu)

In the first half of Miettinen’s piece he outlines three problems that stem from the principle of generalized symmetry (and semiotic symmetrical vocabulary) in Actor Network Theory (ANT). The first problem is that with symmetry theoretically relevant elements of a network are not presupposed. The flipside of this is the goal of “complete explanation,” of producing a description that treats every single element in the activity as equally important. In either case the researcher is left without being able to discern what elements are important and essential for the intended study. The second problem is the problem of “silent actors” (i.e. non-Machiavellian actors and nonhuman elements). Miettinen notes a tendency in ANT accounts of innovative systems to give prominence to the role of “princes” – innovators, managers, politicians. This not only has the effect of marginalizing the work of engineers and user, but with the marginalization of these actors the work of the develo! pment of associations between the human and nonhuman elements in the system is ignored. The final problem Miettinen sees with ANT is its narrow theory of human competence, which is based on the idea of the Machiavellian prince: an actor that is skilled in negotiating and exercising power. Ignored are meditating cultural resources, theories of learning and expertise.

In this the second half of Miettinen’s piece he attempts to provide a solution to these three problems. He does this by presenting an activity theoretical analysis of an innovation network of three research communities organized as project for producing ethanol from wood or other cellulosic materials.

For addressing the problem of structuring the analysis of the network, Miettinen proposes that the unit of analysis for each node in the network be considered an object-oriented, culturally and socially mediated local activity system. The object in this case is the goal of producing ethanol from wood or other cellulosic materials. It is the construction of this object which acts as the organizing principle for the various research communities involved. However, the structure of the network of activities that are organized around this object is not static. A focus on the historically developing nature of activity systems provides insights into how the motives and interests that shape the activity are formed and transformed.

One of the ways in which Miettinen examines the influence of history is by looking at the work of the “silent actors,” in this case material elements like enzymes and waste materials. In the example of the ethanol project, the research groups involved ran into some difficulties that translated into higher costs for the production of the necessary materials. For example, although the enzymes produced were able to break down the waste materials, the waste materials showed a high resistance to the activity of the enzymes. This in itself did not mean that the interested corporations would decide against going ahead with the ethanol project. What did affect the trajectory of the process was the fact that at the time that the project was being conducted, oil prices began to fall. Relative to this historical circumstance, the feasibility (i.e. profitability) of going ahead with the ethanol project was significantly diminished, and the project was scrapped.

Miettinen addresses the problem of ANT’s narrow conceptualization of human competence by proposing an analysis that takes into account the role of tertiary artifacts (Wartofsky) in the functioning of an activity. As an example he discusses the different roles played by the model of ethanol production in the activity system. The model, “serves as a bridge between the past and the future in the activity,” (190). It simultaneously serves as a history of past research achievements, as well as a plan for the development of future industrial systems.

Miettinen continues by extending his critique of semiotic symmetrical vocabulary. By accepting symmetry, he argues, we ignore historically changing relations between science and technology; we ignore the relationship between differences in forms of thinking and their historical origins. Furthermore, a semiotic symmetrical vocabulary gives up, “vocabularies particular to different fields of science and technology,” which, Miettinen argues, are critical to an analysis that addresses how actors conceptualize the activity being studied. Miettinen, qualifies this statement by saying that these vocabularies must be used in “versatile mutual interaction with other voices and points of view,” in order to gain a “rich and credible account of the research subject.” A single privileged point of view (semiotic symmetrical vocabulary) is counter to the multivoiced approach that Miettinen is advocating.

Miettinen ends by elaborating on his proposal to develop a dialogic approach to the study of activity. He sites examples of similar approaches in other philosophical and research traditions (e.g. Manheim, Harding, Markova & Poppa), as well as the specific influence of Bakhtin in Activity Theory: “Multivoicedness is connected to the historical change of activities and social languages. The evolving, historically changing social languages are combined and used situationally by individuals connected to different activities.”192

From: "xavier cagigas" (xcagigas@ucsd.edu)

Regarding Steve's question about simplifying material things to "two kinds," I understood this to be referring to the dual nature of artifacts (i.e. ideal and material).  To follow Steve's reference to Vygotsky's tripartite structure sketch of [stimulus --- artifacts/tools/signs --- response], is this "two kinds" not referring to the middle part of this diagram?  I would also add that in order to highlight the dual nature of artifacts we should perhaps amend the diagram to explicitly include the processing of a material artifact by the brain (this represents the ideal half of the duality of artifacts).  The second that our brains engage a material artifact involved in a particular activity, the dual nature of the artifact comes into play.  In order to maintain a material monism that continually emphasizes the process and dynamic nature of activity, we must redefine the brain as an open system that cannot function without the fuel of cultur! al artifacts.  I know I personally harp on this, but I see Luria as the consummate activity theorist in insisting that our unit of analysis be the functional systems which includes both the cortical and extracortical aspects of activity since one truly cannot exist without the other.

I believe this also speaks to Steve's second question which seems to beg the question of whether the natural and the social can truly be separated in modern humans.  In principle, one would like to think that the natural has some precedence in the temporal scheme of things at least historically through our evolution; however, this brings us back to what originally enabled the social to influence the processing of the natural.  Put a different way, what allows for the external to become internal or the material to become ideal.  Again we see a bootstrapping type problem which involves the co-construction of the social world and the functional circuits of the brain.  Whereas at one time in our individual ontogenetic development we each experienced Jame's buzzing confusion at the sight of a red bulbous material with a green thin extension that hurt us if we touched it, we can no longer return to that state because now the second we see a rose we! recognize it as such and associate it with a sweet fragrance, romantic love, and a host of other culturally determined associations.  We can no longer separate the experience of what it once was from what it has been for generations.  It is now a rose and will always be a rose unless parts of our brain begin to degenerate which do not allow us to recognize it as such.

I was struck by Dewey's three-term structure of functional coordination which echoed the idea's of Luria's functional systems theory and also Hutchin's insistence on the centrality of coordination in distributed cognition.  What do others think about this?

From: "Steve Gabosch" (sgabosch@comcast.net)

I appreciate your willingness to engage in this dialogue, Xavier.  The main question we are wrestling with is: where and when does "ideality" or the "ideal half" of artifacts originate?

You bring up many rich ideas and pivot points.  Let me comment on some of them.

Interactivity
The central idea you stress is that in generating ideality, social and brain processes are interactive - they work together, and neither can do it alone.  You suggest that it may be counterproductive to imagine a dichotomy between the two by asking "which came first," a question you liken to the original "which came first, the chicken or egg" riddle, which you also refer to as a "bootstrapping" problem.  You argue instead that social processes and human brain processes generate ideality simultaneously, and therefore the key is to understand how they do so.  It is not social processes OR brain processes, it is social processes A! ND brain processes.

My response to this line of thinking is it is very good until we begin to ask what are the origins of social processes and brain processes.  What is good about the "interactive" approach is what is good about a successful photograph - it can reveal important relationships and characteristics.  But what is limited about the interactive perspective is it only reveals a snapshot of a process.  It offers a chronological cross-section without the overall developmental view.  It is true that human social and brain processes must interact for humans to experience cultural meaning, but how do these social and brain processes themselves emerge?  What is their genetic relationship?  What is the full chain of causes and effects that lead up to the dual nature of artifacts?  To answer these questions, I believe a developmental approach is necessary.


Interactivist vs Developmental Perspectives!
You make an interesting comment that the developmen! tal pers pective on activity is difficult to always maintain, especially when considering when and where activity begins.  You seem to be implying that the developmental perspective is not always possible.

My response is that it is indeed difficult, but absolutely necessary, and completely possible - with a lot of work by many people.  The interactive perspective is certainly a good place to start, and can be seen as a stage in the developmental approach.  Methodologically, we can think of an interactive analysis as taking snapshots, and a developmental analysis as stringing together enough interactive analyses to begin to get the larger developmental picture.  The "genetic-historical" method Vygotsky advocated can be seen as a developmental perspective that extends to the origins and full history of a process.


Phylogenetic Origins
You raise several interesting points about the phylogenetic or biological origins of! how brains process cultural meaning.  You take the position that certain parts of the human brain had to develop *before* cultural and historical development could be possible.  As an example, you explain that humans have a unique ability to partition space and time not found in other animals.  You raise the argument that if social processes were the only cause, then other social animals would also experience ideality and use tools and signs in a human way.  Another point you make, focusing on the relationship of the biological and the social, is that the neural circuits that underlie higher cortical functions are honed and completed by human social interaction, and these circuits form the basis of the ideal half of artifacts.

My response is to emphasize how all these points argue that human biology preceded the development of human social relations and processes.  I think it is very useful to look at the converse as being possible.!   I am influenced by the classical Marxist theory of the ! origins of humanness, which argues that human labor - the basis of human society, human activity, and the central way humanity interacts with nature - was not only the basis for the development of human social and cultural history, but was also the basis for the development of modern human biology.  Engels, for example, makes a strong case that labor was the basis for the evolution of the hand, language abilities, and other phylogenetic features of the human species.  In this perspective, the social precedes the biological.

Let me underscore this idea and finish this long post by responding to a question you ask.  I suspect you were asking this question rhetorically to emphasize how difficult the developmental perspective can be to maintain, but I think this question has a profound answer that opens the door to a consistently developmental approach.

You ask:  If our starting point is activity, then we also have to ask: where/when does activity begin! ?

I think the answer centers on human labor, on human production.  Human activity begins with human labor, with human production.  What is unique about humans above all else is our species depends on social labor and social production to survive.  Labor is the essential human activity, the fundamental human social process, the central reason for passing culture from one generation to the next, the basis for language, and the foundation of human brain processes.  According to this line of thinking, if we begin with labor, we can explain all the rest - phylogenetically, sociogenetically, psychogenetically, neurogenetically, etc.  I suggest that it is this essential theory - the labor origins of human development and activity - that lies at the heart of the cultural-historical psychology of Vygotsky, Luria, Leontiev, and many others.

You raise many other important points, Xavier, regarding ontogenesis, regarding ways that the dual na! ture of artifacts is distributed and coordinated between artif! acts and the human brain, and other intriguing points.  But I have gone on long enough.  Thank you again for this fruitful ... interaction!

 - Steve

From: "xavier cagigas" (xcagigas@ucsd.edu)

I apologize for not being able to respond sooner to the last two posts, but I had to leave town for a few days to attend to some family matters.

First off, thanks to Anna for adding to this discussion.  I think questions of agency and consciousness are definitely germane to the subject.  Anna, you refer to self-consciousness and self-reflection which of course are made possible through the complex neural feedback loops which allow the human person through their nervous system to interact with their environment (both external and within the human body as a whole) and with itself (other neuronal modules).  I agree that consciousness is only possible in relation to other people or other artifacts we interact with; however, all of this is contingent upon a healthy nervous system as testified to by countless case studies with neurologically damaged patients.  Brain activity is necessary for, but not all inclusive of, a theory of activity as a whole.  I think the insistence on self-reflection and agency you cite brings us back to including the working brain as an integral component of our a! nalysis of activity.  You quote:  "The information gathered by these mechanisms only becomes a fact of cognition when it is assimilated by man."  My questions is what assimilates the gathered information and makes it a fact of cognition?  -- yes, I guess it is rhetorical isn't it :-)  Hopefully other people will chime in with their own thoughts as well. 

With regard to Steve's last post:  I agree that a developmental approach is imperative and it is precisely for this reason that I have a problem with squarely positioning the genesis of higher order cognition strictly within the domain of the social.  I may be wrong here, but I really do believe that it is more about a back and forth co-construction.  Social relations and labor itself also develop over time.  Perhaps it is my own novice understanding of social activity theory, but the way that labor is usually defined presupposes a lot of conceptual assumptions that engage various brain systems.  For example, from the very basic recognition of objects and their properties, to how they can be manipulated in space and time by a human agent, to the "purpose" for engaging them to begin with, to issues of motive, planning, execution, and evaluation...  I don't think we can start with a sophisticated notion of "labor" and perhaps need more of a gra! diental approach.  The act of "laboring" intimately requires the healthy functioning of the nervous system which forms the basis for my advocating for this component of the functional system to be given equal attention and bearing on an analysis of activity.

I agree that even the simplest forms of labor lead to changes at the neurophysiological level and reorganize the functional system which then itself allows for more complex forms of labor to be possible.  My point before in stressing the difficulty in maintaining a developmental perspective is precisely that starting with labor is itself cross-sectional since a lot of neurophysiological stuff needs to happen even if we are talking about gathering a bunch of rocks into a pile.  Even if taking a decentralized approach a la Hutchins to the genesis of labor, I think that it is still difficult if not impossible to place "labor" before all else.  When activity theorists speak of labor, are there differences in degree?  What would constitute a unit of analysis for labor when looking at the genesis of higher cognitive functions?  I wonder what the more basic psychological functions are that are necessary for "labor" to be possible which in turn leads to hig! her cognitive functions?

It seems to me that viewed on a developmental scale, the social and the biological take the lead in constructing each other at different points in time.  Neither the biological nor the social is a static entity which is acted upon by the other.  Rather, when examining the functional systems that subserve higher order cognition, it seems that reorganization in one plane leads to reorganization in the other making new things possible that were not possible in the previous configuration.  It seems to me that this is what Luria was speaking of with regard to the hierarchical organization of functional systems in the nervous system.  I would be interested to learn if a similar hierarchical system exists in other analyses of activity--of which labor is obviously a quintessential example.  In other words, do more basic forms of labor lead to more complex ones?  And if so, if there is a developmental progression of labor itself, then I think the boot! strapping problem becomes clearer...

Steve, you state that "Labor is the essential human activity, the fundamental human social process, the central reason for passing culture from one generation to the next, the basis for language, and the foundation of human brain processes.  According to this line of thinking, if we begin with labor, we can explain all the rest - phylogenetically, sociogenetically, psychogenetically, ontogenetically, neurogenetically, etc."  I would ask whether "labor" is truly the best unit of analysis since activity theory seems to extend beyond Marxist philosophy.  In other words, what of Davydov's wanting to incorporate issues of desire and emotional valence.  There are human activities that are equally universal but that are not "labor" or focused on "production" in a strictly Marxist sense.  If we bring desire into the picture, if we speak of motives and human agency, then we need to incorporate what we know about these things on a neuropsychologic! al level.  I don't think that "labor" is truly an indivisible unit of analysis that is at the foundation of a string of causal events.  I think labor like all other things has a developmental history, and that this history is shared among both the social and biological planes.  It is important to note that when I say biological, I am really refering to physiology in that I am speaking of the functional organization and reorganization of brain systems over time.

I guess I view this in a much less linear fashion.  It may be the perspective and discipline that I am coming from, but I tend to view things as having multiple contributing causes.  I see development as going forth on multiple hierarchical levels simultaneously and not in a simple linear progression.  Everyone seems to bash Cartesian dualism these days, and I guess I'm doing it again today.  I don't believe in "interactionism."  I think that labor has its basis in the material, but I think that this material is found both in historical artifacts and in the working brain which has its own history.   I think the genetic analysis proposed by Vygotsky and his followers naturally had to stress that the social precedes the biological in order to swing the pendulum; however, I'm not so sure that this wasn't simply a devise to move us toward a more true synthesis of the two.

I believe I have gone on long enough for today.  Steve, I'd love to hear your response to all this.  Also, it would be great to hear other perspectives on this as well.  I think this is a sticky point that cuts to the bone of what activity theory is and what should be included in terms of its methodology and epistemology...

Xavier

From: "Steve Gabosch" (sgabosch@comcast.net)

 

Hi Anna,

This question of self-consciousness that you raise is very interesting.  Thank you for bringing it up, and thanks to Xavier for his thoughts on this, too.  What is self-consciousness?  Clearly, an artificial mechanism cannot have self-consciousness, except in science fiction.  And periscopes don’t see, only periscope users do.  I am comfortable with those conclusions.  But I am not so sure about some other points about this general category “self-consciousness.”  This is not one I have thought through very thoroughly.  So here are some of the question! s that pop up for me from your stimulating post, Anna.  One, from something you said – why do you say that only humans can be conscious of themselves?  I (sketchilly) recall reading there is recent evidence that some monkeys and perhaps other animals can look at themselves in mirrors and know they are looking at themselves.  But even without that, what is it about humans that makes their “self-consciousness” specifically human and different from that of animals?  Another set of questions, from the Lektorsky quote you offer: “Man’s perception of the external world presupposes an elementary act of self-consciousness …”  What might these “elementary acts of self-consciousness” themselves presuppose?  In other words, w! hat conditions are necessary to create human self-consciousnes! s?  When do human infants first become “self-conscious”?  What generates that development?  And how do these issues relate to my beginning to feel, as I write this post, a little “self-conscious” about asking all these questions?  :-))

-       Steve

 

From: "Steve Gabosch" (sgabosch@comcast.net)

Great post today, Xavier.  You bring up some new points we actually agree on and give some of the things I said some worthy challenges.  Have thinking cap engaged ...

- Steve

From: "Mike Cole" (mcole@ucsd.edu)

Reijo Mittinen has responded to the discussion as follows:

 

(Recall:  From Steve’s original post:  1.  In paragraph 2 Reijo says: “We are interacting with two kinds of material things: the means and objects of activity. That is why practical activity has a tripartite structure, envisioned by Vygotsky (1978) …”)

 

Sorry for the late response to Steven's good questions

As far the first question, I really mean that from the point of view of the mediated structure  of action, artifacts ( or their functional role  in the structure of activity)  can be characterized as means or objects.
(So I was not referring to double nature of artifacts here), This does not imply that material things are of two kinds in any other sense, or that it makes sense to regard all material artifacts using this
distinction.   The formulation of the following sentence, "that is why practical activity has a tripartite structure"   might be a little strong and careless.

 

(Me; From Steve’s original post: 2.  In paragraph 3 Reijo says: “Under pre-industrial conditions, the natural moment is dominant. Nature (fruitful soils, waters teeming with fish) is primarily a means of subsistence, existing naturally and independently of man. In industrial society, the moment of subjective intervention asserts itself in increasing measure over the material provided by nature.”)

 

 

The other part of Steven's question is the most pertinent. It is a puzzle  and challenge to me as to what is the relationships between  the language of subject -means -object  and stimulus-means -response. The former language has its roots in
philosophy and has been also used by the philosophers who ! have generalized the results of activity theorists (Ilyenkov, Lektorsky),
the latter in physiology  and was, I guess, an unavoidable language to be used in the critical reformation of psychology in the early 1900s.
It would be very interesting to analyze the transition between these two languages in the characterization of mediation. It might be that
Leontjev's uses of "object of activity" plays here an important role.

The expression pre-industrial conditions - I agree much of what Steven says.  Human consciousness is social in its origins and mediated from the beginning, I took this point of view from Schmidt to illuminate the historical  and changing dimension of subjectivity (or agency). his attempt to historisize the problem of subject-object relationship is
in my mind illuminating compared with pure philosophical distinction. I guess I used it also because of Latour's critic of the sense of >(!-- D(["mb","subject-object distinction in general. In some sense it continues  the
historical account of the emergence of specifically human subjectivity
delineated by Leontjev in his study on the development of mind.

Best

Reijo

From: "Steve Gabosch" (sgabosch@comcast.net)

 

Hi Xavier,

 

Giving it some reflection, I think I was trying to pack too much into the word “labor,” which is one of several essential social components that make it possible for humans to be historical, cultural, creative, and, to add Anna’s point, to be self-reflective – to have social histories, community cultures, and individual psychologies.  Other essential components in addition to labor are the forces of production (technology plus labor), the relations of production (social relations, such as classes, regarding labor and the products of labor) and human activity in general (play, intimacy, leisure, etc. etc.).  These are the kinds of things I had in mind in using the term “labor” when I said “if we begin with labor, we can explain all the rest.”  But the concept of labor alone, taken as it is usually me! ant, as you point out, is insufficient for such an explanation.  Other essential components are also required to create a viable explanation and capture the essential dynamics.  I appreciate the criticism.

 

Some really interesting methodological and ontological issues are emerging in our discussion, Xavier.  We agree in principle on a number of points.  We agree that a developmental approach is imperative, and interactionist and linear approaches are inadequate.  We both reject Cartesian dualism.  We agree that labor, social relations, and human activity in general, develop.  I agree with your description, up to a point, that the social and biological constructed one another in the genesis of the human species.  And I very much agree with your statement that development goes forth on multiple hierarchical levels simultaneously and not in a simple linear progression.

 

Let’s step back a moment and think of the levels and layers of reality that create the possibility of humanness today.  Nothing new in my point here, but I think it is useful to remind ourselves of the long view and the big picture.  We have spoken about the relationship of the biological and the social, but there is also the cosmological and much more.  The physics of our universe and galaxy, earth’s relationship to the sun and other cosmic objects, the character of earth’s physics and chemistry, earth’s geology, its atmosphere, bodies of land and water, and climate, its multi-billion natural history of evolving life forms, humanity’s mammalian and primate biological heritage, the transformative nature of social labor that in my opinion has become the defining characteristic of human beings (transf! orming nature, human society, and humanity itself) – all these levels of reality comprise inescapable features of the human species and every individual human’s every living moment, from the atoms of their body to the stars they see at night.  And the above mini-tour just takes us up to just 5 million or so years ago, the dawn of humanity!  Earth’s geology and its species have a natural history, but humanity has developed something unique – a social history.  And with that social history – out of the stuff of all the other “levels” and “layers” of existence – humanity has developed its cultures, its psychologies and its neuropsychological processes.

 

Taking all these levels into account is the challenge of any scientific explanation of human activity and existence.  I appreciate, Xavier, how much thought you have given this.  I believe that the genetic-historical and historical materialist approaches – two sides of the same approach, in my opinion – supplemented by new discoveries and paradigms found in complexity science, hierarchy theory and other recent disciplines – are in the best position to take on this challenge, and lay the basis for a cultural-historical psychology and a human activity science, of which I believe cultural-historical activity theory is an important part.

 

In my opinion, one of the core questions in this challenge to account for the multi-developmental nature of reality is to understand how multiple levels of existence mediate, interpenetrate, and co-develop with one another.

 

You make this point very nicely when you say: “when examining the functional systems that subserve higher order cognition, it seems that reorganization in one plane leads to reorganization in the other making new things possible that were not possible in the previous configuration.”

 

You continue: “It seems to me that this is what Luria was speaking of with regard to the hierarchical organization of functional systems in the nervous system.  I would be interested to learn if a similar hierarchical system exists in other analyses of activity--of which labor is obviously a quintessential example.  In other words, do more basic forms of labor lead to more complex ones?”

 

Very good question.  Marx and Engels answered yes.  So let me turn to Marx for some insight into this and some methodological aspects of the other topics we are discussing.  Chances are pretty good that you and most of us here have studied the paragraphs below by Marx before, from his 1859 Preface to A Contribution to the Critique of Political Economy.  These passages are widely quoted and studied because this is one of Marx’s most succinct explanations of his theory of human social development, historical materialism.

 

I think these are very interesting passages to reread in light of the ideas we are discussing, as well as the question you ask about higher forms of labor, and your point about reorganization in one plane leading to reorganization in another.  I believe Marx’s methodological approach to sociology outlined below reflects the core of Vygotsky’s and Luria’s methodological approach to psychology.  I think, in fact, that this approach can apply to all levels of reality and every scientific discipline, although of course as we move across planes (or levels or layers or domains) of reality, new elements and laws must be understood for each, and how they relate to one another must be understood concretely and historically.  A developmental and historical methodology – a dialectical approach - is not about d! irectly providing the answers, its about asking the right questions so we can get to work.  I think you nail a very important aspect of such a methodology when you emphasize how different planes of reality reorganize each other.

 

In thinking about the genetic-historic approach and applying it to a multi-leveled reality, compare the development of life forms on earth transforming a previously inorganic planet, the emergence of human social production relations overthrowing a previous purely mammalian existence, the emergence of higher cortical functions being generated by the interactions of an individual child with a surrounding and unavoidable social matrix, and Marx’s central idea below, that under certain circumstances, the emergence of new modes of production overthrow outmoded relations of production and create the conditions for social revolution.  I see fundamental dialectical commonalities.  See what you think.

 

I copied this from http://www.marxists.org/archive/marx/works/1859/critique-pol-economy/preface.htm

 

Marx, 1859:

“In the social production of their existence, men inevitably enter into definite relations, which are independent of their will, namely relations of production appropriate to a given stage in the development of their material forces of production. The totality of these relations of production constitutes the economic structure of society, the real foundation, on which arises a legal and political superstructure and to which correspond definite forms of social consciousness. The mode of production of material life conditions the general process of social, political and intellectual life. It is not the consciousness of men that determines their existence, but their social existence that determines their consciousness. At a certain stage of development, the material productive forces of society come into conflict with the existing relations of production or ­ this merely expresses the same thing in legal terms ­ with the pr! operty relations within the framework of which they have operated hitherto. From forms of development of the productive forces these relations turn into their fetters. Then begins an era of social revolution. The changes in the economic foundation lead sooner or later to the transformation of the whole immense superstructure.

 

“In studying such transformations it is always necessary to distinguish between the material transformation of the economic conditions of production, which can be determined with the precision of natural science, and the legal, political, religious, artistic or philosophic ­ in short, ideological forms in which men become conscious of this conflict and fight it out. Just as one does not judge an individual by what he thinks about himself, so one cannot judge such a period of transformation by its consciousness, but, on the contrary, this consciousness must be explained from the contradictions of material life, from the conflict existing between the social forces of production and the relations of production. No social order is ever destroyed before all the productive forces for which it is sufficient have been developed, and new superior relations of production never replace older ones before the material conditions for thei! r existence have matured within the framework of the old society.

 

“Mankind thus inevitably sets itself only such tasks as it is able to solve, since closer examination will always show that the problem itself arises only when the material conditions for its solution are already present or at least in the course of formation. In broad outline, the Asiatic, ancient, feudal and modern bourgeois modes of production may be designated as epochs marking progress in the economic development of society. The bourgeois mode of production is the last antagonistic form of the social process of production ­ antagonistic not in the sense of individual antagonism but of an antagonism that emanates from the individuals' social conditions of existence ­ but the productive forces developing within bourgeois society create also the material conditions for a solution of this antagonism. The prehistory of human society accordingly closes with this social formation.”

 

Best,

-       Steve

 

From: "Matt Brown" (mjb001@ucsd.edu)

Here's a little tidbit from Dewey that I think is interesting for several reasons: it answers the question from earlier about whether Dewey is concerned with the social, it provides a sort of summary statement of central Deweyan theses, and it is exceptionally clear (for Dewey). From Chapter 11 of Art as Experience:


Experience is a matter of the interaction of organism with its environment, an environment that is human as well as physical, that includes the materials of tradition and institutions as well as local surroundings. The organism brings with it through its own structure, native and acquired, forces that play a part in the interaction. The self acts as well as undergoes, and its undergoings are not impressions stamped upon an inert wax but depend upon the way the organism reacts and responds. There is no experience in which the human contribution is not a factor in determining what actually happens. The organism is a force, not a transparency.

Because every experience is constituted by interaction between subject and object, between a self and its world, it is not itself either merely physical nor merely mental, no matter how much one factor or the other predominates... In an experience, things and events belonging to the world, physical and social, are transformed through the human context they enter, while the live creature is changed and developed through its intercourse with things previously external to it.

 

From: "Mike Cole" (mcole@ucsd.edu)

Steve et al-- In the very interesting exchanges that Steve, Javier, and others have gotten us into, the concept of labor
plays a key role. But what is meant by labor has not been explicated.

A while back I looked at this issue with respect to LSV and came away thinking it needed a lot more attention. Question:
where might such attention be focused?? Marx, but what part of Marx? Others? If there is interest, I can try to pull together what I found vis a vis LSV's comments on this central idea.
mike

From: "Matt Brown" (mjb001@ucsd.edu)

I think the places to go in Marx are probably "The Alienation of Labor" from the 1844 Manuscripts and Volume 1 of Capital. Consider the following from Capital vol. 1, chapter 1, sec. 2:


In the use value of each commodity there is contained useful labour, i.e., productive activity of a definite kind and exercised with a definite aim. Use values cannot confront each other as commodities, unless the useful labour embodied in them is qualitatively different in each of them. In a community, the produce of which in general takes the form of commodities, i.e., in a community of commodity producers, this qualitative difference between the useful forms of labour that are carried on independently by individual producers, each on their own account, develops into a complex system, a social division of labour. (my bold)



At a time when I was thinking about the concept of labor, I wanted to distinguish work (activity?) from labor in the following way:


A person's self-actualization, at least in part, consists in doing work. Working is the primary way in which a person creates himself as a particular individual. Recognition from a community of workers in the same field contributes to one's self-actualization as well. A worker is alienated from his work insofar as he sees it not as an end unto itself, but as a means to an end. A person who works joylessly to put food on the table is alienated from his work and the products of his work and is to that extent lacking in his self-actualization. A worker for whom the activity of work is worth doing in itself and who sees himself reflected in his creation obtains a higher degree of actualization. We might call the former labor and the latter work, after Marx, Engels, and in recent times Arendt.



I'm pretty sure when I wrote that I had recently read an excellent essay by Peter Herbst called "Work, Labour, and University Education," from The Philosophy of Education (ed. R.S. Peters, 1973). Looking at it, he uses the distinction I refer to, attributing it to Hannah Arendt in chs. 3 & 4 of The Human Condition, and the whole thing seems clearly influenced by Marxian thought.

Consider the following definition from "The Alienation of Labor":


In this definition--that the laborer is related to the product of his labor as a strange, foreign object, lies all these consequences. For from this hypothesis the following becomes clear: the more the laborer labors, as well as the more powerful the alien, object world which he builds over himself becomes, the poorer he himself becomes, that is, his inner world, as he owns less. The same thing occurs in religion. The more people place in God, the less they retain in themselves. The laborer places his life in the object; but now it [his life] belongs less to him than to the object. Therefore, the more this happens, the more deprived of objects the laborer becomes. What the product of his labor is, he is not. Therefore, the greater this product, the less he becomes. The alienation of the laborer in his product has this significance: since his labor is an object, not only does this labor become a separate existence, but it is also separate from him, independent, alien to his e! xistence and a self-sufficient power which exists above him, that the life, which he has bestowed on the object, confronts him as something hostile and strange.



That's all I can think of tonight. I'd be very interested in what you found in LSV, Mike.

From: "Steve Gabosch" (sgabosch@comcast.net)

 

Thanks for initiating this thread, Mike.  I think grounding this line of inquiry in what Vygotsky had to say is a most excellent place to start.  Your efforts toward this would be most appreciated.

 

“What is meant by ‘labor’?” is an essential question.  Two related questions are how to understand the special case of productive labor in terms of the general case of human activity, and conversely, how to understand general human activity in terms of the uniquely human endeavor, productive labor.

 

On the question of which part of Marx to focus attention on for insights into these questions, I would caution against counterposing the Marx of the 1844 Manuscripts to the more mature Marx, as some do.  I agree with the way Matt goes to Capital.  Marx only began to adopt and develop the labor theory of value – the theory that the exchange value of commodities is based on labor – in 1845, and did not fully develop this concept in terms of money, surplus value and prices until the late 1850’s.  A booklet of essays by Ernest Mandel and George Novack entitled The Marxist Theory of Alienation, 1970, Pathfinder Press, and the book by Ernest Mandel, The Formation of the Economic Thought of Karl Marx, 1971, Monthly Review Press, are very helpful for sorting out this question of Marx’s evolution on the question of labor, value, alienation, etc.&! nbsp; Interestingly, there is little mention of the term "alienation" in Capital (first published 1867, in German) but there is quite a bit more use of the term in the Grundrisse (notebooks written 1857-1858).

 

The question of “what is labor” is closely entwined with the question of the alienation of labor, which takes us back to Hegel and Feuerbach, Marx’s ideas in the 1844 Economic and Philosophic Manuscripts, and confronts us with a host of twentieth century literature and debate on the subject of alienated labor and alienation in general.

 

Some of the ideas Matt raises have been frequent themes in these discussions. 

 

One, stemming from Hegel, is the idea that labor by its nature is “alienated” because a human “objectifies” themselves (a person experiences objectication) and puts their (subjective) work into some external product (object) which becomes “estranged” (a person experiences the estrangement of the work they did).  This seems to be the idea Peter Herbst is describing in the quote Matt offers.  Marxists usually respond to this line of reasoning with the answer that labor is alienated, not in all cases, but under specific social systems and conditions, such as capitalism.

 

Another common theme, captured in the interesting passage Matt wrote and quotes, is that labor is “alienated” when the laborer works joylessly and achieves little self-actualization.  This touches on a host of forms of alienation that have gotten much discussion since Hegel and then Marx looked into them. 

 

Since Marx, there has been a wide range of other arenas where the idea of alienation has been applied – the psychology of the individual, the consumer, the alienation of pre-industrial and pre-agricultural societies from nature due to lack of control of its forces, the alienation caused by the usurpation of peasants from the land and forcing them to populate the growing proletariats of recent centuries, the alienation of living under supposedly socialist societies that were under the steel rule of Stalinist bureaucracies, etc.  The idea of alienation has been discussed and debated throughout the Twentieth Century, especially since WWI.

 

Let me finish up with a passage from Engels I found by googling the Marxist Internet Archives site.  Engels emphasizes that in class society, while other classes come and go, there has always been a working class, a producing class – slaves, serfs, proletarians, etc.  This is a good example of the importance of looking at labor not just in terms of “activity” but also in terms of the real mode of production and associated class structure it takes place in.  I bring this up as a way to remind ourselves that labor is always “situated,” and not just in some field or workplace, but also in a social system.  The nature of labor and its alienation must be discussed not just in terms of the forces of production (technology) or the individual (techniques, skills),! but also the relations of production, the society.

 

 

from:

http://www.marxists.org/archive/marx/works/1881/08/06.htm

From an Article by Frederick Engels in the Labour Standard 1881

Social Classes — Necessary and Superfluous

Source: Reproduced from the newspaper;
Written: early August 1881;
Published: No. 14, August 6, 1881, as a leading article;
Transcribed: director@marx.org, Labor Day 1996.


The question has often been asked, in what degree are the different classes of society useful or even necessary? And the answer was naturally a different one for every different epoch of history considered. There was undoubtedly a time when a territorial aristocracy was an unavoidable and necessary element of society. That, however, is very, very long ago. Then there was a time when a capitalist middle class, a bourgeoisie as the French call it, arose with equally unavoidable necessity, struggled against the territorial aristocracy, broke its political power, and in its turn became economically and politically predominant. But, since classes arose, there never was a time when society could do without a working class. The name, the social status of that class has changed; the serf took the place of the slave, to be in his turn relieved by the free working man -- free from se! rvitude but also free from any earthly possessions save his own labour force. But it is plain: whatever changes took place in the upper, non-producing ranks of society, society could not live without a class of producers. This class, then, is necessary under all circumstances -- though the time must come, when it will no longer be a class, when it will comprise all society.

 

Hickman article: part I

 

From: "emma johnson" (ekjohnso@ucsd.edu)

WELL, I wrote a whole elaborate question, and it got lost in the ether, perhaps due to the borrowed nature of the wireless connection I use at home. I will try again, convinced that I will never be as eloquent as the first time around.

It went something like this: in light of STS, Science Studies, Laboratory studies, SSK, etc, do we need to rethink, rephrase or otherwise modify the notion of experimental science as a model of true inquiry, cognition, thinking? There has been plenty of work debunking an idealized notion of scientific progress and process.

Please do not suspect me of throwing out the baby out with the bathwater! I submit this humbly, I hope. Perhaps I am just being too literal, or have become too suspicious as a result of my education and temperment.


-Emma

(summary follows)
Part One summary (page 17-30, parts 1-V)

I.
Hickman begins this chapter with a summary of the Greek notion of techne, meaning professional competence. It is a realm between nature/order and chance/disorder. Also, it is between experience and theoretical knowledge. It is knowledge in flesh and blood.

Central to Dewey is a rejection of Greek essentialism. Active productive skill is the key to understanding humans, and the construction of theories is a special kind of productive skill. Production is a way of making lived experience meaningful. For Dewey, technology is between doubt and resolution.

II.
Dewey replaces a theory of knowledge with at theory of inquiry. Most of the time we are engaged in “non-reflective” experiences: aversion, suffering, esteem, endeavor, etc. These can later become the subject of inquiry, but a lot of what we do it experienced as immediate, perhaps as the result of earlier inquiry. Experience is formed of focus and context out of a vast and changing world.

When something is tense, difficult, or unresolved our experience changes, and we need help to understand it. There is a search for a tool to resolve the difficulty. This is inquiry,

A tool can be used in a trivial, mundane way as well. And furthermore, a tool only comes into being when it is used to do some sort of work. Hickman gives an example of having to change a broken light switch, which for someone unaccustomed to such work is a challenging task, but for an electrician a very mundane, unreflective experience.

Dewey’s conception of technology is as active, productive inquiry. It is always individual, in relation to an individual in a particular situation, and the situation for him means the total situation, not just the stated goal.

III.
Dewey can then go on to apply a tool-using, instrumentalist account to traditional epistemological and metaphysical account. He find “armchair metaphysics” to be literally inconsequential, because it deals with problems that nobody actually encounters, does not begin with real doubt, and proposes theories that cannot be tested.

Dewey cuts across the inner-outer, mind-body divides. Instead, his key dichotomy is between immanent/natural/testable and transcendent/supernatural/untestable.

For Dewey, vision is active, and “handles” its objects. This is a departure from a passive description of receptive senses. For Dewey, the senses are interactive, and he proposes that the ear is even more active than the eye.

IV.
The history of knowledge is, Hickman quotes from Dewey, “something going on in the world, in the observatory and the laboratory, and in the application of laboratory results to the control of human health, well-being, and progress.” Tools are tactile, observable, and public.

There are two kinds of knowledge: of acquaintance (primarily sensory), and knowledge about (the product of reflection or inquiry). This is from William James, but expanded by Dewey. Dewey points out that even the former may be a response due to previous inquiry, or even a habit or instinct inherited from “forebears.” There is a constant passing back and for the between these kinds of knowledge.

V. Is Dewey a forerunner of phenomenology? This is one idea that comes out of debates over Dewey’s take on different kinds of knowledge. Richard Bernstein and Webster Hood argue this. The view that he is a proto-phenomenologist comes from Dewey’s emphasis on lived experience. However, Hickman points out how this doesn’t take into account the importance of production for Dewey.

Ralph Sleeper points out something that Hickman seems to agree with: that Dewey is not phenomenological, but naturalistic. Dewey’s inquiry is modeled after experimental science.

 

From: "Matt Brown" (mjb001@ucsd.edu)

Mike asked me to summarize the middle sections of John Dewey's Pragmatic Technology by Larry Hickman. There are several interesting things going on in this chapter, even in just sections VI through X, which I'll go through. I'm focusing mostly on Dewey's positive views rather than his critical comments on traditional philosophy, though some of that will be necessary. So, here goes, section by section.




VI

Dewey rejects both realism and idealism. This is crucial for Hickman, because Dewey is offering up something pretty new in philosophy by doing so. Interestingly, the reason that Hickman thinks that Dewey rejects these poles is that his philosophy is technological, that is, it is concerned with tool-using practical activity. This connects right back up to Cole's discussion about artifacts combining ideal and material elements.

A major problem for realists and idealists according to Dewey is their treatment of time. Both traditions
spatialize time in inappropriate ways. Realists argue for eternally present objects, while idealists reify the entire span of history.

In contrast, Dewey "naturalizes" time, which means that he treated real change as an general feature of nature. He was influenced in this by Williams James, whose "stream of consciousness" drew attention to the changing or flowing element of consciousnes, Darwin, who treated human life as continuous with the development of species in general, and Hall, who (like Vygostky) saw development as the keystone to psychology.

VII

The starting point of inquiry for Dewey is not sense-data, but already formed situations, nonreflective experiences that are complex but immediately
had.

Three examples of paradigmatic nonreflective experiences: quenching thirst, enjoying conversation, and the delight of looking at a painting.

They are all experiences undergone, that hang together and exhibit a pervasive quality, in a direct way. These experiences are in a sense at rest, not problematic or tense, and they are of something valued intrinsically. These experiences are noncognitive, not flavored by inquiry. This doesn't mean that they don't have elements that were produced by inquiry, but they do not have the same logical function that reflective experiences do.

Inquiry comes in when things don't fit together, when there are conflicting demands, when the meaning becomes uncertain and problematic. Inquiry, or controlled thinking, uses instruments and tools and produces knowing in response to these situations.

Crucially, inquiry is a whole organism-environment affairs, not merely confined to the cranium.

VIII

Inquiry works to settle a problematic situation in a way that satisfies the initial problem. (This is what Hickman says, but I think, strictly speaking, it is the indeterminate or unsettled situation that inquiry has to resolve; the formulation of the problem, the the way in which the situation is taken as problematic, is up for grabs in the course of inquiry.)

Previous philosophical attempts to find a method for settling problems have proceeded, in the way of traditional epistemology, in a quest for certainty that lacks the practical dimension. This is the project of foundationalism, of the attempt to control thinking by means of immutable Truth, Goodness, etc. Often this gets understood by positing a real world entirely behind appearance and attempting to match our beliefs to the subterranean reality rather than the manifest.

On Dewey's picture, the world of experience
is the real world, in need of transformation in order to render it more coherent and more secure. Knowing the experienced world is instrumental to rearranging it and making it more useful to our purposes, and it takes place within experienced situations.

Knowing isn't outside the causal order, apart from nature, it is just that part that is controlled, whereas other causal chains lack intelligent control.

Knowledge properly so called only refers to the reflective and practical transformation of problematic situations. There are areas of existence / experience that are separate from knowledge, though they might be the entrenched results of inquiry, such as critical skills.

Hickman draws interesting comparisons between Wittgenstein and Dewey that basically come down to an instrumentalist view of language. And not only is language a tool in some sense, but tools are also linguistic, in that they say something about "their use and their consequences."

IX

The purpose of this section is a little obscure, but I think the basic suggestion is that scientific inquiry is more instrumental / technological than magic or poetry because it has extrinsic meaning that creates something new not for its own sake but as a tool to changing an unsatisfactory situation.

X

For the purposes of inquiry, it isn't useful to characterize the organism/environment boundary in terms of the skin. For this purpose, there are things inside the body that don't belong to the organism, while there are tools on the outside that do. Dewey readily refers body and motor skills as tools that a person uses. The mind-body distinction isn't allowed, either, only the "bodily" aspects of an organism that are continuous with the rest of nature, and the "mental" aspects that emerge from more complex interactions and is aware of meanings.




The biggest thing I take away from these sections of the chapter are the non-dualistic and non-dichotomous aspects of inquiry and experience for Dewey, and the similarity between the unity of the material and the ideal in the artifact for cultural-historical psychology. Also, I think the emphasis on cognitive activity being a subset of experience and existence is an important element.

 

 

John Dewey's Pragmatic Technology by Larry Hickman (sec 3)

 

From: "antonieta mercado" (amercado@weber.ucsd.edu)


XI

Hickman argues that although Dewey does not provide a concrete definition of the word “technology” it can be considered in his view, as the appropriate transformation of a problematic situation undertaken by means of the instrumentalities of inquiry”

For Dewey, inquiry is a technological activity, where inquiry takes place there is a shift from passive acquiescence of natural experience toward the active construction of artifacts to affect the control of nature. By developing a method, humans provide a model for successful inquiry (P. 46).

Inquiry is the controlled or directed transformation of an indeterminate situation into one that is so determinate in its constituent distinctions and relations as to convert the elements of the original situation into a unified whole. The study of inquiry, is the study of scientific method.

Dewey posed that the search for effective control of nature is one of the most basic human enterprises. Technology gives human beings control over nature. It transform knowledge from an aesthetic enjoyment of the properties of nature regarded as a work of divine art, to knowledge as a means of secular control. The attitude of control looks to the future: to production.

Science in this regard, is a branch of technology. Ayres, who was Dewey’s disciple argued that technology is the sum of human skills, and the sum of human tools (p. 46). In this regard, scientists use mathematical abstractions or theories, as they use microscopes or hammers.

XII

Dewey conceived “abstract” and “concrete” tools. The objects of logic, such as “if”, “or” are tools, just like skates or crutches that previously existed in raw materials p. 47). Those tools arise out of techniques of control. Scientific and theoretical inquiry is tool-using activity. Thus, logical entities are inventions, products of experience that have been refined and tested. Reflective thinking or inquiry starts as an effort to solve problems (p. 46).

XIII

Dewey was accused of being an idealist, and he argued that intelligence was not otiose (of no use). Thinking is what some of the actual naturally occurring things in the world.
The effects of thinking are existential. Experimentation ends in actual not just ideal alteration of a problematic situation (p. 48). Experimentation he argued is necessary for knowledge.

Dewey’s account have a realistic component, freely admits that certain raw or “brute” existences detected or laid bare by thinking but in no way constituted out of thought or any mental process. He compares the iron ore as an analogy of some of those brute stances of thinking, that later develop into something more concrete.

Dewey separates common sense, or institutional meanings and scientific meanings. Things that form part of everyday experience are turned into data by technology. Thinking thus is a technological task. Thinking is a form of industry that involves extraction, refinement and manufacturing (p. 50). Mature people live in environments that consist in work previously done and knowledge previously acquired.

XIV

For Dewey inquiry itself is a form of manufacture. The raw materials extracted from nature, become tools. For Dewey, instrumentalism is a technological view of knowledge, is a behaviorist theory of thinking and knowing.

A fire that is suggested may cause greater impact than an actual fire. Smoke indicates and signifies fire.

Before Darwing, nature was thought to comprise orders of fixed essences, or natural kinds. The doctrine of fixed essences involves the construction of a realm of essences apart from existences. Meanings should be prized in use, not in contemplation. Meanings are tools and not finalities. Experimentation is the bridge and the checking mechanism between essence and existence. Meanings are signs that imply other meanings. The term “brother” is lifeless until it comes to work in concrete life situations (p. 53).


XV

Dewey makes the analogy of producing metal artifacts and producing ideas, emphasizes his remarks on the logical “connectivenes” such as “and” and “therefore” Those connectors arise from inference and not by inference (p. 54). Russell supported this point arguing that logical connectiveness represents knowledge that is not obtained by inference but is primitive (primary materials for knowledge).

Telephones and mathematical relations are inventions of the mind to apprehend reality. Every instance of intelligent behavior has two important effects (p. 54). It controls the immediate outcome, and initiates an intrinsic meaning that may be put in the storeroom to serve in some later inquiry for which it will be deemed appropriate.

Costello criticized Dewey because he thought that tools were used to manage crises or to solve temporary difficulties, in a firehouse analogy, he criticized Dewey arguing that his theory focused on only “real fires” rather than false alarms, and that thought has to go beyond the temporary (p. 55).

However, the metaphor of the iron ore and the pig iron show that Dewey did not mean thought for temporary issues, but instead to focus on the raw pieces of thought.

XVI

Dewey’s instrumentalism makes knowing a technological activity, a kind of pro-duction. There could be no proper account of technology either in traditional versions of realism because of the view that knowledge must conform to a certain set of conditions, or in traditional forms of idealism, because of their view that our knowledge is in fact constituted in a coherent way by a supernatural or transpempirical thinker, that gets its knowledge without objective data.

Instrumentalism conceives science as the knowledge that “meanings for possible use in remote and contingent situations”, as a special type of technology, and renders theory as a practice. Dewey, meaning has three loci: Meaning of an object (connotative) meaning of an idea (denotative), and meaning of a truth.

Dewey called meanings in common sense or in institutional use it, “coarse” and argued that may be inconsistent with one another because they exclude much of what is necessary for the control of activity or the application of productive skills.

XVII

Finally Hickman argues that meaning and technology are central concepts for Dewey, arguing that meanings of words are always contextual, they do not correspond one-to one with their objects. Words mean what they mean in connection with conjoint activities. Meanings vary with time and with different groups or activities, ideas and conclusions get replaced by other ideas and conclusions developing new meanings. Technology on the other hand is the active use of productive skills as the most satisfactory method of inquiry, as production. Pragmatism, experimentalism, and instrumentalism connote technological production, and construction. Science in this context, is an art to utilize other arts.

 

From: "emma johnson" (ekjohnso@ucsd.edu)

Well, I had prepared a summary of this same section (apparently I got confused. It wouldn't be the first time).

Antonieta has done a much more thorough, and articulate job than I did, so I will refrain from adding my summary here.

I would like to ask some really basic questions, which I have not thought through in much detail.

1- Is the term "experimentation" being used here in as specialized, and idiosyncratic a way as "technology"?

2- Where are the other people in this account? Although a convincing picture of a materialist, instrumental theory of knowledge is described, what about intersubjectivity?

Its hours past my bedtime, and I am running on unreliable borrowed wireless access (from the neighbors). So I hope these questions make sense.

-Emma

 

From: "Matt Brown" (mjb001@ucsd.edu)

Emma,

Here are my thoughts on your questions:

>1- Is the term
>"experimentation" being used
>here in as specialized, and
>idiosyncratic a way as
>"technology"?

As far as I understand Dewey, he doesn't mean "experimentation" in any specialized way. I guess he is relying on a certain ambiguity in the term, though. He means to capture both the highly controlled interventions that characterize modern science, as well as our everyday "experimenting," i.e., trying things out to see if they work. Dewey saw "experimentation" as the great new innovation of the scientific revolution.

>2- Where are the other people
>in this account? Although a
>convincing picture of a
>materialist, instrumental
>theory of knowledge is
>described, what about
>intersubjectivity?

The communal aspect is there in Dewey, and I think Hickman talks about it later in his book, but I think it often gets underemphasized in presentations of Dewey's theory of inquiry (including this one). Think of it this way: if one model for inquiry is science, and scientific inquiry is irreducibly social (I think/hope Dewey agrees), then inquiry is gonna be pretty social.

The key text here for Dewey is Chapter 3 of the 1938 Logic: "The Existential Matrix of Inquiry: Cultural."

From: "emma johnson" (ekjohnso@ucsd.edu)

I marked up my copy of the article with some of the same passages that Steve points to on p300.

But I was wondering about some really different problems with this nature/culture problem. I was thinking not only of brains, but of the natural world itself. So I find I've underlined some slightly different parts:

Just following the passage Steve quotes about pre-industrial and industrial society (and thank you Steve, for pointing out that this is hardly a simple distinction), I underlined: "Nature ceases to be a natural entity, independent of man." and later on the same page, "Natural history is indivisible from human history, which is characterized by transformation of nature by human work, the creation of objects by humans." Then on p301, "Nature cannot be reduced to the historical modes of its appropriation in practice." and then, quoting Lenoir: "Nature is plastic but not infinitely malleable. It resists..."

This article set me thinking that I need to think about how to use these theoretical frameworks (AT, ANT) to look at environmentalism, ecology, and conservation science. I am sure I am not the first (or second, and so on) to think of this, and I would be delighted with suggestions and bibliographic references. Because, depending on your unit of analysis, the nature-culture problem seems to be as big as, well, the atmosphere of our entire planet.

-Emma

From: "antonieta mercado" (amercado@weber.ucsd.edu)

Emma

I am sorry, probably I got confused too, but I wanted to do the summary before my other papers got on the way, so I posted it on Monday.

Let me think, I will just answer your question about intersubjectivity. I think for Dewey, experience entailed the interaction between subjects and objects of knowledge, I do not think he separate those realms. Historical knowlege (accumulated knowledge) is a product of subjects and the transmission of that knowledge is not acquired in a vacuum. Experimentation and use of objects is not isolated from intersubjective activity or interaction with others. Well, it is running a little bit late today. I am still in the offfice, and I have still to go home. See you tomorrow in class. Hope some other person engages in this discussion. Otherwise it will be useful to post some of our notes for class later.

See you tomorrow

Antonieta

Xavier's post:

Regarding Steve's question about simplifying material things to "two kinds," I understood this to be referring to the dual nature of artifacts (i.e. ideal and material).  To follow Steve's reference to Vygotsky's tripartite structure sketch of [stimulus --- artifacts/tools/signs --- response], is this "two kinds" not referring to the middle part of this diagram?  I would also add that in order to highlight the dual nature of artifacts we should perhaps amend the diagram to explicitly include the processing of a material artifact by the brain (this represents the ideal half of the duality of artifacts).  The second that our brains engage a material artifact involved in a particular activity, the dual nature of the artifact comes into play.  In order to maintain a material monism that continually emphasizes the process and dynamic nature of activity, we must redefine the brain as an open system that cannot function without the fuel of cultural ! artifacts.  I know I personally harp on this, but I see Luria as the consummate activity theorist in insisting that our unit of analysis be the functional systems which includes both the cortical and extracortical aspects of activity since one truly cannot exist without the other.

I believe this also speaks to Steve's second question which seems to beg the question of whether the natural and the social can truly be separated in modern humans.  In principle, one would like to think that the natural has some precedence in the temporal scheme of things at least historically through our evolution; however, this brings us back to what originally enabled the social to influence the processing of the natural.  Put a different way, what allows for the external to become internal or the material to become ideal.  Again we see a bootstrapping type problem which involves the co-construction of the social world and the functional circuits of the brain.  Whereas at one time in our individual ontogenetic development we each experienced Jame's buzzing confusion at the sight of a red bulbous material with a green thin extension that hurt us if we touched it, we can no longer return to that state because now the second we see a rose we! recognize it as such and associate it with a sweet fragrance, romantic love, and a host of other culturally determined associations.  We can no longer separate the experience of what it once was from what it has been for generations.  It is now a rose and will always be a rose unless parts of our brain begin to degenerate which do not allow us to recognize it as such.

I was struck by Dewey's three-term structure of functional coordination which echoed the idea's of Luria's functional systems theory and also Hutchin's insistence on the centrality of coordination in distributed cognition.  What do others think about this?

From: "Steve Gabosch" (sgabosch@comcast.net)

 

I was interested by Xavier’s description of the relationship between ideality, artifacts, and brain processing in his 2/16 post.  His statement that “the second that our brains engage a material artifact involved in a particular activity, the dual nature of the artifact comes into play” got my attention.

 

I suggest that the dual nature of the artifact *precedes* individual brain processing, in the same way that Vygotsky and Luria suggest that social relations (the interpsychological) precede the higher mental functions (the intrapsychological).  The argument that brains and artifacts (the cortical and “extra-cortical”) are both essential elements of the process does not address the fundamental question of genesis.  In my opinion, it is precisely this genetic question, of what precedes the psychological (the mind, the subjective, the psyche, brain processes, higher mental functions, pick your terminology) that makes cultural-historical psychology and cultural-historical activity theory stand out from all other approaches.

 

- Steve

 

Mike Cole wrote this post that did not make it to the webboard this morning 2/18:



Steve--  I intepret Xavier's statement differently than you do. I guess its a matter of interpreting "comes into play."  Before we think of, perceive, encounter, (whatever) an aspect of the world, how can we be interacting with/through it?  I do not think Xavier is claiming that the brain creates the properties of the artifact de novo, but rather, that it is the interaction of artifact and human in action from which emerges human thought. The artifact had to have been there, of been created in the interaction (at the latest, and even then, almost certainly owing to prior experience) so issues of priority remain as you say.

My guess.
-mike

My response:
I wonder if the underlying question Xavier and I are answering a little differently is this: where and when does "ideality" originate?

Xavier says "the processing of a material artifact by the b! rain ...  represents the ideal half of the duality of artifacts ..."

Following Ilyenkov as I understand him, I locate ideality in the social processes that precede individual encounters with artifacts.  I suggest that social processes, not brain processes, embody artifacts with ideality, with cultural meaning.  It would surprise me to find a single person seriously arguing that material properties are created by brain processes, but it would not surprise me to find many people arguing that cultural meaning (ideality) is created by them [brain processes].  I think - and Xavier certainly should speak up here, I may indeed be misinterpreting him - that Xavier is suggesting the latter. 

A more specific question that can be asked about where does ideality originate is: does ideality (cultural meaning) originate in social processes or in brain processes?  The particular CHAT approach I am attracted to - I think that cultural-historical activity theory ! is still working this out, so there are multiple approaches - ! argues t hat socially created cultural meanings create the "ideal half" of artifacts that human brains process.  This contrasts with what I think Xavier is saying, that human brain processes create the "ideal half" of artifacts that we socially share and interact with.

- Steve

From: "xavier cagigas" (xcagigas@ucsd.edu)

First off, thank you Steve for picking this up and starting a true dialogue.  I guess I should start by clarifying a few things and taking a more formal approach to what I am trying to say.  I think Mike summarized my perspective quite well by stating "...it is the **interaction** of artifact and human in action from which emerges human thought." [my emphasis]

I don't think it's a matter of either the brain *or* social processes being responsible for the ideal aspect of artifacts, but rather both simultaneously.  I understand the desire of wanting to address the "genesis" of this ideality; however, I take this to be a "chicken/egg" type problem.  I think I referred to it as a "bootstrap" problem in the previous posting.  If it were all social processes then other social animals should also experience the dual nature of artifacts and use tools/signs the way that we do, but they don't...  The complex functional neural circuits in our brains that subserve "higher coritcal functions," which are ultimately honed and completed by human social interaction, form the nexus of the dual nature of artifacts.  In other words, the properties under our scrutiny are distributed between the human individual and the cultural artifacts that are actively engaged in meaningful activity.  It is the coordination in actio! n of our brains, eyes, hands, etc. with historically developed artifacts which allow us to think.

I think it may be counterproductive to dichotomize social processes and brain processes--even in order to address the question of genesis.  If our starting point is activity, then we also have to ask where/when does activity begin?  I think this highlights the difficulty of always maintaining a developmental perspective of things.  We are trying to examine an activity system that is always there...always constructing and reconstructing itself.  Ontogenetic development and the formation of higher cortical functions is dependent on an individual's interaction with others on an interpersonal, historical, and cultural level (at least) simultaneously.  In terms of our phylogenetic development, certain parts of our brains had to evolve before the dual nature of artifacts could even be possible.  Without our neurophysiological ability to partition space and time and simultaneously hold the physical and ideal aspects of an artifact in mind, one ! could argue that the cultural and historical would not exist and even the social would constantly have to be re-created in the moment (as it is in other primates).

I think activity theorists need to start taking the working brain seriously as part of the activity system, as an integral component of its structure.  Steve, you stated that "social processes, not brain processes, embody artifacts with ideality, with cultural meaning."  I would argue that all social processes are embodied in the brain and equally that all brain processes are a product of social processes.  I realize that this may sound tautological at first blush, however, this circularity is only true on the individual level.  When we distribute the agency across different people, then we can see that social processes are embedded in the brains of members of the collective and in the artifacts that they have created through history and both *learn* and *teach* others to use.  History is held in artifacts but only as long as there is someone there to use them.  The same goes with the brain:  use it or lose it.

I guess what I am trying to say is that we should include what we know about the dynamics of the brain as an open system that is acted upon by the world, acts upon the world, and acts upon itself.  Artifacts do not act upon themselves do they?  The brain does--it is the very embodiment of mediation itself.  That is why I believe Luria was really on to something.  His construct of the functional system included both the dynamics of the brain and the dynamics of activity in the world--both of which are inseparable the one from the other in terms of how meaningful action is created and sustained.  Artifacts in the socially shared world have a history and are the products and producers of human thought; however, the mediation of thought is equally made possible by the functional organization of the brain and its creation of and coordination with artifacts.

I am no expert in cultural-sociohistorical activity theory, however, I would ask others with more expertise to chime in.  I think Luria's contribution to activity theory has been under valued and under appreciated.  I see a thread of continuity running throughout his work and believe that the reason why his neuropsychology continues to make sense and bear fruit is because it is firmly rooted in cultural-sociohistorical activity theory.  Why then wouldn't the converse be equally rewarding and true?

Thank you again for raising these issues.  I hope others engage these questions as well.  Don't be shy now.  We all have lots to learn from each other.

- Xavier

From: "xavier cagigas" (xcagigas@ucsd.edu)

First off, thank you Steve for picking this up and starting a true dialogue.  I guess I should start by clarifying a few things and taking a more formal approach to what I am trying to say.  I think Mike summarized my perspective quite well by stating "...it is the **interaction** of artifact and human in action from which emerges human thought." [my emphasis]

I don't think it's a matter of either the brain *or* social processes being responsible for the ideal aspect of artifacts, but rather both simultaneously.  I understand the desire of wanting to address the "genesis" of this ideality; however, I take this to be a "chicken/egg" type problem.  I think I referred to it as a "bootstrap" problem in the previous posting.  If it were all social processes then other social animals should also experience the dual nature of artifacts and use tools/signs the way that we do, but they don't...  The complex functional neural circuits in our brains that subserve "higher coritcal functions," which are ultimately honed and completed by human social interaction, form the nexus of the dual nature of artifacts.  In other words, the properties under our scrutiny are distributed between the human individual and the cultural artifacts that are actively engaged in meaningful activity.  It is the coordination in actio! n of our brains, eyes, hands, etc. with historically developed artifacts which allow us to think.

I think it may be counterproductive to dichotomize social processes and brain processes--even in order to address the question of genesis.  If our starting point is activity, then we also have to ask where/when does activity begin?  I think this highlights the difficulty of always maintaining a developmental perspective of things.  We are trying to examine an activity system that is always there...always constructing and reconstructing itself.  Ontogenetic development and the formation of higher cortical functions is dependent on an individual's interaction with others on an interpersonal, historical, and cultural level (at least) simultaneously.  In terms of our phylogenetic development, certain parts of our brains had to evolve before the dual nature of artifacts could even be possible.  Without our neurophysiological ability to partition space and time and simultaneously hold the physical and ideal aspects of an artifact in mind, one ! could argue that the cultural and historical would not exist and even the social would constantly have to be re-created in the moment (as it is in other primates).

I think activity theorists need to start taking the working brain seriously as part of the activity system, as an integral component of its structure.  Steve, you stated that "social processes, not brain processes, embody artifacts with ideality, with cultural meaning."  I would argue that all social processes are embodied in the brain and equally that all brain processes are a product of social processes.  I realize that this may sound tautological at first blush, however, this circularity is only true on the individual level.  When we distribute the agency across different people, then we can see that social processes are embedded in the brains of members of the collective and in the artifacts that they have created through history and both *learn* and *teach* others to use.  History is held in artifacts but only as long as there is someone there to use them.  The same goes with the brain:  use it or lose it.

I guess what I am trying to say is that we should include what we know about the dynamics of the brain as an open system that is acted upon by the world, acts upon the world, and acts upon itself.  Artifacts do not act upon themselves do they?  The brain does--it is the very embodiment of mediation itself.  That is why I believe Luria was really on to something.  His construct of the functional system included both the dynamics of the brain and the dynamics of activity in the world--both of which are inseparable the one from the other in terms of how meaningful action is created and sustained.  Artifacts in the socially shared world have a history and are the products and producers of human thought; however, the mediation of thought is equally made possible by the functional organization of the brain and its creation of and coordination with artifacts.

I am no expert in cultural-sociohistorical activity theory, however, I would ask others with more expertise to chime in.  I think Luria's contribution to activity theory has been under valued and under appreciated.  I see a thread of continuity running throughout his work and believe that the reason why his neuropsychology continues to make sense and bear fruit is because it is firmly rooted in cultural-sociohistorical activity theory.  Why then wouldn't the converse be equally rewarding and true?

Thank you again for raising these issues.  I hope others engage these questions as well.  Don't be shy now.  We all have lots to learn from each other.

- Xavier

 

From Judy

Hi all,

To begin, I wanted to thank all those people who wrote summaries of Hickman’s interpretation of Dewey, which have been helpful to me. I wished while reading Hickman’s article that the headings were not roman numerals but meaningful titles and I think those summarizing this challenging article were helpful in suggesting the central idea or purpose of each section.

I agree with Matt that I didn’t think that experimentation was being used in a specialized way except that I thought it was a broad definition of the meaning of the word and not restricted to what happens in science labs.

As for Emma’s question about intersubjectivity, I thought that was a good one. I’m glad to hear from Matt that that there’s more we don’t know about from original text by Dewey.

One of the challenges for me is distinguishing Dewey from Vygotsky’s cultural-historical psychology. To do this I started to try to get a ‘big picture’ perspective.

The articles and the summaries talk about broad philosophical positions. So I now have four in my mind. Definitions below, are, by the way, from The Free Dictionary (http://www.thefreedictionary.com/naturalism), which was a convenient source.
:
1. idealism (the object of external perception, in itself or as perceived, consists of ideas),
2. realism (physical objects exist independently of their being perceived.),
3. naturalism (Dewey’s position or according to The Free Dictionary the system of thought holding that all phenomena can be explained in terms of natural causes and laws.) and
4. materialism (The theory that physical matter is the only reality and that everything, including thought, feeling, mind, and will, can be explained in terms of matter and physical phenomena)

Beyond this we seem to have philosophies that are monist or dualist. Idealism and materialism are monist. Realism I think is dualist. I’m guessing Dewey is monist.

I am not a philosopher, which I think is probably evident. I am a psychologist and before that a computer scientists, but I would like some working knowledge of these definitions because I think that mediation is something that challenges idealism and realism.

Everything really boils down to understanding Vygotsky’s triangle of mediated action, which I believe is an attempt, like Dewey’s of finding a middle ground between idealism and realism. By the way, this links to posts by Steve and others on this topic. Either physical objects that exist independently are perceived and manipulated directly or they are manipulated indirectly through cultural artifacts. The cultural artifacts (ideas, conventions, and tools etc.) make possible and constrain our interactions with physical objects and also influence our perception of them. Either way, there is a kind of co-dependency between subject and object and artifact that makes mediation indivisible into parts. So the strong link between Vygotsky and Dewey, in my opinion, is the idea that sensual activity is basic. So, it’s neither ideas or reality but something (action or inquiry) that links a doer and the world that is basic.

I’d be delighted to develop this understanding with others.

One insight I personally had while reading Dewey was that he suggests that there are some experiences that are not inquiry, or possibly even action-oriented for that matter, but that these are direct experiences. Dewey gave the example of being thirsty or taking pleasure in art. This answered, in part, a question I posted earlier about whether or not there was anything that was not activity. I think that Dewey would answer that question by saying yes.

Judy

From: "Mike Cole" (mcole@ucsd.edu)

Hi Judy--

You are absolutely correct that it is essential to go back to the subject/object/medium triangle to understand vygotsky and dewey's similarities, which are extensive. Just how extensive is up for discussion, as you can see.

In my book, Cultural Psychology, another aspect needed for comparisons is to figure out what people are talking about using such terms as
"situation" (Dewey) and "activity" (LSV some and his followers a lot).
mike

From Anna

 

Hi All,
I'm learning a lot from the dialogue you are having here. Xavier wrote: "I guess what I am trying to say is that we should include what we know about the dynamics of the brain as an open system that is acted upon by the world, acts upon the world, and acts upon itself. Artifacts do not act upon themselves do they? The brain does--it is the very embodiment of mediation itself."

This brought into my mind what I had just read from Lektorsky's Subject, Object and Cognition (the part where he discusses self-consciousness and self-reflection as forms of knowledge). He writes:

"The point is that artificial mechanisms gathering information do not implement the process of cognition, they do not have self-consciousness or consciously realise the world of objects. The information gathered by these mechanisms only becomes a fact of cognition when it is assimilated by man. A submarine's periscope by itself does not see anything: the man using it does. Man's perception of the external world presupposes an elementary act of self-consciousness, otherwise it will not see anything even with the aid of a periscope (self-consciousness thus pertains to the man using the periscope rather than the periscope itself)." (Lektorsky 1984, 217.)

I dont know if this is relevant at all here, but wanted to share it with you. The issue of being conscious of oneself somehow relates to this discussion -- As it is only humans that can be conscious of themselves and they also only in relation to or through the objects, not directly... (and because of the developed brain processes and social relations?)

And, related to the actor network theory discussion: Does agency require consciousness?

a bit confused...
Anna

From: "Matt Brown" (mjb001@ucsd.edu)

Judy,

A couple of thoughts on what you've said:

1. Idealism

Idealism takes many forms. Subjective idealism, as set out by Bishop Berkeley, does seem to suggest that the object of perception is an idea, that all of reality consists in ideas, and esse est percipi (to be is to be perceived). Hegel's absolute idealism has ideas structuring the world, but the ideas are in some sense embodied. Peirce's objective idealism has ideas as the fundamental constituents of the universe, but ideas don't require perceivers. Perceivers are just abstract or general ideas, and what we call "matter" are just "effete" mind, ideas that have grown lazy and inactive.

2. Realism

I think you have this basically right. I'd say just that reality exists independently of mind. There might be a sense in which Peirce's idealism is a realism, then. There is also an epistemological sense of realism that says that our knowledge "mirrors" an external reality.

3. Naturalism

Naturalism means very many different things. Sometimes it is entirely negative (the view that there is nothing outside of nature or supernatural (which is a component of Dewey's view)). Sometimes it means materialism (which isn't). Sometimes it is the methodological stance that you've suggested, the view that everything can be explained in terms that don't refer to mysterious or supernatural causes.

4. Materialism

The sense of materialism you've given is probably the most popular. (my sense was: The theory that physical matter is the only reality and that everything, including thought, feeling, mind, and will, can be explained in terms of matter and physical phenomena) There is also
eliminative materialism, which says that physical matter is the only matter, and anything that cannot be explained in terms of physics must be tossed out. (Beliefs, desires, and meaning have all been suggested as candidates for elimination.)

5. Monism and Dualism

I think you're right to bring monism and dualism in to try to understand Dewey and Vygotsky. Most forms of idealism, materialism, and naturalism are going to be monistic, and some forms of realism seem obviously dualistic. Dewey is
not exactly a monist. He does insist that all dualisms should be broken down and that we should see the continuities, there. But he's also going to resist any attempts to eliminate or reduce the real down to one element of it, and he is in that sense a pluralist.

Both Vygotsky and Dewey seem anti-dualistic to me, and they also seem to want to avoid collapsing to idealism or materialism. So I would agree that they are trying to find a middle path, and I think that it is one that stresses continuity. They both rely on the concept of artifacts or tools or mediation in order to do so, though I think (perhaps against Hickman?) that Dewey relies on further resources as well.

6. Direct Experience

I think you are right to highlight the element of Dewey that suggests that there are non-inquirential experiences for Dewey. These are experiences
had rather than known. Rather than being something instrumental, like inquiry or object-oriented action, these experiences are enjoyed or suffered. In fact, this category is so important to Dewey that in Experience and Nature (LW 1), he calls it primary experience.

One should be careful not to treat primary experience as passive, however. If I'm an expert baseball player, playing catch with someone who is competent, my catching a normally-thrown ball will be entirely unproblematic, it will be had or enjoyed, it will be a kind of primary experience. Undisturbed habitual action is of this kind. (I think it would be interesting to compare this category in Dewey to "action" and "operation" in activity theory.)

Overington, Kenneth Burke and the method of dramatism

Overington, “Kenneth Burke and the Method of Dramatism,” Part I (pp. 131-143)

 

From: "noga shemer" (nnsevilla@yahoo.com)  

 

In this essay, Overington presents an exposition of Burke’s dramatism as a method (as opposed to his substantive position).  The following is a summary of key points and terms. 

 

Definitions: (132-3)

Overington first tackles two definitions of the system of dramatism offered by Burke.  The first specifies that this method is “designed to show that the most direct route to the study of human relations and human motives is via a methodical inquiry into cycles or clusters of terms and their functions.”  While the idea of terms is central to this definition, Burke also widens his definition to the study of human relations in terms of action, and uses both positions in his work. 

 

The Empirical Question: (133)

As a method, dramatism addresses these questions:

  How do persons explain their actions to themselves and others?

  What are the cultural and social structural influences on these explanations?

  What effect do connotational links among the explanatory (motivational) terms have on these explanations – and on action itself?

 

The Object of Inquiry: (133)

Burke’s object of inquiry is motive: “the language of motives, motives in language, language as motive.”  His tripartite understanding of motive incorporates both the accounts that people give for their actions (rationalizations) as well as the “motivational influence of sheer terms” (134). 

 

The Method: (135)

Dialectic is the method Burke uses to get at his object of inquiry (motive).  Dialectic is understood as “the concept of contradiction and the ironic presupposition that one approaches a fuller, more true, explanation for social action by taking opposing perspectives on that action.”  This dialectical epistemology is rooted in his dialectical ontology – his assumption of the “ontological priority of the physical, material world” (140).  Burke’s irony of contradiction is viewed as a protection from the influence of vocabularies of motives with roots in the property relations of society.  It is a way to get away from the dominant intellectual forces towards an “essentially” true explanation (136).  Burke also argue! s that it is the role of the creative artist to advance values antithetical to the dominant values of the time (137).  The dialectic does not end with synthesis but with the dialogue itself: “It is only the multiplicity of elements in the dialectic which offers an accurate account” (137).

 

Perspective by Incongruity: (138)

Burke’s dialectical technique, perspective by incongruity, is a method that “operates by bringing together terms and concepts which are normally never found together and which, in their ironic juxtaposition, undermine the ‘taken for granted’ character of the motivational forces of the terms in their conventional relations” (138). 

 

Three Steps to Motivational Analysis: (139)

1.  Identify the modal motivational framework, both its terms and the weighting of these terms on behalf of the ruling elites.

2.  Construct an ironic motivational terminology weighted in opposition to the interests of property.

3.  Offer this analysis in public discourse (leading to the possibility of actually changing the world, not just interpreting it).

 

Pentad: (140)

These are the five basic questions which must be asked when explaining human action. These questions are “collapsed” into the terms Act, Scene, Agent, Agency, Purpose – and the later addition, Attitude.  These terms brought into relationship with one another in pairs are called “ratios.”

 

Why Dramatism: (142)

For Burke, human action is “essentially” dramatic, and drama is his analytic model of the social world.  The dramatic analysis of action is “intended as a corrective to mechanistic perspectives and aggregate analysis,” and is an attempt to “rescue the human person, as a concept, from collapse into a conceptual universe suitable only for particles or organisms” (142). 

From: "david leitch" (dleitch@ucsd.edu)

 

Punchline: Dramatist analysis is at least as useful as other sociological theories in explaining the motivation of actions.

Note -- please read Noga's excellent summary of the first part of Overington before reading this; I don't define terms defined in there.

Four "terminological essences" that capture common ratios, of which the final is Burke's own, and it the most important in his analysis (143-144):
contextual definition (present context explains)
genetic definition (history explains)
directional definition (teleology - future explains)
dialectical essence (creation of clusters, around which groups of ideas interrelate through a god term)
I'm not sure if it's important that the first three definitions are essentially chronological - present, past, and future - while the fourth is not.

Dialectical essence is problematic, because apart from the pendad, Burke does not offer clear rules for defining the clusters. However, he suggests that synecdoche and free association help show us hidden complexes (note Freud resurfacing here after his last appearance in Luria). Therefore, "it is a 'figurative' or 'metaphorical' logic which underpins the connotational organization of terms in particular dialectical clusters." (146)

Overington examines Burke's analysis of anti-Semitism in pre-World War II Germany as an example of how Burke locates dialectical essences. Economic depression leads to frustration, which leads to feelings of persecution, which the mind compensates for by generating a feeling of (misused) goodness. This compensatory feeling is magnified and seeks an archetype, in this case Jesus. Identification includes taking on the hero's enemies as one's own. Therefore, since the Jews had Christ killed, they must, syllogistically, be the solution to economic depression. (147) This story in enriched by using Burke's pentad.

Using the pentad, we can see the economic depression as the scene which structures both the actors and the acts, leading to frustration with the constraints depression puts on acts. Once frustration sets in, it becomes the new scene, which is dialectically invokes feelings of power and goodness. The dialectic is necessary to understand the rise of anti-Semitism. The identification with Jesus takes place because of the nature of clusters: they are governed by a god term, and therefore contain pressures to abstract feelings into that god term, which in this case is the Christian (son of) God. (148) As an aside, I'm not sure how you could predict which terminological essence would rule at any given moment; why does depression lead to frustration through a correspondence of scene-actor and scene-act, but frustration leads to goodness through a dialectic? Is this analysis limited to post-hoc explanation?

Overington addresses this question on page 149, where he introduces this challenge through two linked questions dramatist analysis must answer:
How can we use this as a method and not a meta-method (can this be used by sociologists only)?
Are there limits to the stories that could be told from any set of analytic terms (is this falsifiable)?
I take these questions to be challenging the scientific status of dramatist analysis; I could be wrong, though. Thoughts?

Burke thinks that the dramatist is no worse off on either of these scores than other sociologists for two reasons. First, no sociological method has shown superiority to any other (?!). Second, dramatism has two criteria which Burke claims, "match the claimed reconstructive adequacy of well-formed and testable propositions,": identification of key analytic terms in the text (which is, after all, objective), and use of the pedatic rubric. (150)

These two criteria partially respond to the challenges on 149. Both the text itself and the pedatic rubric provide a limiting framework, which serves to limit the stories that analytic terms yield. Also, the culture in which the dramatist works provides a limiting framework which makes some explanations more and less plausible. Thus, a certain amount of falsifiability exists, and dramatism can be used as a method as well as a meta-method; while dramatism cannot provide itself with the cultural limiting framework, other branches of sociology can, allowing dramatism to work well enough as both method and meta-method. (151)

Thus, dramatism can be treated by sociologists as a legitimate (is this the right word?) tool for sociological analysis, so long as the cultural context is determined by non-dramatist means.

Kenneth Burke – Selections from A Grammar of Motives

From: "Robert Lecusay" (rlecusay@ucsd.edu)



(We forgot to assign someone to summarize this reading last week, so I'm doing it this week.)

I begin with some brief questions/comments. My summary follows.

I see parallels between Dramatism and Activity Theory, but I have some questions about these similarities that I’d like to get answered in order for me to be able to develop an idea of how the dramatic metaphor can be applied to everyday activity:

- For both Burke and Ilyenkov (via Davydov – see the reading for week 3) a theory of mind is central to an understanding of human behavior. Burke: “basic forms of thought . . . are exemplified in the attributing of motives.” This, I think, is similar to what Davydov is addressing in his piece from Week 3 when he brings in Ilyenkov’s idea of the ubiquity of “appealing” in human activity (in Davydov’s second and third theses). To copy and paste from Steve and Xavier:
“But people constantly appeal to one another, as Xavier quotes, "for true cooperation, for control over [their] ... individual actions, and for evaluation of those actions . . . "appealing" by individuals to others in collective activity presupposes an individual's awareness of the positions and potentialities of others, an awareness Davydov refers to as the "ideal plane," following Ilyenkov's theory of the ideal. Appealing also presupposes an individual's reflections on himself or herself and their own actions.”
- As with previous readings, Burke’s text brings up the question of the role of motivation in activity (or rather, in his case, the role of act, scene, agent, agency, & purpose in the development of motive). Is there a difference here in the way that the terms motive and motivation (Activity Theory) are used?
- Is a goal implied in Burke’s use of the term purpose?
- Burke, like Dewey (problems) and Engestrom (contradictions) sees ambiguity as source of transformation. (?)


Summary


Introduction: The Five Key Terms of Dramatism

“This book is concerned with the basic forms of thought which, in accordance with the nature of the world as all men necessarily experience it, are exemplified in the attributing of motives.”

Burke’s five terms of dramatism (the pentad) refer to the five elements that he considers necessary for any “any complete statement about motives”:

Act – What was done
Scene – When or where it was done
Agent – Who did it
Agency – How the agent did it
Purpose – Why

In discussing “a grammar of motives” Burke is referring to an examination of the “terms [of the pentad] alone, without reference to the ways in which their potentialities have been or can be utilized in actual statements about motives. Speaking broadly we could designate as ‘philosophies’ any statements in which these grammatical resources are specifically utilized.” (xvi) (Burke concludes this chapter by claiming that it is in the philosophic mode that “the subject of motivation is to be found.” )

To clarify Burke gives the example of scene. He notes that the term by itself stands for “any situation in which acts or agents are placed;” however, our understanding of the term becomes “grammatical” when we examine the term in usage. Emphasis then shifts to the multiplicity of perspectives (philosophies in Burke’s terms) that people bring to a situation. He gives the example of different terms that people might apply for the “ultimate ground or scene of human action”: God, nature, environment, history, means of production.

Burke goes on to discuss some of the history behind his project. His motive: to examine and develop a theory of a “drastically over-developed motive in the modern world,” – competitive ambition. Burke identifies three categories of “devices” that individuals employ in “outwitting and cajoling” one another: Rhetoric (basic strategies of persuasion, e.g. parliamentary and diplomatic devices, editorial bias, sales methods, etc.), Symbolic (“modes of appeal in the fine arts,”), and Grammar (“formal considerations logically prior to the rhetorical and psychological,” e.g. theological, metaphysical, juridical doctrines)

Burke continues with what I see as a connection with Dewey (problems) and Engestrom (contradictions) – the notion of ambiguity as source of transformation:

“We take it for granted that, insofar as men cannot themselves create the universe, there must remain something essentially enigmatic about the problem of motives, and that this underlying enigma will manifest itself in inevitable ambiguities and inconsistencies among the terms for motives. Accordingly, what we want is not terms that avoid ambiguity, but terms that clearly reveal the strategic spots at which ambiguities necessarily arise.” (xix)

Related to this ambiguity is the fact that the five terms of the pentad (in their use) can be applied to a single concept. For example, depending on how one applies the term, War can be an Agency (if it is treated as a means to an end), as a Purpose (in schemes proclaiming the cult of war), as a collective Act, or as a Scene (as it would be for the enlisted soldier).

Burke compares the elements of his pentand to fingers on the hand – distinct at the edges, but collapsed together at the palm. To move from one element to the next one has to first travel through the palm.


Chpt. 1: Container and Thing Contained


The Scene-Act Ratio
In this section of the chapter Burke discusses the relationship between scene and act, examining what he describes as “the principle whereby the scene is a fit “container” of the act, expressing in fixed properties the same quality that the action expresses in terms of development.”

He introduces several examples in which aspects of the scene are reflective of character development in the protagonists (e.g. the state of mind of a character is objectified through such scenic elements as torn clothes, broken windows and a disorderly study). Through these examples Burke develops his argument that the “from the motivational point of view, there is implicit in the quality of the scene the quality of the action.” In other words, Burke argues, the quality of action can be inferred from the quality of the scene.

The examples Burke provides neatly fit his argument about the relationship between scene and act; however, he admits, this relationship is “’obscured” in situations in which the interactions among characters in a scene may be considered as “scenic conditions or “environment, of one another.” Furthermore, Burke continues, any act can be viewed as the context that “to a degree motivates” subsequent acts.

The Scene-Agent Ratio


If I’m interpreting Burke correctly here, the scene-agent ratio is analogous to the scene-act ratio: qualities of the scene can be used to deduce qualities of the character. The coupling between the two is tight. A narrow conception of “scene as the motive force”
leads to an equally narrow restricting of the personality of the characters in the scene.

Further Instances of these Ratios


In this section Burke discusses the mutually constraining relationship among scene-act-agent: scene-act ratio calls for scenes in keeping with acts, or acts in keeping with scenes, and the same can be said about the scene-agent ratio. To highlight this, he returns to one of his earlier examples, O’Neills Morning Becomes Electra. He describes one scene in which a character orders another to close some shutters, which becomes a reflection of this character’s increasingly shut in personality. The transformation of the scene, Burke argues, constrains future action in the rest of the play.

Ubiquity of the Ratios


Here Burke provides a series of examples of scene-act and scene-agent ratios that appear in a variety of philosophical discourses. He translates the language of Marx, Machiavelli, and Jesus Christ into the code of his pentad:

- Machiavelli: people read history “as though heaven, the sun, the elements, and men had changed the order of their motions and power, and were different from what they were in ancient times.” = Burke: people read history, “ as though human agents and both the supernatural and the natural scenes have changed, with a corresponding change in the nature of motives.”
- Marx: “Justice can never rise superior to the economic conditions of society and the cultural development conditioned by them.” Burke cites this quote as expressing his contention that the scene constrains the act, that the quality of justice is limited by the properties of the scene (for the dialectical materialist this the required scene is a high stage of industrial development.
- Jesus: “I am the way” = Burke: “I represent a system, or synthesis, of the right acts.”

 

 

 

 

 

Week 6

 

Vygotsky, Mead, and the New Sociocultural Studies of Identity
Dorothy Holland and William Lachicotte, Jr.


From: "joanne price" (jprice@activmanuals.com)



In this article, Holland and Lachicotte contrast and compare the theoretical perspectives of identity proposed by Erik Erikson and George Herbert Mead. The authors suggest that current sociocultural research is developing a perspective on identity that integrates these two perspectives.

Erikson Perspective
Although there was a history of literature and research on self and personality, the concept of identity is relatively new, introduced in the 1950s by Erik Erikson. Questions such as “Who am I” and “Where do I belong in today’s society” were associated with this perspective of identity. The focus was on identity development in the individual. “Questions of belonging and of locating oneself in society continue to be core aspects of the concept.”

Mead Perspective
Over time other concepts of identity began to emerge that were more reflective of topics within sociology and anthropology. The best known of these concepts, proposed by George Herbert Mead, highlighted the importance of ‘social types’ and individuals’ development of multiple identities, or ‘senses of themselves.’ According to this perspective of identity, individuals ‘inhabit roles, positions and cultural imaginaries that matter to them, e.g., as a skater, a punk, a radical environmentalist, a theoretically sophisticated anthropologist, a stylish dresser, a good father…” [p. 2]

The authors note that the theoretical school which became known as symbolic interactionism in the 1940s initially used the term ‘self’ and then in the 1960s shift usage of the term ‘self’ to ‘identity.’

Conceptualizing Identity Formation: Mead and Vygotsky

Vygotsky, like Mead, viewed the development of concepts of ‘self’ (identity) as inherently a socially, mutually-constitutive process. Whereas Mead’s focus was on the outcome of this formation, Vygotsky’s focus was on the development process.

Holland and Lachicotte describe three similarities of the perspectives of Mead and Vygotsky:

1. Active Internalization.
This concept, influenced by the writings of psychologist James Mark Baldwin and philosopher Josiah Royce, describes how individuals first imitate the behavior of others, then compare their subsequent behavior to that model, and finally how they internalize a behavior pattern somewhere between the model and the early imitation.

2. Dialogic Selves: Self Authoring in Relation to Others.
Part of the process of active internalization includes comparison of one’s behavior with reactions by others to that behavior. Both Mead and Vygotsky agree with to this viewpoint. Mead’s focus, however, was on how this process influenced the individual’s selection of positions and roles in society. Vygotsky’s focus was more on this process as it affected cognitive and affective development.

3. The Semiotics of Behavior: The Signs of the Other, Signs of the Self
This concept, shared by both Mead and Vygotsky, is that individuals use signs that were initially either directed to others or received them to direct their own behavior.

According to this article, much of the newer literature regarding sociocultural research incorporates ‘Mead-type’ notions of identity – the individual constructs various identities based on various ‘social types’ available from the cultural environment and that these identities provide motivation for action. Further, the integration of these various identities by the individual tends to point back to the Erikson notion of identity.

Reflection on the Reading

This piece made me consider the ways in which the roles and models that comprise the practice of various communities of practice influence the behavior of the individual as well as how the active internalization process by the individual influences the role or model held by that community. A summarizing quote from the article in this regard is:

“The concepts Mead and Vygotsky share—active internalization (self authoring), dialogic selves (self-other dialogues), and the semiotics of behavior, coupled with Vygotsky’s notions of semiotic mediation, higher psychological functions, and agency, constitute a powerful sociogenetic vision of how individuals come to be inhabited by, and yet co-construct, the social and cultural worlds through which they exist.”

Lotman, “The Semiosphere”

 

From: "joanne price" (jprice@activmanuals.com)

The Semiosphere, Part 1


In this article, first part of this essay, Lotman sets the context and presents a metaphor that will be expanded and refined throughout the essay.

First, Lotman presents a ‘historical fly-over at 30,000 feet’ to provide the reader a quick description of semiotic research terrain. “It is generally accepted that two scientific traditions underlie semiotics.” One tradition has its roots in the work of Pierce and Morris. In this tradition, ‘sign’ is taken as the primary element of the semiotic system. An isolated sign becomes the focus of the ‘semiotic laboratory microscope,’ albeit still in its natural environment. All surrounding signs are then compared and contrasted to this sign, in an attempt to understand the whole semiotic structure.
The second tradition is found in the work of Saussure and the Prague school. In this tradition, the exchange of a message between the addressant and the addressee was the primary element of focus.
In both of these approaches, Lotman notes, a piece (an ‘atomic element’) of the semiotic environment is taken as the primary unit of analysis. In short, “a complex object is reduced to the sum of simple objects.” (p. 42)
Lotman describes the problem of attempting to construct a complex entity by assembling the part of it: “Just as we cannot obtain a calf by gluing together veal cutlets, but can obtain veal cutlets by cutting up a calf, so we do not obtain a semiotic universe by summing up particular semiotic acts. On the contrary, only the existence of such a universe, the semiosphere, makes each symbolic act a reality.” (p. 44)
[As an aside here, I believe Lotman agree that the veal cutlets, taken from the whole, are no longer the calf. Thus studying the parts in isolation always has the problem that you are no longer studying the ‘calf.’ It occurs to me that this is precisely why you must study actions and goals as they are part of an activity system and why the activity system is the primary element of analysis. In the Lotman essay, the semiosphere is the primary element of analysis, a unit which is comprised of many interrelated parts and having various attributes, but which cannot be deconstructed without destroying the object of investigation.]
Lotman compares and contrasts the term ‘semiophere’ with Vernadskii’s use of the terms ‘biosphere’ and ‘noosphere.’ The biosphere is “situated on the surface of our planet and comprises the totality of living matter; it processes the radiant energy of the sun into chemical and physical energy, which in turn is directed toward reprocessing the ‘inert,’ nonliving material of our planet. The noosphere is formed when human reason acquires a dominant role in this process.” (p. 43) Using this foundation, Lotman then describes the semiosphere as the totality of an individual’s semiotic world.
Lotman notes the two primary characteristics of the semiosphere:
1. It is bounded.
2. Semiotic unevenness.

Reflections

Some drawings early in this article would have provided a quick point of departure for subsequent discussions. Lotman draws heavily on the metaphor of a cell. Drawings seemed to be implied (or at least they were inferred) and could have been ‘translating artifacts’ for integration into our personal semiospheres. (I will bring some of the drawings suggested by his essay to class.)

 

From: "noga shemer" (nnsevilla@yahoo.com)

 

In the second half of this essay, Lotman builds an argument that the manifestation of righthandedness and lefthandedness is the basis of dialogue, the foundation of all meaning-generating processes (60).  This conclusion is based on a series of characterizations of dialogue, including:

1.  Since a dialogue consists in mutuality and reciprocity in the exchange of information, it is possible to interrupt the transmission of information – therefore dialogue is “discrete.” (52)  The ability to perceive this discreteness depends on one’s vantage point – processes of development often appear continuous from an immanent standpoint.

2.  “The text to be conveyed should, in anticipation of a response, contain in itself elements for transition into the alien language.  Otherwise, dialogue is impossible” (53). 

 

Lotman writes that consciousness is the exchange of messages, and consciousness without communication is not possible (54). 

 

Substructures of the semiosphere interact and only work with mutual support.  These dynamic interrelations form the behavior of the semiosphere.  All of these communicative processes are based upon one invariant principle: symmetry vs. assymetry, “the bisection of some unity by a plane of symmetry as a result of which mirror-image structures are formed – the source of subsequent growth in diversity and functional specification” (54).  The diversity and similarity created by mirror symmetry (enanthiomorphism) enable dialogic relations to be constructed:  “the systems are not identical and produce different texts, but…they are easily converted one into the other, making texts mutually translatable.” Lotman elaborates on the example of reading palindromes to demonstrate how the mechanisms of text formation and consciousness change in the process (56-58).  He argues that the mirror-image mechanism is universal for phenomena defined by the term “text” (58) and these pairs of symmetry-assymetry generate meaning.  Examples include paralled plots, diagonal axes in paintings, and globalization and localization.  Lotman concludes: “Since all levels of the semiosphere, from the human personality or an individual text to global semiotic units, are semiospheres that have invested in one another, so to speak, each of them is a participant in a dialogue (part of a semiosphere) a! nd in the space of a dialogue (the entire semiosphere) at one ! and the same time, and each displays the property of being left or right and contains right-handed and left-handed structures at a lower level” (60).

 

From: "Mike Cole" (mcole@ucsd.edu)

Seems like here is a case where we badly need part one because otherwise Lotman sounds like a kook and he was definitely not.
mike

From: "Matt Brown" (mjb001@ucsd.edu)

You bring up this great quote:

"Just as we cannot obtain a calf by gluing together veal cutlets, but can obtain veal cutlets by cutting up a calf, so we do not obtain a semiotic universe by summing up particular semiotic acts. On the contrary, only the existence of such a universe, the semiosphere, makes each symbolic act a reality." (p. 44)

Mike pointed out in class the relationship between this and that bit of Faust that he quotes in Cultural Psychology, and also with Dewey's analysis of the reflex-arc and stimulus-response psychology. I think this is a great statement of an important element of Dewey's methodology. Dewey is always insistent on the fact that you can analyze things into parts, but one cannot ever build the whole up out of the parts. This is one of the problems, for Dewey, with dualism, with materialism and reductionism, with realist epistemology, etc.

From: "Mary Bryson" (mary.bryson@ubc.ca)

“Just as we cannot obtain a calf by gluing together veal cutlets, but can obtain veal cutlets by cutting up a calf”


Ok, so this really has to be commented on, even though I am already late running in to work... Actually, as I read it, neither is usefully read literally.

It takes a much more complex set of cultural choices than “cutting up a calf” to “obtain veal cutlets”. And I am not pointing this out in some fevered moment of analytical paternalism. It is a very interesting example of what gets left out — to follow the trail of folks like Bruno Latour and Susan Leigh Star...

A cultural historical analysis of the “veal cutlet” would reveal hundreds of important choices about what is valued --- very white meat
How to obtain it ---- not going to go into details here, as you all likely know the infelicitous story

And so it goes....

To obtain versus to produce, invent, willfully impose... These are worlds apart, and importantly so. There is likely no whole, only holes.

 

First half of Wertsch Chapter Three

David Leitch

First Half

Puncline: Whereas Vygotsky never fully specifies how particular cultural institutions shape particular forms of action, the work of Mikhail Bahktin offers translinguistics as a way of making just that specification

Wertsch begins this chapter with a contention that Vygotsky has failed in what should have been his central goal, developing stories of precisely how specific cultural institution operate through mediated action. To have done less than this, Wertsch contends, is an especial failure for a Marxist. Vygotsky scholars have usually attributed this to Vygotsky's early death. But little in Vygotsky's later research shows him beginning to head in that direction, with the exception of his research on schoolhouse dynamics.

Therefore, in order to understand mediation and internalization at a more-than-general level, we must turn away from Vygotsky and towards other (Soviet/Russian) writers. Wertsch presents Mikhail Bakhtin as a relevant author in understanding the link between setting and mental functioning.

There is some dispute about which pieces should be included in a study on Bakhtin; living in the Soviet Union under Stalin, he was arrested in 1929 and likely wrote under two pseudonyms: Voloshinov and Medvedev. Wertsch takes the position that Voloshinov and Medvedev were both committees in which Bakhtin was heavily involved, and that we can therefore include their works in an examination of Bakhtin himself.

Bakhtin's unit of analysis was the utterance, an unusual one from the standpoint of standard linguistic theory, because of its lack of abstraction from everyday use. This was the advantage of it for Bakhtin; it could connect the fixed meanings and constancy of linguistic systems to their use in a wide variety of contexts. Thus, it allowed Bakhtin to study, "the specific object of linguistics, something arrived at through a completely legitimate and necessary abstraction from various aspects of the concrete life of the word." (Bakhtin 181 quoted on Wertsch 50) Bakhtin's reliance on utterance so took him out of traditional linguistics that he termed his approach 'translinguistics'.

There are two central categories in translinguistics: voice and dialogicity. Dialogicity arises from Bakhtin's technical definition of voice and the way in which it links the individual to their social context.

Voice is not just the sounds of speech, but rather the idiosyncratic tone of speech from which utterances arise. Utterances, of course, are not unrelated to the world around them. Instead, utterances are based in a relationship between the speaker's point of view and the social context they find themselves in. Thus, voice partially encapsulates both the individual and the social context (making it quite useful for Bakhtin's project and Wertsch's criticism of Vygotsky). Voice partially encapsulates social context because it is only in that social context that a given point of view voices a particular utterance. This social context is composed of other points of view voicing other utterances. Thus, for Bakhtin, voice always implies an audience, because it is addressive.

This leads him to the other category of translinguistics, dialogicity (or, rather, dialogicity leads to voice -- Wertsch reverse the presentation because of our Western Cartesian tradition, in which that which appears most individual, voice, is prior). Since all voices find themselves (?) in addressed utterances, these voices are in dialogue with one another, and have to be understood as coming out of a dialogue. Dialogue is a broad category, as Wertsch admits, encompassing face-to-face conversation, inner speech, and parody.

Wertsch uses parody to show how dialogicity operates. A parent in the mid 1970s in America might tell their child that earlier statements regarding bedtime were 'no longer operative' as a parody of Ronald Ziegler's statements during the Watergate scandal. This works as a parody, because the parents are taking a term given a significant amount of cultural weight by the visibility and importance of Ziegler's statements, and giving it a different accent through their own voice. Thus, they speak with multiple voices: Ziegler's and their own. This is an example of dialogicity that serves to illustrate how a particular cultural institution (the fall of the Nixon administration) affects particular forms of mental action and mediation.

 Wertsch Ch. 4, Part 2

 

From: "David Mather" (dmather@ucsd.edu)

: Overview of 4 aspects of Bakhtin's approach to meaning:

1. reject disengaged, atomistic self;

2. recognize dialogical function;

3. recognize authority in text;

4. reject concept of literal meaning.

Authority and Text: Bakhtin's view of authoritative discourse is linked to univocal text, and this view is that only certain forms of speech are inflexible, such that the direction of transmission is in one direction (speaker to listener). However, the alternative is Bakhtin's concept of internally persuasive discourse permits speech acts to be analyzed within a dynamic of sociocultural interaction. These two different types of speech correspond with Lotman's concept of functional dualism.

Reported speech is used to illustrate dialogicality at work. Specifically, in direct discourse the speaker's words retain integrity, authenticity and intonation, while in indirect discourse the speaker's words are reported by someone else, which changes their meaning. The reported speech uses analytical categories to convey the meaning of the original speech act, but with a loss of meaning. Also, the reporting of speech eventually infiltrates the speech act through summary and deletion. Reported speech is an example of how multiple voices interanimate each other in speech.

Literal meaning: "The assumption that there is such a thing as the strict and literal meaning of an expression turns out to be an ethnocentric assumption" (Taylor quote). Bakhtin's approach rules out literal meaning as an a priori framework. In place of literalness, Bakhtin uses social languages (speech genres) to account for how meaning can change according to the sociocultural conditions of an utterance.

Illustration and Summary: This example of microgenetic transition in speech of 2.5 year old and her mother illustrates Bakhtin's dialogicality of discourse and also relates to Vygotsky's internalization of speech. There are tree episodes of child-mother conversation about solving a puzzle progress: from multiple guiding questions, to a single guiding question, to the child not needing a guiding question to determine solution. The progress shows internalization of speech, but it is the mother's speech which the child internalizes; the nonverbal communication confirms that the child is in dialogical relation with the mother even when the mother is not verbalizing the guidance. The problem for existing theories of meaning is they cannot address the different degrees of dialogicality in speech acts, as in this example.

The summary restates the main issues of dialogicalization. First, one cannot begin with a concept of the disengaged, atomized self. Second, there is a functional dualism of texts that permits interaction among languages. Third, dialogicality can be internalized, as 'hidden dialogicality', and this links authority (univocality) and internal persuasiveness (multivocality). Fourth, the concept of literal meaning cannot account for microgenetic transitions.

Summary

Wertsch – Voices of the Mind, Chs 3 & 4

 

Wertsch sets forth his agenda for Voices of the Mind in Chapter 1. He notes that attempts at understanding higher mental reasoning vary depending on the focus of interest. The focus of western contemporary psychology has been on universals— universals that transcend time (are ahistorical) and space (are applicable regardless of the social context):

“Many psychologists have concerned themselves with the universals of mental functioning, and this emphasis on mental processes which are assumed to be ahistorical and universal, has dominated research in contemporary western psychology.” (p. 7)

In contrast, Wertsch focuses on what is ‘socioculturally specific.’ He describes the ideas of L. S. Vygotsky and M. M. Bakhtin as particularly important in such an approach.

“The approach I am proposing is indebted to the efforts of many theorists, but I have already mentioned the two that are of particular importance: L. S. Vygotsky (1896-1934) and M. M. Bakhtin (1895-1975).”  (p. 16)

In Chapters 1 and 2, Wertsch describes the sociocultural approach to understanding higher mental functioning as theorized by Vygotsky. In Chapters 3 and 4, Wertsch then introduces the ideas of Bakhtin as an important extension of Vygotsky’s theories.

If Vygotsky provides the skeleton for a sociocultural approach to understanding ideas, then Bakhtin provides the flesh for at least one member of this category. Whereas Vygotsky underscores the concept of mediated action, Bakhtin provides a perspective on the pathways of such mediation.

Wertsch considers Bakhtin’s concept of ‘utterance’ an essential point of departure from traditional linguistic analyses that investigate language form and meaning abstracted from the actual circumstances in which they occurred. Bakhtin’s focus is on language-in-context, both individual and social setting. To make sense of meaning, according to Bakhtin, it is essential to focus on utterances, the things people say in everyday life (as well their inner speech):

“Speech can exist in reality only in the form of concrete utterances of individual speaking people, speech subjects. Speech is always cast in the form of an utterance belonging to a particular speaking subject, and outside this form it cannot exist.” (p. 50)

Utterances have embedded within them both the individual and the community, the social and the cultural, the present and the past. Analyses of meaning therefore must consider these contexts. It follows then that Bakhtin proposes various tools by which these utterances may be analyzed and meaning ‘unearthed.’ These tools include voice, social language, genre, and dialogue. The two essential questions that Bakhtin then holds before us in all analyses of meaning are (1) Who is doing the talking?, and (2) Who owns the meaning?

 

 

 

 

 

 

 

 

 

 

 

 

 

Week 7: Dialogic Approaches to Mediation

 

R. Engestrom, “Voice as communicative action”

 

From: "David Mather" (dmather@ucsd.edu)

Responding to Ritva Engestrom's article ("Voice as communicative action"), I am interested in pointing out one section where she describes disturbances in utterances, because I think it has interest for understanding the interrelation among actions and operations. The specific paragraph is on page 203, and it references her research on conversations within a clinical medical setting:

"My research focuses on troublesome moves and innovative improvisations in the consultations. These are studied through a search for mismatches (discoordinations) of voices in the sequences, for ruptures of conversations, for innovations in interaction, and for exceptions and deviations from the expected voices. Such deviations or 'unidentifiable voices' are particularly considered because they represent locally produced potentialities of change, 'buds' and 'shoots' of emerging novelty in referentiality. Such anticipatory 'innovative' forms often take shapes through disturbances and ruptures in discourse."

Also, I should note that in her illustration of how activity levels (activity, action, operation) correspond with Bakhtin's speech categories (social language, voices, speech utterances).

Using the description of disturbance within conversation, she shows how the thematic action of the participants shifts in time, and the cues for these shifts are indicated (or even possibly generated) by fluctuations in vocal patterns. This is among the only examples we have read this quarter where the dynamic relations between actions (and their 'drift') can be correlated with operations (and their fluctuations). Bakhtin's concept of multivoicedness matches this account well; we can see how certain voices as representatives of actions can assert themselves into an ongoing conversation, thus 'de-stabilizing' the flow and instigating a productive re-framing of meaning. The surface manifestation of spoken language, especially the seeming 'irrationality', can become clearer when mediational objects (such as visual cues and objects) are factored into the flow and also when one accepts that voice 'incursions' are an organic part of conversational development.

We also tried to correlate Engestrom's conceptual approach with Wertsch (Ch. 5 and 6), however Prof. Cole pointed out that Wertsch's emphasis is on how action relate to activities, but R. Engestrom seems to be most interested in the relationship between actions and operations. Perhaps a more explicit connection to Wertsch's work could be beneficial addition to the argument.

From: gordon wells

In this week’s readings I am particularly interested in Ritva Engeström’s way of relating the organization of discourse to the three strata of activity proposed by Leont’ev. I agree with her that Bakhtin’s ideas about dialogue and speech genres is very relevant, but have some problems with the way she shows the relations of realization in Figure 1 (p.200). Basically my problem is that, here, discourse is treated as an activity in its own right:the level of activity is realized as “social languages” and their “referentially semantic content”.
On the face of it, it may seem that certain episodes of discourse, such as doctor-patient consultations or teacher-class discussions, are activities – or at least actions within activities such as organized health care or education. But this implies that the goal of such an action of this kind is a purely discursive one (“constructing meaning”) when, it seems to me, the goal is to transform some aspect of the world beyond the discourse.
This way of looking at things is much easier to grasp in situations where the discourse is clearly “ancillary” (Halliday. 1984) to the action in progress, such as purchasing goods in a family store. Here the goal is “exchange of goods and services” (Halliday), as contrasted with “exchange of information” in what Halliday calls “substantive discourse” (such as the doctor-patient discourse Ritva analyzed). A considerable amount of work has been done within Systemic Functional Linguistics in analyzing the organization of ancillary discourse, notably by Hasan (1989) and some of her colleagues/students.
The analysis is conducted in terms of “genres”, which are defined as “staged, goal-oriented social processes” by Martin and Christie and further characterized as follows:
* they are recognised communicative events with a purpose that is understood by people belonging to the discourse community in which they happen;
* they usually have names that are recognised by at least some of the members of that community;
* they have some predictable structures and conventions - both in terms of their form and also in terms of their lexico-grammatical features;
* established members of a community will have a greater knowledge and understanding of its genres than new members, outsiders or apprentices;
* expert members of the community often exploit generic resources within the constructs of "socially recognised communicative purposes" (this idea is developed by Bhatia, 2004);
* genres are reflections of a community's culture, and in that sense focus on social actions embedded within its practices;
• they are a form of situated cognition embedded in a community's culture.
• (http://btrayner.info/Genre/criticalgenrediscourse.html)

In Halliday & Hasan (1989), Hasan describes the genre of vegetable shopping, setting out the essential stages, SR (sale request), SC (sale compliance), S (sale), P (purchase), PC (purchase closure). These are the sub-actions of this particular genre, as it regulates this sort of shopping in Australia. Each sub-action is achieved interactionally, in what I would call situationally-constrained operations, most of which are realized through linguistic utterances, often accompanied or substituted by non-verbal acts.
Based on this analytic approach, we might set up an alternative to Ritva’s Figure 1, as follows.

Activity Action Operation
Life-sustaining
Shopping
Realized through
Shopping Genre
Required Stages
Realized through Exchanges in language
& N-V acts

[In passing, I would point out the difficulty of deciding which stratum in Leont’ev’s model should be invoked for particular events. For example, is each stage in the shopping genre better seen as a sub-action within the action of shopping, or as a step in the operationalization of the shopping (genre)? Or is the use of the shopping genre itself an operationalization of the action of shopping on a particular occasion? My own feeling is that these decisions need to be made in light of the focus of the intended analysis.]
Can this approach be pursued into settings where the commodity exchanged is information rather than goods and services? In our ISCAR paper (Hiruma, Wells & Ball), we argued that it could. The core of our argument was that all discourse mediates some action in the world: with ancillary discourse, the discourse mediates material action, but in substantive discourse the discourse mediates goals of a different kind, such as solving an intellectual problem, making plans, reviewing an event, etc. These are also actions that realize different activities; they also have sub-actions, such as proposing alternatives, reaching a decision, etc. which are comparable to the stages in a genre such as shopping. And, in turn, each sub-action is operationalized through utterances.
In our view, the merit of this proposal is that it adopts the same fundamental approach to all forms of discourse and recognizes that all forms mediate actions beyond the discourse. At the same time, it leaves many questions unresolved. For example, how should the various aspects of the artifact – a particular human language – (social language, registers (technical v. everyday), grammatical structure, referential word meanings, “sense” (Vygotsky), etc.) be conceptualized within this scheme?
And what about other goals that participants may be trying to achieve simultaneously, such as enforcing status/power differentials, expressing an evaluative attitude to the sub-action, genre, or other participant(s)? Ther is clearly much work still to be done!!
Your reactions to this proposal would be very welcome.

Ritva Engestrom

Dear Dialogic class!

thank you for getting possibility to comment your discussion on my MCA article. I am still working on Bakhtinian approach so that I do not need to recall something from my past. And I still see it as promising new challenges in facing more complex and hybrid activities.

I try to be very short in my comments:
1) Wertsch's work originally inspired me to focus on genres and social languages and their conceptual definitions. Their relations were considered unclear in Bakhtin's texts and the question was left open by Wertsch, as I've understood. I got interested in reflecting them from the point of view of the
object (referentiality – according to B) and in pointing to the significance of voices in the framework of Leontiev's conceptual schema. The schema channels the interest into
the transactions of the levels. Later, I have found important backup from Markova's (2000) comments that social change in language is extremely difficult to conceptualize because the mainstream conception of cognition and language is based largely on various kinds
of individualistic or collectivist, but static, epistemologies. Pointing to the significance of actions (the voices of the speakers) and following them in the analysis, it was shown that a dialogic approach calls for open structures of social languages
where their intersections, conflicting structures, and horizons of new potentialities are considered as a constitutive part of activity. I like a lot the expression of voice 'incursions' (message). In my analysis, the investigator had to decide empirically where to draw the line between medical and everyday language in the analysis of patients' voices. The data
spoke back (the voices) and one crucial finding was to show that patients made sense out of their current problems within the framework of prior understandings of their
bodily experiences as articulated through medicine. Medicine was not simply borrowed by the patients but rather it was reworked with experiences of the significance of these judgments in an unique biological and biographical context and accommodated to the circumstances of daily life. The voices
intersecting everyday and medical languages on the somatic revealed the initial categorization itself as a problematic fabrication of social sciences, and in deeper
sense the epistemological distinction
between 'objective' and 'subjective' knowledge in clinical problem solving.

2) From gordon wells:
In the Fig 1 the level of activity is realized as "social languages and their referentially semantic content". The goal of action is purely discursive one (constructing meaning) instead of the goal to transform some aspect of the world beyond the discourse.

I agree with Gordon that CHAT can provide a lot from the perspective of going beyond
the 'pure' discourse. And that's the reason I have turned to Bakhtin from the perspective of CHAT. In "constructing meaning", the starting point is that meaning is thoroughly contextual. Thus, "referentially semantic content" cannot be separated from the purpose of language use. The approach wants to put linguistic forms into the context of "carried out by the speaker" as well as carried out "for the immediate purposes of speaking"
(Volosinov 1973, 67). The Fig 2 tries to sketch the context in dynamic relations conceptually.
From this follows in my analysis on doctor- patient discourse that the goal or the
activity is not "exchange of information" ("substantive discourse") but to act on patient problem and to make clinical judgments. My analysis focuses on the interactive process of transforming a problem into a solvable problem. A problem is solvable when a doctor proposes a disposal: a limited set of actions which she or he perceives to be a sufficient answer at this time and place to a specific patient problem. It is very practical ( compare to purely discursive) how the patient will leave the consultation room: with a referral to tests, a medical prescription, or with an advice to
wait and see. I think that it is one core point of CHAT that the problem you start to deal with in the consultation is not independent on the tools by which you make the problem solvable (construct the object of consultation).

If you have identified the activity you can approach it historically. This is also an
important methodological point of CHAT. In doing clinical judgments, interaction between
doctor and patient is a central part of the history – interaction is not considered
out of history and universal. That's one significant point I have learned from Bakhtin's texts. As an empirical outcome of voice analysis, I found that the clinical
interview (the main tool to carry out
the conversation between doctor and patient) has its roots in biomedicine although it seems to be 'natural' and 'objective'. It is created in the rising time of biomedicine and from the perspective of certain historical notion of 'disease'. The mismatches of voices, ruptures of conversation etc. made
visible the historical constraints of this tool regarding the historically new object of
consultation. It was also shown that the biopsychosocial model, although it aims to correct the "anatomo-clinical gaze" of the doctor, views "the mind" from the perspective
from biomedicine. And so on...
My question to Gordon: is there in an alternative approach (to Fig 1)counterpart of "subjectivity of speaker" or is it needed?

Markova, I. (2000) Amedee or how to get rid of it: Social representations from
a dialogical perspective. Culture & Psychology 6 (4), 419–460.

my best wishes
Ritva

From: gordon wells

Ritva,

You wrote:
I think that it is one core point of CHAT that the problem you start to deal with in the consultation is not independent of the tools by which you make the problem solvable (construct the object of consultation).

I agree with you on this. But my point is that the discourse is one of the "tools" that is used to solve the problem. And so it seems better to see its use as an operation, together with the other operational use of tools, such as the computer, that are also involved.

Gordon

 

From: "Ana Marjanovic-Shane" (anamshane@speakeasy.net)

Gordon,
I think that discourse as a "tool" is quite different from a computer as
a "tool". Discourse is an activity, it does not "exist" independent of
the particular activity, it is shaped by a particular language which is
a product of centuries of spontaneous development, (i.e. not deliberate
creation of particular "application" like modules which are the result
of the very careful analyses by the hardware/software developers).
I think that the "tool" concept when it comes to symbolic tools has to
be very carefully revisited. Symbolic systems are much more dynamic,
much more spontaneous, much more democratic in the division of labor
(who gets to do what in a symbol system).
Ana

From: "Ana Marjanovic-Shane" (anamshane@speakeasy.net)

 

Yes, discourse functions differently from a material tool.
Nevertheless, the activity to which it pertains is mediated by the
discourse, as it is also mediated by the medical records held in the
computer. As to discourse being a product of centuries of
development, that no doubt applies to most if not all
sign systems, as well as to many material tools, such as ploughs,
wheeled vehicles, material for writing on and with, etc.

The critical question is: what is mediated? Isn't it the actions that
realize activities? My proposal is that there is always an action
goal - an object in view - that recruits discourse as a mediational
means.

Gordon

From: "Ana Marjanovic-Shane" (anamshane@speakeasy.net)

Yes, you are right -- they are all mediators. But I think they "do" the
job of mediation using different processes -- I mean signs and material
tools. All I mean is that discourse as a mediational means is also an
activity which is different than an object tool like for instance a
pencil or a wheel...
Or maybe I am wrong. It seems to me that discourse is much more dynamic
and fluid, being shaped itself in the moment it is used, while material
tools are slower, stabler, much more definite...
Maybe we are just having a different pace of the same processes?
What do you think?
Ana

 

Chpt. 4 Middleton and Brown: Virtualising Experience: Henri Bergson on Memory

 

From: "Robert Lecusay" (rlecusay@ucsd.edu)



This chapter offers a re-evaluation of the thought of the French philosopher Henri Bergson. The focus here is on Bergson’s concept of memory.

Reality as Process
Middleton and Brown begin by introducing Bergson’s “process philosophy” in contrast to traditional Western analytic philosophy, which they describe as concerned primarily with things rather than with actions and events. A commitment to this latter philosophy implies an understanding of memory as “traces . . . encoded” in multiple brain structures, of memory as neural structure.
Alternatives to this “substance” centered philosophy fall short. These alternatives can bee seen as ultimately describing processes that occur among mental substances, bringing us back to the task of having to address the study of mind as the search for where these “mental things” are stored, their true nature, and the process of how these things “enter” into the mind.
What then is reality for the process philosopher (Bergson specifically)? Becoming. “For the process philosopher,” argue Middleton and Brown, “all ‘things’ are actually part of the process of becoming – that is, changing, moving, and transforming – so what is real is change itself,” (61). However, Bergson would argue, it is not our perception of change that seems real to us, but rather our perception of snapshots of this change. Form and qualities represent a “snapshot view of transition,” and our perception of these qualities depends on our particular perspective, on our particular intellect which is a reflection of our biases.

The Time of Living: Duration
If we accept the notion of change as reality, we must consider the hypothetical of a world in perpetual change. In other words, a world where it is impossible to recognize any regularities in this change. Bergson responds with notion of duration. “Duration is the experience of time passing;” Bergson “insists . . . that it be considered as an indivisible, continuous flow of conscious states.” This flow which Bergson describes as “all that is naturally experienced,” stands in contrast with the snapshots of transition – the “things and states,” that are “treated . . . as products or outcomes arrived at by artificially cutting into the flow,” (62-63).
Bergson argues that part of what makes it difficult for us to conceptualize existence in these terms is the fact that our language is not suitable for talking about duration. It favors “substantives over relationships, nouns over prepositions.” The contrast here is between language as a tool for communicative action vs. language, traditionally viewed, as a neutral medium for representing experience. With respect to this latter form of representation, Bergson is interested in particular in how language reflects “a general human tendency to think of the world as an empty homogenous medium in which entities occupy unique spatial positions.” This tendency to divide the world up into parts is derived from our practical experience. Consequently, “our intellect and the language in which it is expressed have an entirely practical bent that is modeled on the use we make of objects (1998, ix): ‘our logic is, pre-eminently, the logic of solids.’” This reflects a central tension for Ber! gson, that between our experience of space and time.
The experience of space implies an experience of objects in space which for Bergson represents an obstacle in the path to gaining “purchase on the fluid continuity of duration.” At this point in the reading the terms “pure” duration and “pure” space are introduced; however, no clear definition is given of either. Instead we are told that “at the practical level,” both experiences are collapsed together. The commingling of the two is referred to by Bergson as “endosmosis.” It is just before the close of this section that we are given something of a definition of the “pure”: “The crucial point Bergson makes is that we simply will not adequately understand this mixing of experience . . . unless we have first understood the nature of ‘pure duration’ and ‘pure space’ – that is, matter.”

Pure Perception
Bergson’s notion of pure perception stands as an alternative to realist (perception allows for unmediated access to information afforded by the environment) and idealist (perception depends on our prior knowledge of the world) epistemologies. His proposal? We see ‘images’:

“Matter, in our view, is an aggregate of ‘images’. And by ‘image’ we mean a certain existence which is more than that which the idealist calls a representation, but less than that which the realist calls a thing – an existence placed halfway between the ‘thing’ and the ‘representation’.”

We are now brought back to the “snapshot view” of change. Images are these “artificial” snapshots of the ongoing flow of life. Perception is the process of extracting these images, the means “by which the ‘fluid continuity of the real’ is artificially divided in such a way as to afford a foothold for the living being,” (69). But there is a dual nature to these images (and I have to admit, I’m a bit unclear about the leap Bergson is making here; unfortunately, it seems to be THE leap for understanding the idea of “virtualizing experience”). There is the superficial, artificial, “crust” of the image which we perceive. This is the “virtual image.” Then there is the part of the image that remains in motion, connected to the “fluid continuity of the real.”
Middleton and Brown conclude by elaborating on the notion of the virtual image. They describe it as selected aspects of the image grasped in terms of potential actions and possible reactions:

“We see things in terms of what they may or may not do to us and what we may or may not do to them. Hence, we perceive neither objectively (we do not see the ‘fluid continuity of the real) nor subjectively (what we see is most definitely ‘in’ the image), but, rather, ‘virtually’ (we extract aspects of image in accord with the demands of action.)”

 

 

 

 

 

 

 

 

 

 

 

 

 

 

 

 

 

 

 

 

 

 

 

 

 

 

 

 

 

 

 

 

 

 

 

 

Week 8 Emails

 

Memory in Madagascar, Jennifer Cole

 

From: "joanne price" (jprice@activmanuals.com)



Cole describes the focus of her article as falling at the juncture of two lines of research and discussion: social memory and colonial and postcolonial subjectivity.
Cole was interested in how the Betsimisaraka villagers of east Madagascar came to terms with a long history of domination from others—140 years of continuous domination (75 years of colonization by the Merina of central Madagascar, 1820-1895, followed by 65 years of colonization by the French, 1895-1960). Reflecting her 18 months of fieldwork, this article describes her conclusions about various mechanisms of remembering and forgetting that allowed these people to endure, though changed, as a relatively cohesive people.

Given their history of domination, Cole anticipated considerable conversation among the villagers about these events. To her surprise, they were strangely silent about these events, and would comment somewhat nonchalantly about them when queried. Cole describes the villagers’ interaction with two worlds: the Betsimisaraka world (a local world of ‘cows and ancestors,’ largely unconcerned with the events of the outside world’) [p. 619], and the outside world (the colonial world).

Her conclusion is that the mechanisms the villagers use, both of remembering and forgetting, allow them to exclude or accommodate outside forces and, except under the threat of physical violence, maintain the basic fabric of the culture.

REFLECTIONS:
This article caused me think in general about struggles of groups/organizations and the dynamics of ‘colonization/domination.’

Cole characterized a culture that persisted in the face of enormous pressure to disband/dis-integrate. How is it that some entities persist while others do not? Is it that they have become a ‘polis’ (a polity?) and thus have an internal cohesion that others do not?

Cole did not consider the Betsimisaraka a ‘polity:’

….This name is not without its ironies, for not only have Betsimisaraka never actually formed a polity [p. 628]

I’m not sure I can divide between ‘polity’ (Cole) and ‘polis.’ Argyris and Schön distinguish between a ‘collection of individuals” and an organization:

“Organizations are not merely collections of individuals, yet there is no organization without such collections. Organizational action cannot be reduced to the actions of individuals, even of all the individuals that make up the organization, yet there is no organizational action without individual action. When, then, does it make sense to say that a collection of individuals constitutes an organization that acts?

“Consider a mob of students who are holding a spontaneous protest against their university’s financial aid policy. At what point do they cease to be a mob and begin to be an organization? The mob is a collectivity, a collection of people who may run, shout, and mill about together. But it is a collectivity that cannot make a decision or take an action in its own name, and its boundaries are vague and diffuse. The mob begins to resemble an organization as it begins to meet three conditions. The individual members of the mob must:

1. devise agreed-upon procedures for making decisions in the name of the collectivity,

2. delegate to individuals the authority to act for the collectivity, and

3. set boundaries between the collectivity and the rest of the world.

“As these conditions are met, members of the collectivity begin to become a recognizable “we” that can make decisions and translate their decisions into action.

“When the members of the mob become an identifiable vehicle for collective decision and action, they become, in the ancient Greek sense of the term, a polis. Before an organization can be anything else, it must be ‘political,’ because it is as a political entity that the collectivity can take organizational action. Then it is the individuals who decide and act, but they do these things on behalf of the collectivity, as its agents. And in order for individuals to be able to decide and act in the name of the collectivity, there must be rules that determine the boundaries of the collectivity, when a decision has been made and when authority for action has been delegated to individuals. Insofar as members of a collectivity create such rules, which we call ‘constitutional,’ and become a polis, they have organized.

“The rule-making that brings organizations into being need not be conscious, and constitutional rules need not be explicit. What is essential is that the members’ behavior be rule-governed in the crucial respects.”

– Argyris and Schön, 1996, Organizational Learning II, pp. 8-9

Some cultures endure the pressure from the outside, others do not. What factors allow a ‘polis’ to endure as the Betsimisaraka. Are they better at remembering and forgetting?

Brockmeier, “Remembering and Forgetting: Narrative as Cultural Memory”

 

 From: "noga shemer" (nnsevilla@yahoo.com)

 

 

Objectives:

1) explore the dialectics of remembering and forgetting

2) question the widespread dichotomy of individual and social memory

 

Brockmeier begins by historically situating and then challenging the longstanding notion that Remembering is something positive (the hero of the “play”), and Forgetting is negative (the villain), and that the two forever dual over the high stakes of truth.  He argues that this same drama is also played out in the public cultural discourse of memory, citing the “culture of anniversaries” and proliferation of modes of public commemoration (17).  These acts of collective remembering, interweaving social and temporal dimensions, open up a symbolic space that binds individuals together and creates a sense of belonging. 

 

He then describes the Western public culture of memory as being a historical phenomenon while also creating the appearance of naturalness.  This latter attempt is a symptom of modern society which is characterized by permanent change: “the faster modern societies change and traditions of knowledge, religion, ethics and lifeworld lose influence, the more energy flows into public practices, institutions and the establishment of artifacts that conjure up ‘lasting’ cultural memories” (19).  This “memory crisis” is about the “very notion of memory” and is based on “highly culture-specific practices of remembering” (20).

 

Brockmeier points out that the entities of Remembering and Forgetting are actually inextricably interrelated, and proposes a cultural-historical perspective to “instead conceive of them as two sides of one process, a process in which we give shape to our experience, thought and imagination in terms of past, present and future” (21).  He points out that all memory constructions are based on selection – what gets left in, out, or reconfigured – and that people “remember and forget according to the memory frames and practices of the groups of which they are members” (23).  He calls these frames and practices “contexts of cultural participation.”  But how does culture define these frames, shaping the practices and notions of memory?

 

Here, Brockmeier turns to the sociohistorical and sociocultural tradition, citing Luria, Vygotsky, Bakhtin, and Lotman, among others,  to depict cultural memory as a process which itself is “culturally mediated within a symbolic space laid out by a variety of semiotic vehicles and devices” (25).  Individual memories are not really ‘individual’ but are ‘distributed’ – “we might conceive of this larger layout of memory as an array of texts, documents and other artifacts that have become intermingled with the texture of one’s autobiographical memory” (25). 

 

At this point, Brockmeier shifts to a discussion of memory practices and his focus on narrative’s critical role in constructing meaning within the symbolic space of culture.  Narrative is defined as “every text that tells a story, while a text is every meaningfully organized sign system” (32).  He argues that the multifunctional nature of narrative discourse is the key to accounting for its central role, as well as its distinctive ability to endow the “inherent historicity of human existence with cultural meanings” (27).  He turns to the Berlin Memorial to the 1933 Bookburning as an example (see pp. 28-32), and then distinguishes 3 orders of narrative which correspond to his analysis of the monument:

1.  Narrative as linguistic order.  E.g. when I tell someone the story of the memorial.  Combines the elements of scene, agent, action, intentionality, predicament, solution.  This also points to narrative’s function as “the only form in which [our most intricate constructions of temporality] can be communicated and integrated into our social life” (34). 

2.  Narrative as semiotic order.  E.g. an analysis of the monument’s material structure, medium, and relationship with surroundings.  Follows spatial and temporal dimensions of cultural memory, looking at narrative as a counter-narrative responding to its material and symbolic environment.

3.  Performative or discursive order.  E.g. the architectural and historical background of Berlin’s Bebelplatz is the only “stage” on which the memorial can be performed.  Emphasis on narrative as a process and performance of meaning with an aim.  Requires identification of the way it is situated in a local cultural context.  Narrative as performance “refers to a process of co-narration, a social process of telling and acting in which teller and listener are not stable and permanent positions but moments of an interplay whose outcome remains open” (36).    

 

In his conclusion, Brockmeier relates his two arguments – A1: narrative is an important integrating force in the mnemonic system of a culture, and A2: narrative is a particular synthesis of distinct elements.  He proposes that “narrative is such a powerful integrating force within this symbolic space precisely because of the synthesis function….  Narrative not only ‘emplots’, on all three levels, diverse elements into a whole (A1); it also interweaves, at the same time, these three orders, fusing quite diverse forms of discourse and symbolic mediation (A2)” (38). 

 

 

Middleton and Smith Ch 2 – first half – on Bartlett

 

From: nathaniel smith

This chapter begins with an introduction to the overall goal of the
book: "examin[ing] how we can approach the social psychology of
experience in the study of remembering and forgetting". They briefly
review the history of work in this area. The subject has been
considered by as diverse a range of thinkers as William James and John
Locke. In addition, it has immediate relevance to the real world,
including the recovered memory debate, the construction of truths in
politics, our "routine engagement with commemorative activities", and
so on. However, this creates a paradox -- if we take seriously the
social component of memory, it becomes unclear how to integrate this
with our feeling of memory as "intensely private and personal". This
core problem motivates the rest of the chapter, as well as the several
that follow: they will examine a range of approaches to memory
developed outside of psychology's contemporary bounds, for the
insights that they may give.

The first approach is that of Bartlett. According to the authors, his
"research career was forged in the use of psychological methods to
persue anthropological questions concerning the 'conventionalisation'
of cultural materials". Bartlett rejects the Ebbinghaus tradition of
researching memory in simplified, abstracted, controlled settings.
Instead, given that memory is oriented towards real world use, and
given that psychological experiments are themselves embedded in a
social context, we should instead use experimentation to study these
social processes directly.

This argument is lost in most present-day descriptions of Bartlett.
While psychologists accept the fact of social factors being relevant
to memory, they do not take the theoretical critique to heart;
instead, they assimilate the fact into their existing framework by
adding "social factors" as another set of independent factors that can
be included in experimental designs. Similarly, in Bartlett the word
"schema" plays a key role; while future authors would use this word in
a very impoverished sense, for him, it refers to the "ongoing dynamic
adaptation between people and their physical and social environments",
an "organised setting". The key description begins at the end of page
17:

What Bartlett tries to capture is the essential integration of
individual mentality and culture, of the interdependency of
cognition, affect and cultural symbols... For Bartlett, remembering
is indicative of the kind of liberty that schemata, as organised
settings, afford us. To exist within an organised setting is to
have some of the burden of being forced to continuously adjust to
the changing vicissitudes of the environment removed... [T]he
'special character' of human psychological functioning emerges as
the human organism 'discovers how to turn round upon its own
"schemata", or, in other words, it becomes conscious'. To be
conscious is to have a reflexive awareness of the organised setting
in which one's thoughts and actions are situated... It is this
ability to turn around on schema that constitutes remembering as a
constructive process of living development -- in other words as a
kind of ongoing dialogue between our thinking and the cultural
symbols that feature in a given organised setting.

Thus, memory is not a simple record that may be inaccurate or not;
rather, memory is our conscious access to, influence by, and control
over the experienced environment we live within. As a result, it
becomes interesting to study such things as how memory is modified and
created to meet the needs of particular situations. Critically,
conversation is pulled out as one especially important locus of this
(re)construction.

The chapter then continues by profiling four key themes from Bartlett
-- "commemoration", "conventionalization", "objectification", and
"mediation" -- describing how each relates to other work, and finally
summarising the overall approach as studying "experience situated
within organised setting". The next chapters will discuss two other
theorists, who will help us integrate this insight into social
psychology.

Middleton & Brown Ch 3 part 1

 

From: "Matt Brown" (mjb001@ucsd.edu)

 

Halbwachs was a student of both Bergson and Durkheim who wrote two major works on collective memory. While often represented as repudiating Bergson's philosophy in favor of Durkheim's empirical sociology, M&B argue for a reading which has significant, sympathetic, and productive links between Halbwachs and Bergson.

Halbwachs tries to approach both sociological and psychological audiences in his works on collective memory, arguing against an asocial approach to memory and for a social conception of the subject. The hard case here is dreaming, which is seemingly completely isolated from the social. But Halbwachs attacks the phenomenon of dreaming in two ways: (1) dreaming still utilizes language, which is fundamentally social, and which lends form to subjective experience, and (2) dreaming is an escape from the social organization of waking life (compare Latour's use of walls?).

For H, the function of memory is comparing the "collectivity" or "social milieu" from our past with the present. We achieve individuality by moving between different social groups, where the constraints and obligations are different, and comparing. So we achieve freedom by being able to look back and see that we aren't constrained now as we were, then. (And presumably we can see the contingency of our current constraints by comparing them with the past as well?)

Memory is a group activity in that it is a reconstruction of past events based on what we have in the present, and it uses resources of the group (by adopting "the perspective of the group") to do so. While "individuality" is a mode of social organization, it is still individuals, rather than groups, that do the remembering. Groups are neither the source nor the agent of remembering. H prefers to speak of impersonal "frameworks" precisely because the actual presence of the group isn't necessary to accomplish memory. Instead, making use of group memory has to do with "locating" oneself "within the framework that lent the group coherence."

Then there are some sections on localization and territorialization in this chapter that I didn't really get a good grasp on. I decided to go ahead and post this for everyone's benefit. In the meantime, I'll try to make a little more sense out of it. If not, we can hopefully talk about it in class.

From: "Matt Brown" (mjb001@ucsd.edu)

Localization - recollections must be located within the common framework, a framework that is prior to any particular act of remembering. This network is an impersonal, shared background knowledge. Our tendencies to recollect prototypical scenes, rather than particular facts or episodes, and to project the present ordering of relationships on the past are both instances of localization. The main mechanism is again linguistic.

Territorialization - the projection of social relationships or "the framework" onto natural things. (Like Marx's commodity fetishism?)

 

 

Middleton & Brown Ch 3 part II

 

From: "xavier cagigas" (xcagigas@ucsd.edu)

The 'implacement' of frameworks

The question posed by this section is:  how do frameworks persist in the absence of living group members?  The spatial and physical dimension of collective frameworks is explored here by developing how groups leave their imprint on the spaces they inhabit.  It is a given that a reciprocal relationship exists between groups and the spaces they live in such that once groups carry out their cultural practice in a given setting, the physical space itself begins to take on meaning which further reinforces particular practices in the group. 
Since a constant, dynamic back and forth exists between group and space, the fact that spaces change more slowly in effect slows down the interplay and gives the illusion of constancy.  This functional "slowing down" by physical artifacts/spaces is a central component of H's argument. 

 

Objects as markers of relationships

The example of a table being a place where one eats, studies, or discusses family matters illustrates how the physical object of "table" and the collective space that it creates are proxies for entire activity scripts that represent group relationships.  An elaborate and informative example is given regarding the value of objects and how "prices" of objects are set.  The price of an object represents a negotiation between buyer and seller such that various social processes are "slowed down" and crystalized for the moment.  It is not simply that objects activate collective memories, but rather that they initiate relationships which must be played out in order to be resolved (i.e. schemas?).

 

Displacement, disposal, and forgetting

The ways in which collective spaces and groups/individuals change is explored in the dialectic between remembering and forgetting.  What happens when a collective space is destroyed or a collective dies out?  The focus on the dynamic process is well stated in the following:

"Remembering is, then, the manner in which the framework is able to extend itself within this network, to expand and distribute the elements it encompasses in order to achieve the effect of an 'expanded impersonal duration' for members.  Forgetting -- the corresponding dynamic -- is the way in which the framework simultaneously contracts within the network, ramifies and consolidates its elements and then redraws the boundaries and intersections with neighbouring networks."

In order to 'remember' anything certain things need to be 'forgotten.'  Selective remembering and forgetting allows one to go back and forth between the present and the past and to find meaning in certain collective spaces and not others. 

 

Selfhood as multiplicity

In this section, the sense of self is deconstructed to reveal the intersection of two collective frameworks.  Even our most intimate, private, and individual experiences simply reflect the juxtaposition and active negotiation of the frameworks that we participate in.  Our subjective experience of self, again, in this case a 'perceived' slowing down of this dynamic negotiation between collective frameworks.

 

Observation:

I recently read a few articles by Nisbett (Miyamoto, Nisbett, & Masuda, 2006; Nisbett & Miyamoto, 2005; Chua, Boland, & Nisbett, 2005) that seem to explore the tight coupling between groups and their collective spaces.  The authors make the argument that specific collective spaces go on to shape specific patterns of attention (i.e. eye movements, the number of objects incorporated into categorization judgments) which can be differentiated between groups.  The articles are readily available in pdf if anyone is interested.

 

Chpt. 4: Middleton and Brown: Virtualising Experience: Henri Bergson on Memory

From: "Robert Lecusay" (rlecusay@ucsd.edu)


This chapter offers a re-evaluation of the thought of the French philosopher Henri Bergson. The focus here is on Bergson’s concept of memory.

Reality as Process
Middleton and Brown begin by introducing Bergson’s “process philosophy” in contrast to traditional Western analytic philosophy, which they describe as concerned primarily with things rather than with actions and events. A commitment to this latter philosophy implies an understanding of memory as “traces . . . encoded” in multiple brain structures, of memory as neural structure.
Alternatives to this “substance” centered philosophy fall short. These alternatives can bee seen as ultimately describing processes that occur among mental substances, bringing us back to the task of having to address the study of mind as the search for where these “mental things” are stored, their true nature, and the process of how these things “enter” into the mind.
What then is reality for the process philosopher (Bergson specifically)? Becoming. “For the process philosopher,” argue Middleton and Brown, “all ‘things’ are actually part of the process of becoming – that is, changing, moving, and transforming – so what is real is change itself,” (61). However, Bergson would argue, it is not our perception of change that seems real to us, but rather our perception of snapshots of this change. Form and qualities represent a “snapshot view of transition,” and our perception of these qualities depends on our particular perspective, on our particular intellect which is a reflection of our biases.

The Time of Living: Duration
If we accept the notion of change as reality, we must consider the hypothetical of a world in perpetual change. In other words, a world where it is impossible to recognize any regularities in this change. Bergson responds with notion of duration. “Duration is the experience of time passing;” Bergson “insists . . . that it be considered as an indivisible, continuous flow of conscious states.” This flow which Bergson describes as “all that is naturally experienced,” stands in contrast with the snapshots of transition – the “things and states,” that are “treated . . . as products or outcomes arrived at by artificially cutting into the flow,” (62-63).
Bergson argues that part of what makes it difficult for us to conceptualize existence in these terms is the fact that our language is not suitable for talking about duration. It favors “substantives over relationships, nouns over prepositions.” The contrast here is between language as a tool for communicative action vs. language, traditionally viewed, as a neutral medium for representing experience. With respect to this latter form of representation, Bergson is interested in particular in how language reflects “a general human tendency to think of the world as an empty homogenous medium in which entities occupy unique spatial positions.” This tendency to divide the world up into parts is derived from our practical experience. Consequently, “our intellect and the language in which it is expressed have an entirely practical bent that is modeled on the use we make of objects (1998, ix): ‘our logic is, pre-eminently, the logic of solids.’” This reflects a central tension for Ber! gson, that between our experience of space and time.
The experience of space implies an experience of objects in space which for Bergson represents an obstacle in the path to gaining “purchase on the fluid continuity of duration.” At this point in the reading the terms “pure” duration and “pure” space are introduced; however, no clear definition is given of either. Instead we are told that “at the practical level,” both experiences are collapsed together. The commingling of the two is referred to by Bergson as “endosmosis.” It is just before the close of this section that we are given something of a definition of the “pure”: “The crucial point Bergson makes is that we simply will not adequately understand this mixing of experience . . . unless we have first understood the nature of ‘pure duration’ and ‘pure space’ – that is, matter.”

Pure Perception
Bergson’s notion of pure perception stands as an alternative to realist (perception allows for unmediated access to information afforded by the environment) and idealist (perception depends on our prior knowledge of the world) epistemologies. His proposal? We see ‘images’:

“Matter, in our view, is an aggregate of ‘images’. And by ‘image’ we mean a certain existence which is more than that which the idealist calls a representation, but less than that which the realist calls a thing – an existence placed halfway between the ‘thing’ and the ‘representation’.”

We are now brought back to the “snapshot view” of change. Images are these “artificial” snapshots of the ongoing flow of life. Perception is the process of extracting these images, the means “by which the ‘fluid continuity of the real’ is artificially divided in such a way as to afford a foothold for the living being,” (69). But there is a dual nature to these images (and I have to admit, I’m a bit unclear about the leap Bergson is making here; unfortunately, it seems to be THE leap for understanding the idea of “virtualizing experience”). There is the superficial, artificial, “crust” of the image which we perceive. This is the “virtual image.” Then there is the part of the image that remains in motion, connected to the “fluid continuity of the real.”
Middleton and Brown conclude by elaborating on the notion of the virtual image. They describe it as selected aspects of the image grasped in terms of potential actions and possible reactions:

“We see things in terms of what they may or may not do to us and what we may or may not do to them. Hence, we perceive neither objectively (we do not see the ‘fluid continuity of the real) nor subjectively (what we see is most definitely ‘in’ the image), but, rather, ‘virtually’ (we extract aspects of image in accord with the demands of action.)”

 

Boesch, “The Sound of the Violin”

 

From: "noga shemer" (nnsevilla@yahoo.com)

 

Boesch uses the violin as an example of “man’s relationship with the objects surrounding him and, thus, with the culture in which he lives” (6).  His analysis is divided into three sections: the “phylogenesis” of the violin (i.e. its creation), its “ontogenesis” (how it becomes an instrument to be played), and the goal of producing a beautiful sound. 

 

The creation of the violin required three discoveries, and these required “a reorientation of attention and perception, a restructuring of actions, and thereby opened to man a new dimension of action, the aesthetic one” (6).  The many experiments in violin design were guided by materials and the human possibilities of handling (7).   The goal for these design efforts seems elusive: a beautiful sound which, while pursued, was unknown.  Boesch calls this a “should-value” (8), and notes that the sound goal would vary through history and between cultures. 

 

The violin “comes into being” only on being played.  The 5-7 year old child (the optimum age for starting) is shaped by the process of learning to play the violin: “learning to master an object implies shaping the development of the individual,” including sensory and motor adaptations (8).  In order to understand the child’s motivations (especially through the frustrations and harsh sounds of a beginner), Boesch argues that the player becomes an important audience: he must satisfy himself in pursuit of this “beautiful sound” goal. 

 

Boesch distinguishes between making objects produce a sound and the “search for perfection of sound” – a moment in which the resistant non-I is transformed into a compliant extension of the I (10), a utopic moment.  Boesch points out that notions of beauty vary through history and cultures, and distinguishes sound from noise.  He characterizes the pursuit of beautiful sound as a “truly cultural endeavor” which “belongs to the ideational realm of purity” (11).  The child learner will have to side with purity against “the natural propensity for noise and dirt” (11).  Thus, learning the violin impacts the self and the child’s view of the world.  The learner will be drawn into cultural myths about purity and sound, w! hile also constructing his “personal meanings by merging them with subjective experiences and aspirations” (12).

 

Sound is also discussed as a “trace” – “the material imprints of our actions in the world” (13).  The beauty of sound is therefore felt as symptomatic of the self which is able to create it.  “Mastery of the instrument turns into mastery of oneself” (13).  Yet the trace left by music is impermanent, and music, therefore, must be constantly re-created.

 

In his conclusion, Boesch notes the multiple levels of his analysis and the larger implications.  The sound of the violin “exemplifies the cultural as well as individual construction of objects; it demonstrates the extent to which these processes do not simply produce some isolated mastery, but systems of meaning.  Mastery is not independent of goals, and goals are polyvalent and anchored in networks of coordinated action, of thought, belief, rules and values.  More than that, objects are in movement, they change with the flow of culture on the one hand, and with the nature and progress of individual actions on the other” (15). 

 

 

 

 

 

 

 

 

 

 

 

 

 

 

 

 

 

 

 

 

 

 

 

 

 

 

 

Week 9 Emails

 

Michael Bamberg “Positioning”

From: "antonieta mercado" (amercado@weber.ucsd.edu)


Punch-line: In his article Positioning with Davie Hogan, Stories, Tellings and Identities, Michael Bamberg argues that when engaging in story-telling individuals position themselves into the story, the plot, the characters, but also into the context where the story is being told. In that exercise, there is a simultaneous re-construction of the story and a process of identity formation/reaffirmation for the story-teller and its audience. This process goes back and forth from person to the world-world to the person.

Bamberg points out that positioning analysis helps to understand that narratives are situated actions. They are about the self of the speaker, and indexical regarding the speaker’s subjectivity. Therefore, narratives have to be analyzed as performed situated actions, and identity as well as subjectivity have to be taken into account because positions are grounded in discourses or master narratives, plot lines, dominant discourses, or cultural texts, which at the same time provide the meanings and values for the subjects to be “positioned” (p. 1). Nevertheless, the subject still keeps agency in the sense that she can pick and choose a position among the ones that are available, turning “positions” into an agentive resource or repertoire. Despite the predetermination of discursive frames, Bamberg focuses a great deal of agency to the individual when “positioning” within a particular angle or characteristic of the story. He also argues that isolating the subject in a story-tel! ling situation can be more complex than originally perceived, because the context and the person are being constantly re-made in the process of story-telling.


The analysis of how speakers actively and agentively position themselves in talking, in particular with their stories, assumes that the orderliness of story-talk is situationally interactively accomplished.

Stories gain their structure and content from their situatedness in interactional settings but also with the action of the subject or story-teller and her decision to emphasize certain parts of the story over others.

In his analysis of the story of Davie Hogan and how it is being told in the movie Stand by Me on a campfire in the woods where four adolescents are searching for a missing boy’s dead body. While in the campfire, the four 12 year old boys, engage in smoking, and story-telling. Gordie, one of the adolescents, engages in telling the story of Davie Hogan, a fat and apparently unpopular boy who engages in a pie-eating contest that turns into a barfing revenge against people that put him down. In the movie, at the time when Gordie starts to tell the story, the images shift from the campfire and go directly to the fictional Davie Hogan setting. Bamberg asked two students to replace this part by two written narratives intended to be from the point of view of Gordie, the main narrator in the movie. Bamberg proposes three levels of analysis for the story and the context:


1) How characters are designed in time and space inside story worlds in order to get an initial hold of the “identity claims” of the teller (positioning analysis level 1).

2) scrutinize the interactional means employed for getting the story accomplished (positioning analysis level 2).

3) How speakers and audiences establish and display particular notions of selves (positioning analysis level 3).

The first 2 levels of analysis are to differentiate how speakers work-up the construction of normative discourses. Analyzing stories in interactions is useful to see the interaction between the subject being determined by an existing discourse, and the other is to see how the subject is the ground from where discourses are constructed.

Bamberg analyzes carefully the data of the story, the passive and active clues that bring the principal character (Davie) to the front, and position other characters as background, such as the crowd in the town where the eating contest is taking place. In level one, Bamberg shows how the story and its characters are laid out, in the second level, he shows how Gordie engages in story-telling with his friends at the campfire, and positions himself within the story and the narration context (4 male adolescents in a campfire). Bamberg also pays attention to the reaction of the other adolescents to Gordie’s story and how they con-construct the story when asking why it ended as it did, and what happened after Davie Hogan made the participants and attendants of the pay context barf. In the third level, Bamberg points out that the adolescents in the campfire highlight the “barfing” as a revenge part of the story as a way to assert themselves (smoking cigarettes, telling disgusting ! stories, asserting their “maleness” doing something that they are not supposed to do at their age).



Positioning then is linked to story-telling but also to identity construction for both the teller and the listener (s).

 

Bruner, Two Modes of Thought

 

From: "xavier cagigas" (xcagigas@ucsd.edu)

This section is summed up by the following quote:  "discourse...narrative speech acts...must depend upon forms of discourse that recruit the reader's imagination -- that enlist him [sic] in the 'performance of meaning under the guidance of the text.'  Discourse must make it possible for the reader to 'write' his own virtual text."

3 key features to discourse:

1) presupposition - "the creation of implicit rather than explicit meaning"

2) subjectification - "the depiction of reality not through an omniscient eye that views a timeless reality, but through the filter of the consciousness of protagonists in the story"

3) multiple perspective - "beholding the world not univocally but simultaneously through a set of prisms each of which catches some part of it"

* bonus) metaphor - another device employed in discourse that "keeps meaning open or 'performable' by the reader"

The above devices create a "subjunctive reality" -- trafficing in human possibilities which are co-constructed by the discursive text and the reader.  Bruner goes on to give examples of how one can phrase things in order to mean more than what is said such that the reader must fill in the gaps and is therefore recruited and invested in the meaning-making process.

Bruner suggests that discursive texts form a sort of scaffolding upon which the reader actively constructs their own meaning pulling both from the text itself and from their own experiential stock of meanings.  In this way, it becomes clear how discursive texts serve as mediators.

Bruner explores this process through various examples in which the reader employs both a 'strategy' and 'repetoire' for engaging the text.  Great literature is 'accessible' and allows the reader to 'rewrite' the story using imagination and thus creating meaning.  It's as if the text serves to catapult the reader into meaning, but the direction of flight and even the final destination is dependent on what the reader adds and eventually forges out of the text and what they bring to it.

From here, Bruner shifts to describing 'character' in a way that taps on the reader's own 'theory of mind' -- a theory of how people are, which is inclusive of both states and traits.  In other words, he develops the tension between the agency that purportedly dwells within a person and and yet is also distributed in the circumstances which they inhabit (i.e. 'the inseparability of character, setting, and action' -- all of which the reader simultaneously brings to and extracts fromt the text)

He then makes an attempt to understand this multifaceted construct by examining it in layers a la Rorty similar to other discursive psychologists: characters, figures, persons, selves, individuals

However, "we are different entities as we conceive ourselves enlightened by these various views.  Our powers of action are different, our relations to one another, our properties and prorieties, our characteristic defeats, our conception of society's proper strictures and freedoms will vary with our conceptions of ourselves as characters, persons, selves, individuals...each is a mode of interpreting as well as a mode of depiction, and in both, the lines are not clear."

And so 'positioning' shifts...  "it is this alternativeness -- this inherent restlessness in deciding on the right depiction of personhood -- that gives the novel of character, the psychological novel, its force, its subjunctivity, and its power to disturb."

Toward the end, Bruner introduces the question of whether this approach (e.g. narrative) leaves the door open for 'errors' which a more "logico-scientific method" avoids.  However, he then suggests that all thinking involves meaning making, and while some forms may be more 'varifiable' they are still created by human beings...  "Can anyone say a priori that history is completely independent of what goes on in the minds of its participants?"  Here he seems to lay bare the post-modern subjectivity that has come to characterize two modes of thought...?

From: "beth ferholt" (bferholt@weber.ucsd.edu)

I did not find the structure of this chapter strange: perhaps this has to do with the fact that Bruner is writing for (at least) two audiences, the psychologists and the literary theoriest, and, as he predicts, what some of us quarrel with the others of us find bizarrely obvious.
On this topic, the reason he can forcus on the narrative when the title of the chapter, and introduction, include the narrative and the paradigmatic is, as he tell us in the final paragraph of the chapter, because narrative is used to create the reality of both art and life. You can't have one mode of thought without the other, so to talk about one is to talk about both.

From: "beth ferholt" (bferholt@weber.ucsd.edu)

P. 15 -- In response to the question of how studying the work of trained and gifted writers is about studying mediation, I think the idea is, as we read in Wertsch, that the Narrative mediates.

 

Bruner, Two Modes of Thought – 2nd half

From: "xavier cagigas" (xcagigas@ucsd.edu)

This section is summed up by the following quote:  "discourse...narrative speech acts...must depend upon forms of discourse that recruit the reader's imagination -- that enlist him [sic] in the 'performance of meaning under the guidance of the text.'  Discourse must make it possible for the reader to 'write' his own virtual text."

3 key features to discourse:

1) presupposition - "the creation of implicit rather than explicit meaning"

2) subjectification - "the depiction of reality not through an omniscient eye that views a timeless reality, but through the filter of the consciousness of protagonists in the story"

3) multiple perspective - "beholding the world not univocally but simultaneously through a set of prisms each of which catches some part of it"

* bonus) metaphor - another device employed in discourse that "keeps meaning open or 'performable' by the reader"

The above devices create a "subjunctive reality" -- trafficing in human possibilities which are co-constructed by the discursive text and the reader.  Bruner goes on to give examples of how one can phrase things in order to mean more than what is said such that the reader must fill in the gaps and is therefore recruited and invested in the meaning-making process.

Bruner suggests that discursive texts form a sort of scaffolding upon which the reader actively constructs their own meaning pulling both from the text itself and from their own experiential stock of meanings.  In this way, it becomes clear how discursive texts serve as mediators.

Bruner explores this process through various examples in which the reader employs both a 'strategy' and 'repetoire' for engaging the text.  Great literature is 'accessible' and allows the reader to 'rewrite' the story using imagination and thus creating meaning.  It's as if the text serves to catapult the reader into meaning, but the direction of flight and even the final destination is dependent on what the reader adds and eventually forges out of the text and what they bring to it.

From here, Bruner shifts to describing 'character' in a way that taps on the reader's own 'theory of mind' -- a theory of how people are, which is inclusive of both states and traits.  In other words, he develops the tension between the agency that purportedly dwells within a person and and yet is also distributed in the circumstances which they inhabit (i.e. 'the inseparability of character, setting, and action' -- all of which the reader simultaneously brings to and extracts fromt the text)

He then makes an attempt to understand this multifaceted construct by examining it in layers a la Rorty similar to other discursive psychologists: characters, figures, persons, selves, individuals

However, "we are different entities as we conceive ourselves enlightened by these various views.  Our powers of action are different, our relations to one another, our properties and prorieties, our characteristic defeats, our conception of society's proper strictures and freedoms will vary with our conceptions of ourselves as characters, persons, selves, individuals...each is a mode of interpreting as well as a mode of depiction, and in both, the lines are not clear."

And so 'positioning' shifts...  "it is this alternativeness -- this inherent restlessness in deciding on the right depiction of personhood -- that gives the novel of character, the psychological novel, its force, its subjunctivity, and its power to disturb."

Toward the end, Bruner introduces the question of whether this approach (e.g. narrative) leaves the door open for 'errors' which a more "logico-scientific method" avoids.  However, he then suggests that all thinking involves meaning making, and while some forms may be more 'varifiable' they are still created by human beings...  "Can anyone say a priori that history is completely independent of what goes on in the minds of its participants?"  Here he seems to lay bare the post-modern subjectivity that has come to characterize two modes of thought...?

Stanley and Billig: Dilemmas of storytelling and identity

 From: "emma johnson" (ekjohnso@ucsd.edu)


In this fairly short chapter, the authors introduce discursive psychology, and then use the method to analyze a segment of an interview that one author conducted with a graduate student, Hanako, who is in her last year of study. The interview is one of 15 that were conducted as part of a larger study. The analysis is concerned with the power relations between faculty and graduate students, but they use specific, situated exchanges to do this.

The authors explain that discursive psychology is both a theoretical position closely related to social construction, and an empirical method of closely analyzing talk. They are particularly interested in identity claims and ideological dilemmas. These can be observed by paying close attention to the telling of stories. The example that they choose here is a story about submitting an article for publication, an article that has not been accepted at the time, and needs further revision.

There are (broadly) two particular rhetorical strategies examined and detailed by Stanley and Billig: the use of pronouns; and the sequential ordering of the narrative, particularly in respect to pronoun use. In this example, Hanako shifts between using the first person pronoun (I) and the plural (we) in describing a chapter submitted for publication, and in this shifting back and forth, we can observe an ambivalence or tension between autonomy and dependency. Also, she begins with the first person, prioritizing her contribution, but shifts to the plural, and then back again, at one point even correcting herself, beginning with “I” and then changing it to “we”. The order in which she introduces the story, and the ordering of her use of singular and plural is significant, since she is constantly striking a balance between her roles and her relationship to her advisor.

The authors conclude by pointing out that this chapter, about the tensions and balance struck in coauthoring, is itself coauthored, which suggests that discursive psychology could be practiced on the text itself.

I am struck reading this by something I often notice with this kind of microanalysis—that in order to understand the meaning of these small shifts, you must already be aware of the context, of the ideological tensions. This point is rather obvious and often noted, but I’ll just make it again anyway. I am not trying to suggest that it invalidates their findings; actually, while I am not a discursive psychologist, or discourse analyst I do think it is important to attend to this kind of meaning-making in writing and speaking. But maybe it just bolsters my suspicion that there just can’t only one correct unit of analysis.

I also worry that in doing this sort of research, the researcher will be attuned to something in particular, and that is what they will elucidate, draw out, etc from the material. Here it’s the power relation between faculty and student. But what if it was gender, or nationality/ethnicity, for instance, would these shifts in pronoun use be marshaled to say something about that? (Gender is brought up on numerous occasions in the chapter, and the choice of the pseudonym Hanako signifies that the subject probably is not British-born, or at least her parents probably are not.) Again, a rather worn-out critique, and not one even has to be a problem for this method. The authors might be interested in detailing the different kinds of identity claims that are simultaneously made in conversational use of pronouns, although that sounds like it could become a really biig job.

-Emma

Note: I found the concluding sentence funny: “That is why academic work cannot escape the reach of ideology.” Seems a bit optimistic, to suggest that we were thinking our work escapes the reach of ideology? Do you all think that about your work?